You are on page 1of 115

Dr.

Fried Katalin

Dr. Gercs Lszl

Szmad Lszl

MATEMATIKA 9.
A tanknyv feladatai s a feladatok megoldsai
A megoldsok olvasshoz Acrobat Reader program szksges, amely ingyenesen letlthet az internetrl (pldul: adobe.la.hu weboldalrl). A feladatokat fejezetenknt kln-kln fjlba tettk. A fejezet cmmel elltott fjl tartalmazza a fejezet leckinek vgn kitztt feladatok rszletes megoldsait. A feladatokat nehzsgk szerint jelltk: K1 = kzpszint, knnyebb; K2 = kzpszint, nehezebb; E1 = emelt szint, knnyebb; E2 = emelt szint, nehezebb feladat. Lektorok: PLFALVI JZSEFN KONCZ LEVENTE Tipogra: LRINCZ ATTILA Szakgraka: DR. FRIED KATALIN Dr. Fried Katalin, Dr. Gercs Lszl, Szmad Lszl, Nemzeti Tanknyvkiad Zrt., 2009 Nemzeti Tanknyvkiad Zrt. www.ntk.hu Vevszolglat: info@ntk.hu Telefon: 06 80 200 788 A kiadsrt felel: Kiss Jnos Tams vezrigazgat Raktri szm: RE 16102 Felels szerkeszt: Szloboda Tiborn Mszaki igazgat: Babicsn Vasvri Etelka Mszaki szerkeszt: Marcsek Ildik Grakai szerkeszt: Grg Istvnn, Mikes Vivien Terjedelem: 14,9 (A/5) v 1. kiads, 2010

MATEMATIKA

Tartalom

Jelmagyarzat . . . . . . . . . . . . . . . . . . . . . . . . . . . . . . . . . . . . . . . . . . . . . . . . . . . . . . . . I. Halmazok 1. Halmazok, jellsek . . . . . . . . . . . . . . . . . . . . . . . . . . . . . . . . . . . . . . . . . . . . . 2. Specilis halmazok, intervallumok . . . . . . . . . . . . . . . . . . . . . . . . . . . . . . . . . . . 3. Halmazok unija, metszete . . . . . . . . . . . . . . . . . . . . . . . . . . . . . . . . . . . . . . . 4. Halmazok klnbsge, komplementer halmaz . . . . . . . . . . . . . . . . . . . . . . . . . 5. A matematikai logika elemei . . . . . . . . . . . . . . . . . . . . . . . . . . . . . . . . . . . . . . II. Algebra s szmelmlet 1. A hatvnyozs s azonossgai . . . . . . . . . . . . . . . . . . . . . . . . . . . . . . . . . . . . 2. A hatvnyozs azonossgainak kiterjesztse . . . . . . . . . . . . . . . . . . . . . . . . . . 3. Gyakorlati szmtsok . . . . . . . . . . . . . . . . . . . . . . . . . . . . . . . . . . . . . . . . . . 4. Algebrai kifejezsek sszevonsa, szorzsa . . . . . . . . . . . . . . . . . . . . . . . . . . . 5. Nevezetes szorzatok . . . . . . . . . . . . . . . . . . . . . . . . . . . . . . . . . . . . . . . . . . . . 6. Tovbbi nevezetes szorzatok (Emelt szint) . . . . . . . . . . . . . . . . . . . . . . . . . . . . 7. sszegek szorzatt alaktsa . . . . . . . . . . . . . . . . . . . . . . . . . . . . . . . . . . . . . 8. Algebrai trtek egyszerstse, sszevonsa . . . . . . . . . . . . . . . . . . . . . . . . . . 9. Algebrai trtek szorzsa, osztsa, sszetett mveletek algebrai trtekkel . . . . 10. Oszthatsg . . . . . . . . . . . . . . . . . . . . . . . . . . . . . . . . . . . . . . . . . . . . . . . . . . 11. Prmszmok, a szmelmlet alapttele . . . . . . . . . . . . . . . . . . . . . . . . . . . . . . 12. Legnagyobb kzs oszt, legkisebb kzs tbbszrs . . . . . . . . . . . . . . . . . . 13. Osztk szma, ngyzetszmok (Emelt szint) . . . . . . . . . . . . . . . . . . . . . . . . . . 14. Szmrendszerek . . . . . . . . . . . . . . . . . . . . . . . . . . . . . . . . . . . . . . . . . . . . . . . III. Fggvnyek, sorozatok 1. Hozzrendelsek, fggvnyek . . . . . . . . . . . . . . . . . . . . . . . . . . . . . . . . . . . . . 2. Ponthalmazok a koordinta-rendszerben . . . . . . . . . . . . . . . . . . . . . . . . . . . . . 3. A lineris fggvny . . . . . . . . . . . . . . . . . . . . . . . . . . . . . . . . . . . . . . . . . . . . . . 4. Az abszoltrtk-fggvny . . . . . . . . . . . . . . . . . . . . . . . . . . . . . . . . . . . . . . . . 5. Az f : x 7 x2 fggvny . . . . . . . . . . . . . . . . . . . . . . . . . . . . . . . . . . . . . . . . . . . 6. A msodfok fggvny sszetett transzformcii . . . . . . . . . . . . . . . . . . . . . . . 7. Tovbbi fggvnyek . . . . . . . . . . . . . . . . . . . . . . . . . . . . . . . . . . . . . . . . . . . . . IV. Bevezets a geometriba 1. Pontok, egyenesek, skok . . . . . . . . . . . . . . . . . . . . . . . . . . . . . . . . . . . . . . . . 2. Szakasz, flegyenes, szg . . . . . . . . . . . . . . . . . . . . . . . . . . . . . . . . . . . . . . . . 3. Hromszgek . . . . . . . . . . . . . . . . . . . . . . . . . . . . . . . . . . . . . . . . . . . . . . . . . 4. Tovbbi sszefggsek a hromszg alapadatai kztt . . . . . . . . . . . . . . . . . . 5. sszefggs a derkszg hromszg oldalai kztt . . . . . . . . . . . . . . . . . . . 6. Geometriai szmtsok . . . . . . . . . . . . . . . . . . . . . . . . . . . . . . . . . . . . . . . . . . 7. Geometriai szerkesztsek . . . . . . . . . . . . . . . . . . . . . . . . . . . . . . . . . . . . . . . . 8. Thalsz-ttel . . . . . . . . . . . . . . . . . . . . . . . . . . . . . . . . . . . . . . . . . . . . . . . . . . 9. A hromszg oldalfelez merlegesei s kr rt kre . . . . . . . . . . . . . . . . . . . 10. A hromszg szgfelezi, bert s hozzrt krei . . . . . . . . . . . . . . . . . . . . . . 11. Sokszgek . . . . . . . . . . . . . . . . . . . . . . . . . . . . . . . . . . . . . . . . . . . . . . . . . . .

7 9 11 12 14

17 17 18 19 20 22 23 24 26 28 29 30 31 32

35 37 40 43 46 47 49

55 56 58 60 61 62 64 66 67 70 72

9 .

V F O LYA M

MATEMATIKA

TA R TA LO M

V. Egyenletek, egyenletrendszerek 1. Elsfok egyismeretlenes egyenletek . . . . . . . . . . . . . . . . . . . . . . . . . . . . . . . . 2. Szveges feladatok megoldsa egyenletekkel . . . . . . . . . . . . . . . . . . . . . . . . . . 3. Egyenletek megoldsi mdszerei . . . . . . . . . . . . . . . . . . . . . . . . . . . . . . . . . . . 4. Egyenltlensgek . . . . . . . . . . . . . . . . . . . . . . . . . . . . . . . . . . . . . . . . . . . . . . . 5. Abszolt rtket tartalmaz egyenletek, egyenltlensgek . . . . . . . . . . . . . . . . 6. Elsfok ktismeretlenes egyenletrendszerek s megoldsuk behelyettest mdszerrel . . . . . . . . . . . . . . . . . . . . . . . . . . . . . . . . . . . . . . . . . . . . . . . . . . . . 7. Elsfok ktismeretlenes egyenletrendszerek megoldsa egyenl egytthatk mdszervel . . . . . . . . . . . . . . . . . . . . . . . . . . . . . . . . . . . . . . . . . 8. Elsfok ktismeretlenes egyenletrendszerek megoldsa grakus mdszerrel . . 9. Egyenletrendszerrel megoldhat szveges feladatok . . . . . . . . . . . . . . . . . . . . .

75 76 78 80 82 84 85 86 87

VI. Geometriai transzformcik 1. Nhny geometriai transzformci . . . . . . . . . . . . . . . . . . . . . . . . . . . . . . . . . . 89 2. Egybevgsgi transzformcik a skon . . . . . . . . . . . . . . . . . . . . . . . . . . . . . . 91 3. Alakzatok egybevgsga . . . . . . . . . . . . . . . . . . . . . . . . . . . . . . . . . . . . . . . . 94 4. Szimmetria . . . . . . . . . . . . . . . . . . . . . . . . . . . . . . . . . . . . . . . . . . . . . . . . . . . . 96 5. Tovbbi nevezetes pontok s vonalak a hromszgben . . . . . . . . . . . . . . . . . . . 97 6. Vektorok . . . . . . . . . . . . . . . . . . . . . . . . . . . . . . . . . . . . . . . . . . . . . . . . . . . . . 98 7. Ponthalmazok . . . . . . . . . . . . . . . . . . . . . . . . . . . . . . . . . . . . . . . . . . . . . . . . . 100 8. Szg, krv, krcikk . . . . . . . . . . . . . . . . . . . . . . . . . . . . . . . . . . . . . . . . . . . . . 104 VII. Kombinatorika 1. Sorrendek . . . . . . . . . . . . . . . . . . . . . . . . . . . . . . . . . . . . . . . . . . . . . . . . . . . . 105 2. Leszmllsok . . . . . . . . . . . . . . . . . . . . . . . . . . . . . . . . . . . . . . . . . . . . . . . . . 106 VIII. Statisztika 1. Adatok gyjtse, rendszerezse, jellemzse . . . . . . . . . . . . . . . . . . . . . . . . . . . 109 2. Adatok szemlltetse . . . . . . . . . . . . . . . . . . . . . . . . . . . . . . . . . . . . . . . . . . . . 110 3. A ktarc statisztika . . . . . . . . . . . . . . . . . . . . . . . . . . . . . . . . . . . . . . . . . . . . . 115

9.

V F OLYAM

MATEMATIKA

Jelmagyarzat

Az A pont s az e egyenes tvolsga: d(A; e) vagy Ae Az A s B pont tvolsga: AB vagy AB vagy d(A; B) Az A s B pont sszekt egyenese: e(A; B) Az f1 s f2 egyenesek szge: B (f1; f 2) vagy (f1; f 2) B A C cscspont szg, melynek egyik szrn az A, msik szrn a B pont tallhat: ACBB A C cscspont szg: CB Szg jellse: a, b, c, f Az A, B s C cscsokkal rendelkez hromszg: ABC9 Az ABC9 terlete: T(ABC) vagy TABC Az a, b s c oldal hromszg fl kerlete: s = a+b+c 2 A derkszg jele: * Az e egyenes merleges az f egyenesre: e = f Az e egyenes prhuzamos az f egyenessel: e < f Egybevgsg: ,; ABC9 , Al Bl C l 9 A hasonlsg arnya: m Az A pontbl a B pontba mutat vektor: AB Egyenl, nem egyenl: =, ! ; a = 2, b ! 5 Azonosan egyenl: / ; a + b / 5 Kzeltleg egyenl: . ; a . 2,3 ; 8,54 . 8,5 Kisebb, kisebb vagy egyenl: <, #; 2 < 3, 5 # x Nagyobb, nagyobb vagy egyenl: >, $; 6 > 4, a$2 A termszetes szmok halmaza: N; {0; 1; 2; } Az egsz szmok halmaza: Z; {; 2; 1; 0; 1; 2; } A pozitv, a negatv egsz szmok halmaza: Z+, Z; {1; 2; 3; }, {1; 2; 3; }

A racionlis, az irracionlis szmok halmaza: Q, Q* A pozitv, a negatv racionlis szmok halmaza: Q+, Q A vals szmok halmaza: R A pozitv, a negatv vals szmok halmaza: R+, R Eleme, nem eleme a halmaznak: !, "; 5 ! N , -2 g Z+ Rszhalmaz, valdi rszhalmaz: 3, 1; A 3 R , N1Q Nem rszhalmaza a halmaznak: j; Z Y 1 Q+ Halmazok unija, metszete: ,, +; A , B, A + B Halmazok klnbsge: \; A \ B res halmaz: Q, { } Az A halmaz komplementere: A Az A halmaz elemszma: A ; "0; 1; 2, = 3 Zrt intervallum: [a; b] Balrl zrt, jobbrl nylt intervallum: [a; b[ Balrl nylt, jobbrl zrt intervallum: ]a; b] Nylt intervallum: ]a; b[ Az x szm abszolt rtke: x ; -3,1 = 3,1 Az x szm egsz rsze, trt rsze: [x], {x}; [2,3] = 2, {2,3} = 0,3 Az a osztja b-nek: a b ; 2 8 Az a s b legnagyobb kzs osztja: (a, b); (4, 6) = 2 Az a s b legkisebb kzs tbbszrse: [a, b]; [4, 6] = 12 Az f fggvny hozzrendelsi szablya: f: x 7 f] x g ; f: x 7 2x + 3 vagy f] x g = y ; f] x g = 2x + 3 Az f fggvny helyettestsi rtke az x0 helyen: f (x0) ; f (5), ha x0 = 5

9 .

V F O LYA M

MATEMATIKA

I. Halmazok
1. Halmazok, jellsek
1. K1 Dntsk el, hogy halmazt adtunk-e meg az albbiakban! a) A pros termszetes szmok. b) A bartsgos emberek. c) A kerek szmok. d) A kis trtek. e) Az 1-nl kisebb pozitv trtek. Halmaz: a), e). 2. K1 rjuk fel, hogy az albbiak kzl melyek az egyenl halmazok! A = {a pozitv egyjegy pros szmok}; B = {a nem 0 pros szmjegyek}; C = {a pros szmjegyek}; D = {0, 2, 4, 6, 8}; E = {2, 4, 6, 8}; F = {2 egyjegy tbbszrsei}. A = B = E, C = D = F.

3. K1 a) Adjuk meg elemei felsorolsval a kvetkez halmazokat! A) a 3-nl nagyobb, 10-nl nem nagyobb egsz szmok; B) a 0 tbbszrsei; C) 2 egyjegy pozitv tbbszrsei; D) 30 pozitv oszti; E) a 18 s a 30 legkisebb kzs tbbszrse. b) Szemlltessk a fenti halmazokat ktfle mdon!

a) A = "4, 5, 6, 7, 8, 9, 10,; B = !0+; C = "2, 4, 6, 8,; D = "1, 2, 3, 5, 6, 10, 15, 30,; E = !90+ .
b) Mindegyik halmazt szemlltethetjk Venn-diagramon s a szmegyenes pontjaiknt. A) A

4, 5, 6, 7, 8, 9, 10

0
B) B

10

9 .

V F O LYA M

MATEMATIKA

I. HALMAZOK

C) C

2, 4, 6, 8

0
D) D

1, 2, 3, 5, 6, 10, 15, 30

0 1 2 3
E)

5 6

10

15

30

E 90

10

90

4. K1 Adjuk meg elemei egy kzs tulajdonsgval a kvetkez halmazokat! A = {2, 3, 5, 7, 11, 13, 17, 19}; B = {5, 10, 15, 20, 25, 30, 35, }; C = {3, 9, 27, 81, 243, 729, }; D = {0, 1}. A = {a legfeljebb ktjegy pozitv prmszmok}; B = {az 5 pozitv tbbszrsei}; C = {a 3 pozitv egsz kitevj hatvnyai}; D = {a 0 s az 1} = {a 2-nl kisebb termszetes szmok}. 5. E1 Igazoljuk, hogy kt racionlis szm a) sszege; b) klnbsge; is racionlis szm!

c) szorzata;

d) hnyadosa (ha van)

A racionlis szmok minden esetben felrhatk kt egsz szm hnyadosaknt. a) Az sszeadshoz kzs nevezre hozzuk a szmokat. Tovbbra is egsz szmok hnyadosai lesznek. Az sszeg nevezje a kzs nevez (egsz szm), a szmll a kt szmll sszege (egsz szmok sszege egsz szm). Ezrt az sszeg kt egsz szm hnyadosa, vagyis racionlis szm lesz. b) Ugyanezzel a gondolattal oldhat meg, csak a szmll a kt szmll klnbsge, de tovbbra is egsz szm lesz. c) A szorzat szmllja a kt szm szmlljnak, a nevez a kt szm nevezjnek a szorzata, teht egsz szm. d) A hnyados az osztand s az oszt recioproknak (ha van) a szorzata, ami szintn racionlis. 6. E2 Lehet-e egy racionlis s egy irracionlis szm a) sszege; b) klnbsge; c) szorzata; racionlis, illetve irracionlis szm?

d) hnyadosa

a) Irracionlis biztosan lehet. Ha pldul a racionlis tag 0, akkor az sszeg irracionlis. Ha az sszeg racionlis lenne, akkor a racionlis tagot kivonva belle mivel a klnbsg
9. V F OLYAM

I. HALMAZOK

MATEMATIKA

szintn racionlis , a msik tag is racionlis lenne. Ez az eset nem fordulhat el. Racionlis teht nem lehet. b) rjuk fel a racionlis szm kivonst az ellentett hozzadsval. Ekkor ugyanazt kapjuk, mint az a) esetben: mindig irracionlis. c) Irracionlis biztosan lehet. Ha pldul a racionlis tnyez 1, akkor a szorzat irracionlis. Racionlis is lehet, ha pldul a racionlis tnyez 0. Ekkor ugyanis a szorzat racionlis, mert 0. Mskpp azonban nem lehet racionlis a szorzat, klnben osztva a racionlis tnyezvel, racionlis szmot kapnnk, vagyis racionlis lenne a msik tnyez is. d) Legyen a krdses hnyados a . b nem lehet 0. Ha a = 0 , akkor 0 = a is racionlis. b b Ha sem a, sem b nem 0 s b racionlis, akkor 1 is az, ha b irracionlis, akkor 1 is az. A c) felb b adat szerint akkor a $ 1 irracionlis. b A hnyados csak abban az esetben lehet racionlis, ha a = 0 . 7. E2 Lehet-e kt irracionlis szm a) sszege; b) klnbsge; racionlis, illetve irracionlis szm?

c) szorzata;

d) hnyadosa

a) Mindkett lehet. r + ]-rg = 0 racionlis; r + r = 2r irracionlis. b) Mindkett lehet. r - r = 0 racionlis; r - ]-rg = 2r irracionlis. c) Mindkett lehet. d) Mindkett lehet.

2 $ 1 = 1 racionlis; 2
2 : 1 = 2 racionlis; 2

2 $ 3 = 6 irracionlis. 6 : 2 = 3 irracionlis.

2. Specilis halmazok, intervallumok


1. K1 brzoljuk szmegyenesen a kvetkez intervallumokat! a) ]10; 6]; b) ]3; 10[; c) ]3; 5]; d) ]4,5; 3[; e) [2,25; 7,5]; f) ]6; 3[.

a) b)

10

0 01

6 10

c) d)

01 01 4,5 7,5

e) f) 6

2,25 0 1 01

2. K1 Adjuk meg s szemlltessk a kvetkez egyenltlensgek megoldshalmazt, ha az alaphalmaz A) a termszetes szmok; B) az egsz szmok; C) a nemnegatv vals szmok halmaza! a) x < 10; b) x > 5; c) x $ 3; d) 2x < 0. a) A termszetes szmok alaphalmazn a megoldshalmaz "0, 1, 2, 3, 4, 5, 6, 7, 8, 9, . Az egsz szmok halmazn a "f, -3, -2, -1, 0, 1, 2, 3, 4, 5, 6, 7, 8, 9, . A nemnegatv vals szmok halmazn a 60;106 intervallum.

9 .

V F O LYA M

10 MATEMATIKA

I. HALMAZOK

A)

0 1 2 3 4 5 6 7 8 9
B)

...9 8 7 6 5 4 3 2 1 0 1 2 3 4 5 6 7 8 9

C)

0 1

10

b) Ha - x 2 5 , akkor x 1 -5 . [Mindkt oldalhoz hozzadunk ] x - 5g-t.] Eszerint ha egy alaphalmazon van megoldsa, az negatv. A termszetes szmok halmazban nincs megoldsa. "f, -10, -9, -8, -7, -6, . Az egsz szmok halmazn a megoldshalmaz a A nemnegatv vals szmok halmazban sincs megoldsa. B)

... 8 7 6

0 1

c) A termszetes szmok halmazn a megoldshalmaz: "3, 4, 5, 6, 7, f, , az egsz szmok halmazn "f, -8, -7, -6, -5, -4, -3, 3, 4, 5, 6, f, , vagyis az egsz szmok halmazbl elhagyva a "-2, -1, 0, 1, 2, halmazt. A vals szmok halmazn a ]-3g-nl kisebb vagy egyenl, illetve a 3-nl nagyobb vagy egyenl szmok tartoznak a megoldshalmazhoz. A)

0 1
B) C)

3 4 5 6 7 ... 3 4 5 6 7 8 ... 3

. . .8 7 6 5 4 3 3

0 1 0 1

d) x < 0, a termszetes szmok halmazn nincs megolds. Az egsz szmok halmazban a 0-nl kisebb egsz szmok. A vals szmok halmazban a 0-nl kisebb vals szmok. B) C)

. . .8 7 6 5 4 3 2 1 0 1 0 1

3. E1 Az albbi egyenltlensgek alaphalmaza a vals szmhalmaz. A megoldshalmazokat rjuk olyan sorrendben, hogy mindegyik halmaz utn kvetkez halmaz rszhalmaza legyen neki! a) x 2 > 5; b) x 10 $ 15; c) x < 10; d) 25 < x; e) x - 2 2 5 . a) A megoldshalmaz: A = " x 2 5 vagy x 1 -5, . c) A megoldshalmaz: C = ! x 2 10+ . e) A megoldshalmaz: E = " x 2 7 vagy x 1 -7, . b) A megoldshalmaz: B = ! x $ 25+ . d) A megoldshalmaz: D = ! x 2 25+ .

Ha szemlltetjk a megoldshalmazokat szmegyenesen, akkor knnyen leolvashatjuk, hogy A 2 E 2 C 2 B 2 D .


4. K2 rjuk fel az brval adott intervallumokat, illetve azt a halmazt, amely azon elemekbl ll, amelyek nincsenek az adott halmazban!

a)

x 0 1

9.

V F OLYAM

I. HALMAZOK

MATEMATIKA 11

b)

x 0 0 0 0 1 x 1 x 1 x 1
b) -2 1 x # 3, illetve " x # -2 vagy x 2 3, ; d) x 2 0 , illetve x # 0 ;

c)

d)

e)

A halmazok s prjaik: a) x 2 -5 , illetve x # -5 ; c) x 1 3,5 , illetve x $ 3,5 ; e) -1 1 x 1 1, illetve " x # -1 vagy x $ 1, .

3. Halmazok unija, metszete


1. K1 Egy sporttagozatos osztly ltszma 24 f. Az osztlyban mindenki atletizl vagy kosrlabdzik. 16-an atletizlnak, 14-en kosaraznak. Hny olyan tanul van az osztlyban, aki csak kosarazik? Ha x azoknak a szma, akik mindkt sportot zik, akkor 16 + 14 - x = 24 , ahonnan x = 6 . gy azok szma, akik csak kosaraznak: 8. 2. K1 Egy osztly minden tanulja elment a tanv hrom iskolai koncertjnek valamelyikre. Az els koncerten 12-en voltak, a msodik koncerten ugyancsak 12-en vettek rszt, a harmadik koncerten pedig 13-an. Mindhrom koncerten 3 dik vett rszt. Azok szma, akik csak egy koncerten voltak: 14. Mennyi az osztlyltszm? A feladat szvegnek megfelel halmazbra: a + b + c = 14 . 12 + 12 + 13 - ^ x + y + z h - 6 = 14 + 3 + x + y + z . Innen x + z + y = 7 . Teht az osztlyltszm: 14 + 7 + 3 = 24 . 3. K2 Legyen A halmaz a 2-vel, B halmaz a 3-mal, C halmaz a 4-gyel oszthat szmok halmaza. Ksztsnk halmazbrt, s helyezzk el benne a kvetkez szmokat: 0, 4, 6, 8, 12, 15, 18, 27, 162, 300! A megfelel halmazbra s a megadott szmok elhelyezse: 4. E1 Adjunk meg 5 halmazt gy, hogy kzlk brmely 4-nek a metszete ne legyen az res halmaz, de az t halmaz metszete az res halmaz legyen! Legyenek a, b, c, d, e klnbz vals szmok. A megfelel halmazok: A = " a, b, c, d ,; B " a, b, c, e ,; C = " a, b, d, e ,; D " a, c, d, e ,; E = " b, c, d, e , . 5. K2 Egy zeneiskola egyik vfolyamnak 56 dikja hegedlni, zongorzni vagy csellzni tanul. (Mindenki jtszik valamelyik hangszeren.) Azok szma, akik pontosan kt hangszeren jtszanak, ngyszer, akik pedig pontosan egy hangszeren jtszanak, kilencszer annyi, mint azok szma, akik mindhrom hangszeren jtszanak. Hnyan vannak azok, akik csak egy hangszeren jtszanak? Ksztsnk egy halmazbrt! A felttelek szerint a + b + c + x + y + z + h = 56,

I.(12) y a 3 x c z b

II.(12)

III.(13)

A C 4 8 0 12 300 6 18 162 B 15 27

H y a h x c z b

x + y + z = 4h,

a + b + c = 9h .
9 .

V F O LYA M

12 MATEMATIKA

I. HALMAZOK

Ezek szerint h + 4h + 9h = 56 , azaz 14h = 56 , ahonnan h = 4 . A csak egy hangszeren jtszk szma: a + b + c = 9h = 36 . 6. E1 Az iskolai traszakosztly mind a 42 tagja rszt vett az idei hrom tra valamelyikn. A msodik kirndulson 1-gyel, a harmadikon pedig 5-tel tbben vettek rszt, mint az elsn. Azok szma, akik kt trn vettek rszt, 3-szor, akik pedig egy trn vettek rszt, 10-szer annyi, mint azok szma, akik mindhrom trn rszt vettek. Hnyan vettek rszt az els, a msodik, illetve a harmadik kirndulson? a + b + c = 3h, p + q + r = 10h , teht 10h + 3h + h = 14h = 42, azaz h = 3. x + x + 1 + x + 5 - ]a + b + c g - 2h = 42, azaz 3x = 51, ahonnan x = 17 . Teht az els, a msodik, illetve a harmadik trn rszt vevk szma rendre 17, 18, 22. 7. E1 Egy autjavt zemben 49 szakmunks dolgozik: autszerelk, lakatosok s autvillamossgi szerelk. 5 olyan szakmunks van kzttk, aki mindhrom szakmban jrtas. Azok az autszerelk, akik nem rendelkeznek a lakatos szakmval is, hromszor annyian vannak, mint akik csak a lakatos szakmval rendelkeznek. Ht olyan szakmunks van az sszes kztt, akik az autszerel s a lakatos szakmt is tudjk. Azok a villamossgi szerelk, akik nem rtenek az autszerelshez, 14-gyel kevesebben vannak, mint azok az autszerelk, akik nem rtenek a lakatos munkhoz. Hnyan vannak, akik csak a lakatos szakmval rendelkeznek?
A y a 5 x v V z l L

I.

x p a r b h c III. x+5 q

II. x+1

Ksztsnk egy halmazbrt, s tntessnk fel mindent, amit tudunk. A felttelek szerint a + x = 3l, y = 2, v + z + 14 = a + x . Mivel a + 2 + l + z + v + x + 5 = 49 , gy 3l + l + 3l - 14 + 7 = 49 , teht 7l = 56 , ahonnan l = 8 . Vagyis a csak lakatos szakmval rendelkezk szma: 8.

4. Halmazok klnbsge, komplementer halmaz


1. K2 Legyenek az A, B s C halmazok rendre a 3-mal, 6-tal, illetve 5-tel oszthat szmok halmaza. Mely szmok tartoznak az albbi halmazokba? a) (A \ B) + C; b) A \ B \ C. a) ] A \ Bg + C = {a 15-tel oszthat pratlan szmok}. b) A \ B \ C = {a 3-mal oszthat, de 5-tel nem oszthat pratlan szmok}. 2. E1 Adottak az U alaphalmazon az A, B s C halmazok. Szemlltessk egy halmazbrn az albbi halmazokat! a) ] A , Bg , C ; b) ] A \ Bg , C . a)
A B

b)
A B

C U U

]B + Ag , ] A + Bg = ] A , Bg \ ] A + Bg!
U
9. V F OLYAM

3. E1 Adottak az U alaphalmazon az A s B halmazok. Igazoljuk, hogy

Az egyenlsg mindkt oldalnak a bal oldali halmazbra felel meg.

I. HALMAZOK

MATEMATIKA 13

4. K1 rjuk fel az A , B, A + B s A \ B halmazok elemeit, ha A = {a, b, c, g, h, j}; B = {a, c, f, h, k}! A , B = {a, b, c, f, g, h, j, k}; A + B = {a, c, h}; A \ B = {b, g, j}.

5. K1 Adott hrom halmaz: A = {1, 2, 4, 7, 8, 9, 12}; B = {2, 3, 5, 6, 7, 11, 12}; Adjuk meg az albbi halmazok elemeit! a) (A , B) \ C; b) (A + B) , (B + C );

C = {4, 5, 6, 7, 10, 13}. c) A + (B \ C ).

A knnyebb ttekinthetsg kedvrt elszr ksztsnk halmazbrt, s rjuk be a megfelel szmokat a megfelel helyre. a) ] A , Bg \ C = "1, 2, 3, 8, 9, 11, 12, ; b) ] A + Bg , ]B + C g = "2, 5, 6, 7, 12, ; c) A + ]B \ C g = "2, 12, . 6. K1 Igazoljuk halmazbrk segtsgvel az albbi egyenlsgeket! a) A \ (B , C ) = (A \ B) + (A \ C ); b) A \ (B + C ) = (A \ B) , (A \ C ). a) Az egyenlsg mindkt oldala a kvetkez brhoz vezet: b) Az egyenlsg mindkt oldala a kvetkez brhoz vezet:

A 8

1 9 4 2 12 7 5 13

3 11

10

B A A

7. K2 Igazoljuk, hogy nem minden esetben igaz az albbi egyenlsg! A \ (B \ C ) = (A \ B) \ C. Az egyenlsg mindkt oldalhoz brt ksztnk, ami mutatja az lltst.

B A A

C
A \ ]B \ C g

C
] A \ Bg \ C

8. K2 Legyen az alaphalmaz a vals szmok halmaza. Az A halmaz az x $ 2, a B halmaz az x # 10 , a C halmaz az x # 6 vals szmok halmaza. Hatrozzuk meg az albbi halmazokat! a) A , B ; b) B \ A ; c) A + C .

9 .

V F O LYA M

14 MATEMATIKA

I. HALMAZOK

Szemlltessk az A, B, C halmazokat egy szmegyenesen!

C B A 10
a) A , B = ! x 1 -10+ ;
Barlangsz

10

b) B - A = " x 1 -10 vagy 2 # x , ;

c) A + C = R .

5. vf. 6. vf. 7. vf. 8. vf.

12 14 8 7

16 18 6 5

8 12 17 7

9. E1 Egy ltalnos iskola fels tagozatn hromfle szakkr mkdik: fotszakkr, biolgiaszakkr s barlangsz szakkr. E szakkrk ltszmt a bal oldali bra mutatja vfolyamokra lebontva. Azok szma, akik pontosan kt szakkrre jrnak, ktszer, akik pedig pontosan egy szakkrre jrnak, hromszor annyi, mint azok szma, akik mindhrom szakkr munkjban rszt vesznek. Az iskola fels tagozata 216 dikjnak kb. hny szzalka nem jr semmilyen szakkrre?

Biolgia

Fot

Fot (41) a p c q x

Biol. (45) b r

Barlang. (44)

Fotszakkrre 41, biolgiaszakkrre 45, barlangsz szakkrre pedig 44 dik jr. A felttelek szerint: p + q + r = 2x s a + b + c = 3x . Azoknak a dikoknak a szma, akik legalbb egy szakkrre jrnak: 6x. 41 + 45 + 44 - ^ p + q + r h - 2x = 130 - 4x , vagyis 130 - 4x = 6x, ahonnan x = 13. Teht azoknak a dikoknak a szma, akik jrnak legalbb egy szakkrre, 6x = 78 . Ez az iskola 216 dikjnak kb. 78 $ 100 . 36,1% -a. Azok szma, akik semmilyen szakkrre nem jrnak a 216 fels tagozaton: 100% - 36,1% = 63,9% .

5. A matematikai logika elemei


1. K1 rjuk fel a kvetkez jelzk tagadst, valamint kt klnbz, jelentst kifejez ellenkezjt! a) szp; b) nagy; c) okos; d) vastag; e) kerek; f) homor. eredeti kifejezs a) szp b) nagy c) okos d) vastag e) kerek f) homor a tagadsa nem szp nem nagy nem okos nem vastag nem kerek nem homor kt klnbz jelents ellenkezje csnya kicsi buta vkony szgletes dombor gynyr hatalmas zsenilis tlagos vastagsg ovlis sk

2. K1 rjuk fel a kvetkez kijelentsek tagadst! Dntsk el, hogy melyik igaz; az llts vagy a tagads! a) Minden termszetes szm nagyobb, mint 0. b) Vannak pratlan egsz szmok. c) Minden hromszgnek van legalbb kt hegyesszge. d) Minden tengelyesen szimmetrikus ngyszgnek van kt-kt egyenl szgprja. e) Van olyan skngyszg, amelyben a derkszgek szma 3. f) Brmely kt nem prhuzamos egyenes metszi egymst.
9. V F OLYAM

I. HALMAZOK

MATEMATIKA 15

a) Hamis. A tagadsa: Van 0-nl nem nagyobb termszetes szm. Igaz, pldul a 0. b) Igaz. A tagadsa: Nincsen pratlan egsz szm. Hamis, pldul az 1. c) Igaz. A tagadsa: Van olyan skbeli hromszg, amelynek nincs legalbb kt hegyesszge (vagyis legfeljebb egy hegyesszge van). Hamis. d) Hamis (pldul egy olyan deltoid, amely nem rombusz). A tagadsa: Van olyan szimmetrikus ngyszg, amelynek nincs kt-kt egyenl szgprja. Igaz. e) Hamis. A tagadsa: Minden skngyszgben a derkszgek szma 3-tl klnbz (nem 3). Igaz, hiszen ha 3 derkszge lenne, akkor 4 is lenne. f) Nem igaz, mert lehetnek kitr egyenesprok. A tagadsa: Van olyan egyenespr, amely nem prhuzamos s nem is metsz. Igaz. 3. K2 Ttelezzk fel, hogy igaz az az llts, hogy Ha fttyentesz, elhallgatok.. Mi kvetkezik abbl, hogy a) nem hallgattam el; b) nem fttyentettl; c) elhallgattam; d) fttyentettl? a) Nem fttyentettl, hiszen ha fttyentettl volna, elhallgattam volna. b) Semmi. Lehet, hogy nem hallgattam el, de az is lehet, hogy csak gy magamtl elhallgattam. c) Semmi. Lehet, hogy fttyentettl, s azrt, de az is lehet, hogy csak gy magamtl elhallgattam. d) Elhallgattam, hiszen ha fttyentesz, elhallgatok. 4. K2 Ha megnyitom a csapot, folyik a vz. Az albbiak kzl melyik llts fejezi pontosan ugyanezt? a) Ha nem nyitom meg a csapot, nem folyik a vz. b) Ha folyik a vz, megnyitottam a csapot. c) Ha nem folyik a vz, nem nyitottam meg a csapot. A c). Hiszen ha megnyitottam volna a csapot, akkor folyna a vz. 5. K1 rjuk fel a kvetkez lltsok megfordtst! a) Ha havazik, akkor fagy. b) Ha pntek van, akkor moziba megyek. c) Ha nincs kifogsod ellene, akkor ablakot nyitok. d) Ha rrsz, akkor eljhetsz. a) Ha fagy, akkor havazik. c) Ha ablakot nyitok, akkor nincs kifogsod ellene. b) Ha moziba megyek, akkor pntek van. d) Ha eljhetsz, akkor rrsz.

6. K2 Dntsk el, hogy igazak-e az albbi lltsok! rjuk fel az lltsok megfordtst, s azokrl is dntsk el, hogy igazak-e! a) Ha egy egsz szm pros, akkor 2-esre vgzdik. b) Ha egy egsz szm oszthat 9-cel, akkor a szmjegyeinek az sszege 9. c) Ha egy hromszg derkszg, akkor a kt rvidebb oldalra emelt ngyzet terletsszege egyenl a leghosszabb oldalra emelt ngyzet terletvel. a) Hamis. Megfordtva: Ha egy egsz szm 2-esre vgzdik, akkor pros. Igaz. b) Hamis. Megfordtva: Ha egy egsz szm szmjegyeinek az sszege 9, akkor a szm oszthat 9-cel. Igaz. c) Igaz, ez a Pitagorasz-ttel. Megfordtva: Ha egy hromszgben a kt rvidebb oldalra emelt ngyzet terletsszege egyenl a leghosszabb oldalra emelt ngyzet terletvel, akkor a hromszg derkszg. Igaz, ez a Pitagorasz-ttel megfordtsa. Egy hromszg akkor s csak akkor derkszg, ha a kt rvidebb oldalra emelt ngyzet terletsszege egyenl a leghoszszabb oldalra emelt ngyzet terletvel.

9 .

V F O LYA M

MATEMATIKA 17

II. Algebra s szmelmlet


1. A hatvnyozs s azonossgai
1. K1 Mivel egyenl? a) 15 = 1; d) 36 = 729 ; g) 72 = 49 ; j) 2 $ 3 = 36 ;
2 2

b) 51 = 5 ; e) 42 = 16 ; h) 63 = 216 ; k) 6 $ 1 = 36 ;
2 10

c) 02 = 0 ; f) 2 4 = 16 ; i) 3 4 = 81; l) 11000 = 1.

2. K1 Mivel egyenl? a) d) g) j)
]-1g3 = -1; ]-3g6 = 729 ; ]-5g2 = 25 ; 52 $ ]-5g2 = 625 ;

b) e) h) k)

]-2g3 = -8 ; 43 = 64 ; 52 = 25 ; ]-2g2 $ 0 4 = 0 ;

c) f) i) l)

]-2g4 =16 ; ]-3g4 = 81; ]-5g3 = -125 ; ]-1g100 = 1.

3. K1 rjuk fel hatvny alakban a kvetkez szmokat, ha lehet, tbbflekppen is! a) 1000 pldul: =103 ; c) 81 pldul: = 3 4 = 92 ; e) 1 pldul: =12 =10 = 20 ; b) 1024 pldul: = 210 = 45 = 322 ; d) 100 pldul: =102 ; f) 625 pldul: = 5 4 = 252 .

4. K1 rjuk fel prmszmok hatvnyainak szorzataknt a kvetkez szmokat! a) 10 = 21 $ 51; d) 36 = 2 $ 3 ; g) 243 = 73 ; j) 54 = 2 $ 33 ; m) 1260 = 22 $ 32 $ 5 $ 7 ;
2 2

b) 12 = 22 $ 31; e) 81 = 3 ; h) 1024 = 210 ; k) 160 = 25 $ 5 ; n) 10 000 = 2 4 $ 5 4 ;


4

c) 60 = 22 $ 31 $ 51; f) 54 = 21 $ 33 ; i) 2500 = 22 $ 5 4 ; l) 128 = 27 ; o) 64 000 = 29 $ 53 .

5. K1 Mely szmok prmtnyezs alakjt rtuk fel? a) 23 = 8 ; d) 42 =16 ; b) 33 = 27 ; e) 23 $ 32 = 72; c) 22 $ 33 =108 ; f) 211 = 2048 .

2. A hatvnyozs azonossgainak kiterjesztse


1. K1 Mely szmokat rtuk hatvnyalakban? a) 21; -1 e) b 1 l ; 5 a) 1 ; 2 e) 5; b) (1)1; -1 f) b- 1 l ; 5 b) -1; f) -5 ; c) 51; g) 1 ; -1
3

d) (1)5; -2 h) b 3 l . 4 d) -1; h) 16 . 9

c) 1 ; 5 g) 3;

9 .

V F O LYA M

18 MATEMATIKA

II. ALGEBRA S SZMELMLET

2. K1 rjuk fel a megadott szmokat hatvnyalakban, ha lehet, tbbflekppen is! a) 100; e) 0,01; b) 0,1; f) 1 ; 81 c) 0,25; g) 0,0001;
-2 a) 100 = 1001 = 102 = b 1 l ; 10

d) 1; h) 0,001.

Pldul: c) 0,25 = 0,52 = 2-2 ;


2 f) 1 = 3-4 = b 1 l ; 81 9

b) 0,1 = 10-1;
2 e) 0,01 = 10-2 = b 1 l ; 10

d) 1 = 12 = 1-10 ; g) 0,0001 = 10-4 = 0,012 ;

h) 0,0001 = 0,13 = 10-3 .

3. K1 Szmtsuk ki a szorzsok eredmnyt! 2 4 a) 27 $ 2- 4 ; b) b 1 l $ 2 4 ; c) b 1 l $ 2 4 ; 2 3


2 -2 e) b 3 l $ b 4 l ; 4 3

d) 4- 1 $ b 1 l ; 4
-1 -4 h) b 2 l $ 2 4 . 3

f) 5- 2 $ 2- 5 ;

-2 g) b 2 l $ 52 ; 5

a) 8; e) 256 ; 81

b) 4; f) 1 ; 800

c) 16 ; 81 625 g) ; 4

d) 1; h) 81.

4. K1 Szmtsuk ki a mveletek eredmnyt! a) 4- 2 : 3- 2 ; e) ]-2g-1 : ]-1g-2 ; a) 9 ; 16 e) 1 ; -2 b) 1- 1 : 5- 1; f) 2- 4 : ]-2g4 ; c) 43 $ 33 ; g) 7- 6 $ 76 ; d) ]-2g- 2 : 22 ; h) 7- 5 : b 1 l . 7


5

b) 5; f) 1 ; 256

c) 1728; g) 1;

d) 1 ; 16 h) 1.

5. K1 lltsuk nagysg szerint nvekv sorrendbe a kvetkez szmokat!

a = 3-1; b = ]-1g-3; c = 31; d = ]-1g3; e = 1-3; f = ]-3g1; g = 13; h = ]-3g-1.

a = 1; b = -1; c = 3; d = -1; e = 1; f = -3; g = 1; h = - 1 . 3 3


Eszerint: f 1 b = d 1 h 1 a 1 e = g 1 c .

3. Gyakorlati szmtsok
1. K1 Fejezzk ki a kvetkez szmokat normlalakban! a) 6 000 000; b) 125 000; c) 1560; e) 0,1; f) 1,5; g) 0,000 05; a) 6 $ 106 ; e) 1 $ 10 ; 2. K1 Mennyi a) a 20 15%-a; a) 3;
-1

d) 4 540 000; h) 10 000,000 01. d) 4,54 $ 106 ; h) 1,000 000 001 $ 10 4 .

b) 1,25 $ 105 ; f) 1,5 $ 10 ;


0

c) 1,56 $ 103 ; g) 5 $ 10 ;
6

b) a 15 20%-a; b) 3;

c) a 10 5%-a; c) 0,5;

d) az 5 10%-a? d) 0,5.

9.

V F OLYAM

II. ALGEBRA S SZMELMLET

MATEMATIKA 19

3. K1 Mennyi a) egy szm 20%-nak 120%-a; c) egy szm 125%-nak 80%-a; a) A szm 24%-a; c) a szm 100%-a, azaz maga a szm; 4. K1 Melyik szm 45%-a a) a 10; b) a 45;
: a) 22,2 = 200 ; 9

b) egy szm 80%-nak 120%-a; d) egy szm 180%-nak a 10%-a? b) a szm 96%-a; d) a szm 18%-a.

c) a 135; c) 300;

d) az 1,5? d) 3,3 = 10 . 3
:

b) 100;

5. K1 Tekintsk a Fldet egy olyan gmbnek, amelynek a kzppontjn tmen krk kerlete 40 000 km! a) Megkzeltleg mekkora a Fld tmrje? b) Megkzeltleg mekkora a Fld sugara? c) Megkzeltleg mekkora a Fld trfogata? d) Megkzeltleg mekkora a Fld felszne? e) A Fld felsznnek krlbell hny szzalkt bortja vz, ha az sszes vzfellet nagysga krlbell 3,4 108 km2? (Emlkeztetl: Az r sugar kr kerlete 2rr, terlete r 2r. Az r sugar gmb felszne 4r 2r, trfogata 4 r3 r .) 3 a) d . 1,27 $ 10 4 km. d) A . 5,1 $ 108 km2. b) r . 6,4 $ 103 km. e) Kb. 67%-t. c) V . 1,1 $ 1012 km3.

6. K1 a) Hny szzalka a Fld tmrje a Napnak? b) Hny szzalka a Fld tmege a Napnak? A szksges adatok megtallhatk a ngyjegy fggvnytblzatban. a) A Nap tmrje: 1,4 $ 106 km; a Fld tmrje: 1,27 $ 10 4 km. A kett arnya: 1,4 $ 106 . 1,1 $ 10 . Vagyis a Fld tmrje a Nap tmrjnek 1,1%-a. 1,27 $ 10 4

b) A Fld tmege: 6 $ 1024 kg, a Nap tmege: 2 $ 1030 kg.


24 A kett arnya: 6 $ 1030 = 3 $ 10-6 . 2 $ 10- 4 szzalka. Ez 0,0002%. 2 $ 10

4. Algebrai kifejezsek sszevonsa, szorzsa


1. K1 Vgezzk el az albbi szorzsokat! a) 4a5 c3 $ 5a2 bc 4 ; b) 1 x 4 y2 z3 $ 3 xy3 z5 ; 3 4 a) 20a7 bc7 ; b) 1 x5 y5 z8 ; 4 c) 5 p3 q3 s $ b-14 pq 4 s6l . 7 3 c) - 10 p 4 q7 s7 . 3

2. K1 Vgezzk el az albbi szorzsokat! a) ^2x + 3y h _3x2 - 5xy - 6y2i ; b) b 2 a - 3 a2 bl b 1 a3 + 2 ab2 - 4 a2 bl . 3 4 2 3 3 a) 6x3 - 10x2 y - 12xy2 + 9x2 y - 15xy2 - 18y3 = 6x3 - x2 y - 27xy2 - 18y3 ; b) 1 a 4 + 4 a2 b2 - 8 a3 b - 3 a5 b - 1 a3 b3 + a 4 b2 . 3 9 9 8 2

9 .

V F O LYA M

20 MATEMATIKA

II. ALGEBRA S SZMELMLET

3. K1 Vgezzk el az albbi szorzst! ]a - 1g ^1 + a + a2 + a3 + a 4 + a5h . a + a2 + a3 + a 4 + a5 + a6 -1 - a - a2 - a3 - a 4 - a5 = a6 -1. 4. K1 Vgezzk el az albbi szorzsokat! a) 2 x 4 a2 $ 3 xa3 ; b) 1 p 4 q3 $ _-6pq5i ; 3 4 2 a) 1 x5 a5 ; 2 b) -3p5 q8 ;

c) b-2 xy2l $ b 3 x3 y3l . 3 8 c) - 1 x 4 y5 . 4

5. K2 A kvetkez feladatokban egy tbbtag sszeget kell szoroznunk egy taggal. a) 2x2 ^ x3 - 3x2 + 4x h ; b) 6ab^2ab2 + 3ab - 4a2 bh ; c) 3 x3 y2 b 1 x2 y - 5 xy2 + 10 x2 y3l . 5 2 9 3 a) 2x5 - 6x 4 + 8x3 ; b) 12a2 b3 + 18a2 b2 - 24a3 b2 ; c) 3 x5 y3 - 1 x 4 y 4 + 2x5 y5 . 10 3

6. E1 A kvetkez feladatokban egy tbbtag sszeget kell szoroznunk egy taggal. a) 4x n y k _3x2 y - 2x k y n + 5xy i ; b) 1 q m p2n b 3 q n p2m - 2 q m - 1p n + 1 + 4 q m + n p m - nl . 3 2 3 5 a) 12x n + 2 y k + 1 - 8x n + k y n + k + 20x n + 1y k + 1; b) 1 q m + n p2n + 2m - 2 q2m - 1p3n + 1 + 4 q2m + n p m + n . 2 9 15

7. K1 Az albbi feladatokban tbb tagot kell tbb taggal szoroznunk. a) ]a + 2x g ]a - 2x g; b) ^ y - 2h _ y2 - 2xy + y i ; c) ]1 - x g ^ x3 - 3x2 + x + 1h . a) a2 - 4x2 ; b) y3 - 2xy2 + y2 - 2y2 + 4xy - 2y = y3 - 2xy2 - y2 + 4xy - 2y ;

c) x3 - 3x2 + x + 1 - x 4 + 3x3 - x2 - x = - x 4 + 4x3 - 4x2 + 1. 8. E1 Az albbi feladatokban tbb tagot kell tbb taggal szoroznunk. a) _ x n + y k i ^1 + x + y h ; b) _ p n + 1 - q k i ^ p + q + pqh ; c) b 1 x k + 2 y k + 1l b2x1 - k - 3 x k y k + 1 y k - 1l . 2 3 2 6 a) x n + x n + 1 + x n y + y k + xy k + y k + 1; b) p n + 2 + p n + 1q + p n + 2 q - pq k - q k + 1 - pq k + 1; c) x - 3 x2k y k + 1 x k y k - 1 + 4 x1 - k y1 + k - x k y2k + 1 + 1 y2k . 4 12 3 9

5. Nevezetes szorzatok
1. K1 Vgezzk el az albbi mveleteket! 2 a) ^5x - 3y h2 ; b) ^2a2 b + 4ab2h ; a) 25x2 - 30xy + 9y2 ; b) 4a 4 b2 + 16a3 b3 + 16a2 b 4 ; c) _5x3 y - 2xy i .
2

c) 25x6 y2 - 20x 4 y2 + 4x2 y2 .

2. K1 Alaktsuk szorzatt az albbi kttag sszegeket! a) 49b6 - 121x2 ; b) 36a 4 b2 - 64a2 b 4 ; c) 1 p6 x 4 - 4 a2 b8 . 16 25

a) ^7b3 + 11x h ^7b3 - 11x h ; b) ^6a2 b + 8ab2h ^6a2 b - 8ab2h ; c) b 1 p3 x2 + 2 ab 4 l b 1 p3 x2 - 2 ab 4 l . 4 5 4 5

9.

V F OLYAM

II. ALGEBRA S SZMELMLET

MATEMATIKA 21

3. K1 Elvgeztk egy kttag sszeg ngyzetre emelst, s eredmnyl azt kaptuk: ]..................g2 = 4a2 - 12ab + ..... Sajnos az utols tag elmosdott a papron. Milyen sszeget emeltnk ngyzetre?

]2a - 3bg2 = 4a2 - 12ab + 9b2

vagy

]-2a + 3bg2 = 4a2 - 12ab + 9b2 .

4. K2 Szmtsuk ki az albbi kttag sszegek kbt! 3 3 a) ^a2 + 3ah ; b) ^2x - y h3 ; c) b 1 k2 - 3 n2 k l . 2 2 a) a6 + 9a5 + 27a 4 + 27a3 ; c) 1 k6 - 9 k5 n2 + 27 n 4 k 4 - 27 n6 k3 . b) 8x3 - 12x2 y + 6xy2 - y3 ;

5. K1 Kt tag sszegnek, illetve klnbsgnek a ngyzetrl tanultak alapjn vgezzk el az albbi ngyzetre emelseket! 2 a) ^2x - 3y h2 ; b) ^a2 - 4bh ; c) ^4p + 3qh2 . a) 4x2 - 12xy + 9y2 ; b) a 4 - 8a2 b + 16b2 ; c) 16p2 + 24pq + 9q2 .

6. K1 Vgezzk el az albbi ngyzetre emelseket! 2 2 2 a) ^ x2 - x h ; b) _2y3 + 3y i ; c) ^4a2 + 3abh . a) x 4 - 2x3 + x2 ; b) 4y6 + 12y 4 + 9y2 ; c) 16a 4 + 24a3 b + 9a2 b2 .

7. K1 Kt tag sszegnek, illetve klnbsgnek a szorzatrl tanultak alapjn vgezzk el az albbi szorzsokat! a) ^ x + 3y h ^ x - 3y h ; b) _2x2 + y i _2x2 - y i ; c) ^5a3 b + 2ab2h ^5a3 b - 2ab2h . a) x2 - 9y2 ; b) 4x 4 - y2 ; c) 25a6 b2 - 4a2 b 4 .

8. E1 Vgezzk el a ngyzetre emelseket! 2 2 a) b 1 x3 y2 - 2xy l ; b) b 3 a2 b3 + 5 a3 b2l ; 3 5 3 a) 1 x6 y 4 - 4 x 4 y3 + 4x2 y2 ; 9 3 c) 16 x2n y 4 - 8 x n + 2 y n + 2 + 25 x 4 y2n .

2 c) b 4 x n y2 - 5 x2 y nl . 5 3

b) 9 a 4 b6 + 2a5 b5 + 25 a6 b 4 ; 25 9

25

9. K2 Kt tag ngyzett szmoltuk ki; mi lehet az eredmny hinyz harmadik tagja? a) ]...........g2 = 16x2 + 8xy ..........; b) ]...........g2 = 4a 4 - 12a3 b ..........; c) ]...........g2 = 25p6 - 20p5 q .......... a) ^4x + y h2 = 16x2 + 8xy + y2 ; c) _5p3 - 2p2 qi = 25p6 - 20p5 q + 4p 4 q2 .
2 2 b) ^2a2 - 3abh = 4a 4 - 12a3 b + 9a2 b2 ;

10. K2 Kt tag sszegnek, illetve klnbsgnek harmadik hatvnyrl tanultak alapjn vgezzk el az albbi kbre emelseket! 3 a) ]2a + 1g3 ; b) ^2x + 3y h3 ; c) ^k2 - 2k h . a) 8a3 + 12a2 + 6a +1; c) k6 - 6k5 + 12k 4 - 8k3 . b) 8x3 + 36x2 y + 54xy2 + 27y3 ;

9 .

V F O LYA M

22 MATEMATIKA

II. ALGEBRA S SZMELMLET

6. Tovbbi nevezetes szorzatok (Emelt szint)


1. E1 Vgezzk el a ngyzetre emelseket! a) ^ x + 2y + z h2 ; b) ]2a + 3b - c g2 ;
2 c) b 1 a - 3b2 + 2 abl . 2 3

a) x2 + 4y2 + z2 + 4xy + 2xz + 4yz ; c) 1 a2 + 9b 4 + 4 a2 b2 - 3ab2 + 2 a2 b - 4ab3 . 4 9 3

b) 4a2 + 9b2 + c2 + 12ab - 4ac - 6bc ;

2. E1 Alaktsuk szorzatt az albbi kttag sszegeket! a) x7 + 1; b) p6 - q3 ; c) a5 + 32; a) ] x + 1g ^ x6 - x5 + x 4 - x3 + x2 - x + 1h ; c) a5 + 25 = ]a + 2g ^a 4 - 2a3 + 4a2 - 8a + 16h ;


3

d) 27k3 - y9 .

b) _ p2i - q3 = _ p2 - qi _ p 4 + p2 q + q2i ; d) _3k - y3i _9k2 + 3ky3 + y6i .

3. E1 Igazoljuk, hogy 12324 - 1 oszthat 15 130-cal!


^123 4h6 - 16 = ^123 4 - 1 h $ K, ahol K egsz szm. Teht ^1232 - 1 h ^1232 + 1h $ K .

De 1232 +1 =15 130 , teht a kifejezs oszthat 15 130-cal. 4. E1 Hromtag sszeg ngyzetrl tanultak alapjn vgezzk el az albbi ngyzetre emelseket! a) ]a - b + 2c g2 ; b) ^2x - 3y - z h2 ; c) ^ p + 2q + 3z h2 . a) a2 + b2 + 4c2 - 2ab + 4ac - 4bc ; c) p2 + 4q2 + 9z2 + 4pq + 6pz + 12qz . 5. E2 Hromtag sszeg ngyzetrl tanultak alapjn vgezzk el az albbi ngyzetre emelseket! 2 2 2 a) ^2a3 - 3ab + b2h ; b) b 3 p2 + 2 p2 q2 - 1 q2l ; c) ^2a k - 3b k + 1 + a k b k - 1h . 4 3 2 a) 4a6 + 9a2 b2 + b 4 - 12a 4 b + 4a3 b2 - 6ab3 ; b) 9 p 4 + 4 p 4 q 4 + 1 q 4 + p 4 q2 - 3 p2 q2 - 2 p2 q 4 ; b) 4x2 + 9y2 + z2 - 12xy - 4xz + 6yz ;

16

c) 4a + 9b

2k

2k + 2

+a b

2k

2k - 2

- 12a b

k+1

+ 4a b k - 1 - 6a k b2k .

2k

6. K2 Szmtsuk ki az albbi kifejezsek megfelel helyettestsi rtkt! 2 a) ^2a2 - 3bh - ^2a2 + 3bh ^2a2 - 3bh + 12a2 b, a = -1,9, b = 1 ; 3 2 3 2 2 4 b) ^1 + x + x h - 2x^1 + x h - x , x = - ; 5
2 c) ^6k2 - 5nh ^6k2 + 5nh - 6k2 ]12 + 10ng + ^6k2 + 5nh ,

k = 1,

n = -12,5 .

a) 4a 4 -12a2 b + 9b2 - 4a 4 + 9b2 +12a2 b = 18b2 = 18 $ 1 = 2; 9 b) 1 + x2 + x 4 + 2x + 2x2 + 2x3 - 2x - 2x3 - x 4 = 3x2 + 1 = 52 ; 25 c) 36k 4 - 25n2 - 72k2 - 60k2 n + 36k 4 + 60k2 n + 25n2 = -72k2 = -72.

9.

V F OLYAM

II. ALGEBRA S SZMELMLET

MATEMATIKA 23

7. sszegek szorzatt alaktsa


1. K1 Alaktsuk szorzatt a kvetkez kifejezseket! a) 2ax2 + 6ax - 4ax3 ; b) 3k2 - 12k + 24k6 ; a) 2ax^ x + 3 - 2x2h ; b) 3k^k - 4 + 8k5h ; c) 5p3 q 4 - 25p 4 q3 + 10p5 q5 . c) 5p3 q3 _q - 5p + 2p2 q2i .

2. K2 Alaktsuk szorzatt a kvetkez kifejezseket! a) 2a + 2b + a2 - b2 ; b) p3 q - q3 p + 4p2 + 8pq + 4q2 ; a) 2]a + bg + ]a + bg ]a - bg = ]a + bg ]2 + a - bg;

c) x3 - y3 + x2 - y2 .

b) pq _ p2 - q2i + 4 ^ p + qh2 = pq ^ p - qh ^ p + qh + 4 ^ p + qh2 = ^ p + qh 7 pq ^ p - qh + 4 ^ p + qhA ; c) ^ x - y h _ x2 + xy + y2i + ^ x - y h ^ x + y h = ^ x - y h _ x2 + xy + y2 + x + y i . 3. K2 Alaktsuk szorzatt a kvetkez kifejezseket! a) a 4 + a - 14 ; b) a2 + b2 + c2 + 2ab + 2ac + 2bc - 4x2 ;

c) k3 + k2 - 2.

a) a 4 - 16 + a + 2 = ^a2 - 4h ^a2 + 4h + a + 2 = ]a - 2g ]a + 2g ^a2 + 4h + a + 2 = = ]a + 2g 7]a - 2g ^a2 + 4h + 1A = ]a + 2g ^a3 - 2a2 + 4a - 7h ; b) ]a + b + c g2 - 4x2 = ]a + b + c + 2x g ]a + b + c - 2x g; c) k3 - 1 + k2 - 1 = ]k - 1g ^k2 + k + 1h + ]k - 1g ]k + 1g = ]k - 1g ^k2 + 2k + 2h .

4. K1 Alaktsuk szorzatt az albbi sszegeket! a) 6a2 x - 12ax2 + 3a2 x2 ; b) 8p3 q + 2p2 q2 + 6pq3 - 4pq ; c) 2ab2 c + 4a2 bc - 8abc2 + 20abc . a) 3ax]2a - 4x + ax g; b) 2pq _4p2 + pq + 3q2 - 2i ; c) 2abc]b + 2a - 4c +10g.

5. K2 A tagok megfelel csoportostsval alaktsuk szorzatt a kvetkez sszegeket! a) 5ax + 2x + 5ay + 2y ; b) k3 + 3k2 + 3k + 9 ; c) a2 c + 2bc - 3a2 d - 6bd ; d) a2 - b2 + a + b ; a) ^ x + y h ]5a + 2g; d) ]a + bg ]a - b +1g; e) x3 - x2 y + y3 - xy2 . b) ]k + 3g ^k2 + 3h ; c) ]c - 3d g ^a2 + 2bh ;

e) ^ x - y h2 ^ x + y h .

6. E1 A nevezetes szorzatok alkalmazsval alaktsuk szorzatt a kvetkez kifejezseket! a) pq^k2 - n2h + k2 n + kn2 ; b) 4 ^ xz + yr h2 - _ x2 - y2 - r2 + z2i ;
2

c) a5 + a 4 - 2a3 - 2a2 + a + 1. a) pq]k - ng ]k + ng + kn]k + ng = ]k + ng ^ pqk - pqn + knh ; b) ^2xz + 2yr h2 - _ x2 - y2 - r2 + z2i = _2xz + yr + x2 - y2 - r2 + z2i _2xz + yr - x2 + y2 + r2 - z2i = 2 2 2 2 = 7] x + z g - ^ y - r h A $ 7^ y + r h - ] x - z g A =
2 2 c) a 4 ]a + 1g - 2a2 ]a + 1g + a + 1 = ]a + 1g ^a 4 - 2a2 + 1h = ]a + 1g ^a2 - 1h .

= ^ x + z + y - r h ^ x + z - y + r h ^ y + r + x - z h ^ y + r - x + z h;

7. E2 Egy hromszg oldalai a, b s c. Igazoljuk, hogy ekkor az albbi kifejezs rtke negatv szm! ^b2 + c2 - a2h2 - 4b2 c2 . A kttag sszeg kt ngyzet klnbsge, ezrt gy alakthat szorzatt: ^b2 + c2 - a2 + 2bc h ^b2 + c2 - a2 - 2bc h = ]b + c + ag ]b + c - ag ]b - c + ag ]b - c - ag. A kapott ngytnyezs szorzat els hrom tnyezje a hromszg-egyenltlensg miatt pozitv, utols tnyezje negatv, teht a szorzat negatv.
9 . V F O LYA M

24 MATEMATIKA

II. ALGEBRA S SZMELMLET

8. Algebrai trtek egyszerstse, sszevonsa


Az albbi feladatokban felttelezzk, hogy a vltozk semmilyen rtkre sem lesznek 0-k a feladatokban elfordul trtek nevezi. 1. K1 Egyszerstsk az albbi trteket! 2 ^ p + qh x6 y a) 3x3 b ; b) 2 ; _ p + 2pq + q2i y2 b x a) 3x ; b2 b) x6 ; ^ p + qh y

c)

5a2 - 5x2 . 10a + 20ax + 10x2


2

] g] xg a-x . c) 5 a - x a + = 2]a + x g ] 10 a + x g2

2. K1 Egyszerstsk az albbi trteket! 3x3 - 3y3 2ax + 6bx + aq + 3bq a) ; b) 2 . 2 2 a + 6ab + 9b 9x - 18xy + 9y2 a) b) 2x]a + 3bg + q]a + 3bg 2x + q ; = a + 3b ]a + 3bg2

3 ^ x - y h _ x2 + xy + y2i x2 + xy + y2 . = 3^ x - y h 9 ^ x - y h2

3. K2 Vgezzk el a kijellt mveleteket! x x ; a) 1 - 1 ; b) + a-1 a+1 2x + 1 2x - 1

c)

a+b a-b 1 . + 2a2 - 2b2 3a + 3b a - b

g ] g ] a) a + 1 2- a - 1 = 2 2 ; a -1 a -1 ]2x - 1g + x]2x + 1g x 4x2 ; b) = 2 2 4x - 1 4x - 1


c)

a+b a-b 1 3]a + bg + 2]a - bg2 - 6]a + bg + = = ] g ] g ] g a b 2 a-b a+b 3 a+b 6^a2 - b2h 2]a - bg2 - 3]a + bg . 6^a2 - b2h

4. E1 Vgezzk el a kijellt mveleteket! 2y - 3 y - 1 1 + 2y + 2 + 2x - 3 - x - 3 . a) ; b) x 2 + 9y2 - 1 3y + 1 6y - 2 4x - 1 4x + 2 2x - 1 2 ^2y - 3h + 2 ^3y -1h ^ y -1h - ^3y + 1h ^1 + 2y h -9y - 5 ; = 2 ^3y -1h ^3y + 1h 2 ^3y - 1h ^3y +1h g] ] g ] 3g - 2]2x +1g ] x - 3g 4x +13 . b) 2 x + 2 + 2x -1 2x = 2^4x2 -1h 2^4x2 -1h a) 5. E1 Egyszerstsk az albbi trteket! a) a) 4p3 q + 4pq3 ; p4 - q4 4pq ; p - q2
2 2 2ab + b2 ; b) a + 4 2a - 2b 4

c)

^ x + y h2 - a 2 . x+y+a

b)

a+b ; 2]a - bg ^a2 + b2h

c) x + y - a .

9.

V F OLYAM

II. ALGEBRA S SZMELMLET

MATEMATIKA 25

6. E2 Egyszerstsk az albbi trteket! 2 2xy - x2 - y2 + a2 a) 2 ; b) x2 + 2x + 1 ; 2 2 x + 8x + 7 a + x - y + 2ax a)

3 e2 f + ef 2 . c) e - 3 e + f3

a 2 - ^ x - y h2 ^ a + x - y h ^ a - x + y h a - x + y ; = = ]a + x g2 - y2 ^a + x + y h ^a + x - y h a + x + y b) A nevezt gy alakthatjuk: x2 + 8x + 7 = x2 + x + 7x + 7 = x] x + 1g + 7] x + 1g = ] x + 1g ] x + 7g, ] x +1g2 x 1 = + ; ] x +1g ] x + 7g x + 7 e^e2 - ef + f 2h e . c) = ]e + f g ^e2 - ef + f 2h e + f 7. K1 Vgezzk el a kvetkez sszevonsokat! y a) x + ; b) b + 2a - b ; 3 ab ac a) cx + by ; abc

c) c)

2y 5y . + 3a - 3 a - 1 17y . 3]a -1g 3p 1 2 . + p - 1 p + 1 p2 - 2p + 1

] g b) 2 a + b ; 3

8. K2 Vgezzk el a kvetkez sszevonsokat! 5 2 a2 3 x-3 ; a) b) + - 2- + 2; 2 2x + 6 x2 + 6x + 9 2a + 6a a - 9

c)

] - 3g ] g 5x + 3 ; a) 3 x + 3 + 2 x = 2] x + 3g2 2] x + 3g2
b) c)

^ p -1h ^ p +1h + 2 ^ p -1h2 - 3p ^ p +1h -7p +1 . = 2 ^ p -1h ^ p +1h ^ p -1h2 ^ p +1h

5 2 - a2 5]a - 3g - 2a^2 - a2h + 4a]a + 3g ]a - 3g 6a3 - 35a - 15 ; +2 = = 2a]a + 3g ]a - 3g ]a + 3g 2a]a + 3g ]a - 3g 2a^a2 - 9h

9. E1 Vgezzk el a kvetkez sszevonsokat! 2 3x + 5 1 2x 6 ; a) 4x - 2 x3 - 1 x +x+1 x-1 3xy y-x b) 1 - 3 ; x - y x - y3 x2 + xy + y2 1 1 1 c) . ]a - bg ]b - c g ]b - c g ]a - c g ]c - ag ]b - ag


2 ^ 2 h ] g] g 12x ; a) 4x - 3x + 5 - x -13 1 - 2x - 6 x + x +1 = -3 x -1 x -1

x2 + xy + y2 - 3xy - ^ y - x h ^ x - y h ^ x - y h2 + ^ x - y h2 2x - 2y ; = = 2 2 ^ x - y h _ x + xy + y i ^ x - y h _ x2 + xy + y2i x2 + xy + y2 c) c - a + a - b + b - c = 0 . ]a - bg ]b - c g ]c - ag b)

9 .

V F O LYA M

26 MATEMATIKA

II. ALGEBRA S SZMELMLET

9. Algebrai trtek szorzsa, osztsa, sszetett mveletek algebrai trtekkel


1. K1 Vgezzk el a kvetkez mveleteket! 6p3 k2 2pk3 6p2 + 12pq + 6q2 2p3 q3 a) 2 $ ; b) ; $ 2 5 ^ p + qh x y 3xy 4a3 b5 a) 4p 4 k5 ; x3 y3 b) 3 ^ p + qh p3 q3 ; 5a3 b5 c) c) 4xy3 3x2 y 4 5x 4 y5 . $ $ 7rs5 r2 s 2r 4 s3 30x7 y12 . 7r7 s9

2. K1 Vgezzk el a kvetkez mveleteket! 2 5xy3 2pa2 4b3 a3 2a2 b a) 2a b : ; b) . : e o$ 3xy 14a3 b2 3b2 5p2 7p5
5 3 a) 28a2 b4 ; 15x y

b)

10p3 a2 2a2 b 5a . $ = 12b5 a3 7p5 21b 4 p2

3. K2 Vgezzk el a kijellt mveleteket! a) c 1 + 1 m $ c 1 - 1 m : ^ x + y h ; x y x y 1 2 2x + 2y ; b) d 1 + + n: x2 y2 xy x3 y3 c) c 2k + 1 - 2k - 1m : 8k . 2k - 1 2k + 1 6k - 3 a) d b)

xy ^ x + y h ^ x + y h2 y2 + x2 + 2xy x3 y3 x3 y3 ; $ $ = = 2 2 2 2 2 2^ x + y h 2^ x + y h x y x y ] g ] g2 ]2k - 1g2 3]2k - 1g 8k c) 2k + 1 $ $ 3 2k - 1 = 3 . = ]2k - 1g ]2k + 1g 8k 8k 2k + 1 4k2 - 1 4. E1 Hatrozzuk meg az albbi kifejezs rtkt, ha x = 4! c1 + 1 - 1 m $ ^ x2 - 1h . x-1 x+1 x2 - 1 + x + 1 - ] x - 1g $ ^ x2 1h x2 1 17 . - = + = x2 - 1 5. E1 Igazoljuk, hogy ha x-y x 1 1 2 = , akkor + = ! y-z z x z y

^ y + xh^ y - xh y+x y-x y-x $ $ 1 = 2 2 ; n$ 1 = xy xy x+y x+y x2 y2 x y

Szorozzuk keresztbe a megadott egyenlsget: azaz xz - yz = xy - xz , 2xz = xy + zy . Most osszuk el a kapott egyenlet mindkt oldalt xyz-vel, ami biztosan nem 0; azt kapjuk: 2 1 1 = + . y z x 6. E1 A kijellt mveletek elvgzsvel hozzuk egyszerbb alakra a kvetkez kifejezseket! 2 a) c x + 1m : d1 - 3x 2 n ; x+1 1-x p 2q 2 p 2q 2 n -d n ; + 2q p 2q p 2 a b a c) c 2 b + m : c + - 2m ; a b a + ab a + b b2 + ab 2 a 2 a 2 a 2 1 + + d) c 2 + - ; m$ a a + 2a 2a + 4 a + 2 b) d
9. V F OLYAM

II. ALGEBRA S SZMELMLET

MATEMATIKA 27

e) e

y-x y x2 1 + o:e 2 o. y + xy x2 + xy y3 - x2 y x + y

2 2 ] g] g a) x + x + 1 : 1 - x -23x = 2x + 1 $ 1 - x 1 + x = 1 - x ; x+1 x + 1 ]1 - 2x g ]1 + 2x g 1 - 2x 1-x

b) d

p 2q p 2q p 2q p 2q 2p 4q $ n$d n= + + + + = 4; 2q p 2q p 2q p 2q p 2q p 2 2 2 2 ] g2 c) b - 2ab + a : b + a - 2ab = a - b $ ab 2 = 1 ; ] g ] ab a+b ab a + b ab a + bg ]a - bg

2 ] g2 ] 2g 1 2a + 2 1 a + 1 1 a 1; d) 4a + 4 + a $ 2a + 2 - 1 = a + 2 2a + - = - = - = = a a 2a a a a a 2a]a + 2g a + 2 2a]a + 2g2

e)

xy ^ x + y h x2 + y2 - xy xy - x2 - y2 x2 + y2 - xy x x . : $ = == y-x x-y y ^ y - x h ^ y + x h xy ^ x + y h y ^ y - x h ^ y + x h xy - x2 - y2

7. E1 Legyen k egy pozitv vals szm. Mivel egyenl a3 + b3 , ha a + b = k s a2 + b2 = 2k ?


2 ]a + bg2 = k2 = a2 + b2 + 2ab = 2k + 2ab $ ab = k - 2k . 2 2 3 ]a + bg3 = k3 = a3 + b3 + 3ab]a + bg. Teht a3 + b3 = k3 - 3k $ k - 2k = -k + 6k . 2 2

8. E1 Igazoljuk, hogy brmely pozitv szmnak s reciproknak sszege legalbb 2! Azt kell igazolnunk, hogy a + 1 $ 2. Szorozzuk meg mindkt oldalt a 2 0 -val: a2 - 2a + 1 $ 0 , a azaz ]a - 1g2 $ 0 , ami nyilvnval. Egyenlsg akkor s csak akkor, ha a = 1. 9. E2 Igazoljuk a kvetkez egyenltlensget! ]a + b + c g $ b 1 + 1 + 1 l $ 9 . a b c Elvgezve a szorzst, azt kapjuk: azaz 1 + a + a + b + 1 + b + c + c + 1 $ 9, c a b b c a Ez pedig az elz feladat alapjn mr nyilvnval. 10. E2 Bizonytsuk be, hogy ha x2 + 2yz + 2xz + y2 1 1 1 1, akkor + = = 2! x y z x2 y2 z z= xy . Ezt a bizonytand egyenlsgbe helyettestve addik az llts. x+y

a a b b c c $ 6. + + + + + b c a c a b

11. E2 Bizonytsuk be, hogy ha k pozitv egsz szm, akkor az albbi kifejezs rtke is egsz szm! 9 k 1 3 1 c 2 + m : c + 2 - m. 3 3 k k k

]3 k g ^k2 - 3k + 9h 27 + k3 $ 3k 2 = + 2 = 3 + k , ami pozitv egsz, ha k pozitv egsz. 2 2 3k k - 3k + 9 k - 3k + 9

9 .

V F O LYA M

28 MATEMATIKA

II. ALGEBRA S SZMELMLET

10. Oszthatsg
1. K1 Vizsgljuk meg az albbi szmokat 3-mal, 4-gyel s 6-tal val oszthatsg szempontjbl! a) 3672; b) 88 716; c) 52 491; d) 41 782. a) Ez esetben a szmjegyek sszege 18, teht a krdses szm oszthat 3-mal. Mivel az utols kt jegye 72, ami oszthat 4-gyel, gy a megadott szm oszthat 4-gyel is s 6-tal is. b) A megadott szm oszthat 4-gyel is s 6-tal is (gy termszetesen 3-mal is). c) A szmjegyek sszege oszthat 3-mal, teht a szm is oszthat 3-mal. Mivel a krdses szm pratlan, gy nem lehet 4-gyel (s 6-tal sem) oszthat. d) A megadott szm pros, de nem oszthat 4-gyel. Mivel a szmjegyek sszege nem oszthat 3-mal, gy ez a szm sem 3-mal, sem 4-gyel, sem 6-tal nem oszthat. 2. K2 Igazoljuk, hogy ha 19 | 2a + 3b s 19 | a + 6b, akkor 19 a-nak is s b-nek is osztja! Ha 19 a + 6b , akkor 19 2a + 12b , de a felttelek szerint 19 2a + 3b . E kt utbbi miatt 19 2a + 12b - 2a - 3b = 9b . Ebbl kvetkezik, hogy 19 b , innen pedig 19 a is teljesl. 3. K1 Mekkora legyen az X szmjegy, hogy a 217 s 54X szmok sszege oszthat legyen 9-cel? 217 = 9K +1, ezrt 54X-nek 9-cel osztva 8 maradkot kell adnia. Mivel 5 + 4 = 9 , ezrt X = 8 . 4. K2 Mennyi maradkot kapunk, ha az albbi kifejezst elosztjuk 5-tel? (4k + 1)2 + (3k 1)2 + 3(k + 1). 16k2 + 8k + 1 + 9k2 - 6k + 1 + 3k + 3 = 25k2 + 5k + 5 = 5^5k2 + k + 1h . Teht a kifejezs 5-tel oszthat, nincs maradk. 5. K1 Az albbi szmok kzl melyek oszthatk 4-gyel, illetve 9-cel? a) 11 648; b) 33 336; c) 27 549; d) 5080; e) 32 974. 4-gyel oszthat az a), b), d); 9-cel oszthat a b) s a c). 6. K1 Az albbi tjegy szm oszthat 45-tel. Milyen szmjegy lehet X s Y? 12 X6Y Y = 0 s X = 0 vagy Y = 0 s X = 9 vagy Y = 5 s X = 4. 7. K1 Az a termszetes szm 7-tel osztva 3 maradkot ad, a b termszetes szm 7-tel osztva 4 maradkot ad. Mit kapunk maradkul, ha az albbi szmokat elosztjuk 7-tel? a) a + 2b; b) 5a + 3(b + 2); c) b(a2 + 1). a) a + 2b = 7r + 4 ; b) 5a + 3]b + 2g = 7s + 5 ; c) b^a2 +1h = 7k + 5 .

8. K1 Mennyi maradkot kapunk, ha az albbi szmokat elosztjuk 13-mal? a) 26k + 28; b) (2n + 3)(6n 1) + n(n 2) n + 6. a) 26k + 28 = 13n + 2; b) 13n2 - 13n + 3 = 13s + 3.

9.

V F OLYAM

II. ALGEBRA S SZMELMLET

MATEMATIKA 29

11. Prmszmok, a szmelmlet alapttele


1. K1 Hny darab prmszm van 50 s 100 kztt? Az 50 s 100 kz es prmek: 53, 59, 61, 67, 71, 73, 79, 83, 89, 97; 10 darab. 2. K1 Vgezzk el az albbi szmok prmtnyezs felbontst! a) 8565; b) 4002; c) 1539. a) 8565 = 3 $ 5 $ 571; b) 4002 = 2 $ 3 $ 23 $ 29 ; c) 1539 = 3 4 $ 19 .

3. K2 Keressk meg az sszes olyan p prmszmot, melyre 4 + p s 8 + p is prmszm! Ha p 3-mal osztva 1 maradkot ad, p = 3k +1, akkor 8 + p = 8 + 3k +1 = 9 + 3k oszthat 3-mal, teht nem lehet prm. Ha p 3-mal osztva 2 maradkot ad, p = 3k + 2, akkor 4 + p = 4 + 3k + 2 = 6 + 3k szintn oszthat 3-mal, teht nem lehet prm. Ha p oszthat 3mal, akkor csak p = 3 lehet; ekkor 4 + p = 7, 8 + p = 11, mindkett prm. Teht egyedl a p = 3 lehetsges. 4. E1 Melyek azok a p prmek, melyekre 2p + 1 egy termszetes szm kbvel egyenl? 2p +1 = n3 , ahol n biztosan pratlan szm, n = 2k +1. Ekkor

ahonnan 2p +1 = ]2k +1g3 = 8k3 +12k2 + 6k +1, p = k^4k2 + 6k + 3h . Ez csak akkor lehetsges, ha k =1, s ezzel 4k2 + 6k + 3 =13 valban prm. Teht egyetlen prm felel meg a feltteleknek: p =13. Ekkor 2p +1 = 27 = 33 . 5. K2 Fejtsk meg ezt a keresztrejtvnyt, ahol a ngyzetekbe szmjegyeket kell rni! Vzsz.: 1. 20-nl kisebb prmszm, mely jegyeinek sszege kbszm. 2. Egy prmszm ktszerese. 3. 9-cel oszthat szm. Fgg.: 1. Ngyzetszm. 2. Azonos a vzsz. 1-gyel. 6. E1 Legyenek p > q > r prmszmok. Mi a megoldsa az albbi egyenletnek? p + q + r = 22. Csak r = 2 lehetsges. Ekkor p + q = 20 , ahonnan p = 17, q = 3, vagy p = 13, q = 7 . 7. E1 Milyen pozitv egsz n-re teljesl, hogy n2 + 10n prmszm? n]n + 10g = prm csak gy lehetsges, ha n = 1; ekkor 10 + n = 11 az egyetlen ilyen prmszm.
2

1 2 3

1 7 1 4 3 7 4 7 1 7 1 0 3 7 0 2
2 1

9 .

V F O LYA M

30 MATEMATIKA

II. ALGEBRA S SZMELMLET

12. Legnagyobb kzs oszt, legkisebb kzs tbbszrs


1. K2 Legyenek A = 23 5 112, B = 23 32 52 7, C = 33 7 113. Szmtsuk ki az albbi kifejezsek rtkt! a) [(A; B); C ]; b) ([B; C ]; A). a) ^ A; Bh = 23 $ 5 , teht 6^ A; Bh; C @ = 23 $ 33 $ 5 $ 7 $ 113 ; b) 6B; C @ = 23 $ 33 $ 52 $ 7 $ 113 , teht ^6B; C @; Ah = 23 $ 5 $ 112 . 2. K2 Milyen pozitv egsz n s k szmokra teljesl, hogy (n; k) = 26 s [n; k] = 4784? 26 = 2 $ 13, 4787 = 2 4 $ 13 $ 23. Az n s a k is tartalmazza a 13-at. Egyikkben 2, a msikban 2 4 szerepel. A 23 prmtnyez brmelyikben lehet. Teht n = 2 4 $ 13 $ 23, k = 2 $ 13 , vagy n = 2 4 $ 13, k = 2 $ 13 $ 23. 3. K2 Melyik az a legkisebb 1-nl nagyobb pozitv egsz szm, amelyik 4-gyel, 5-tel, 6-tal, 7-tel, 8-cal s 9-cel osztva egyarnt 3 maradkot ad? Ha a keresett szm n, akkor n - 3 a megadott szmok mindegyikvel oszthat, vagyis e szmok legkisebb kzs tbbszrst keressk. 6 4, 5, 6, 7, 8, 9@ = 2520 . Teht a keresett szm n = 2523. 4. K1 Hatrozzuk meg az albbi szmok legnagyobb kzs osztjt s legkisebb kzs tbbszrst (p, q, r, s, t klnbz prmek)! a) 5660, 315; b) 3444, 720; c) p3qs2t, pqs4r2. a) ^5660; 315h = 5, 65660; 315@ = 2 $ 32 $ 5 $ 7 $ 283;

b) ^3444; 720h = 22 $ 3, 63444; 720@ = 2 4 $ 32 $ 5 $ 7 $ 41; c) _ p3 qs2 t; pqs 4 r2i = pqs2, 7 p3 qs2 t; pqs 4 r2A = p3 qs 4 tr2 .

5. K2 Legyenek A = 110, B = 120, C = 450. Szmtsuk ki az albbi kifejezsek rtkt! a) [(A; C); B]; b) [(A; B); (C; B)]; c) ([B; C]; A). a) 6^ A; C h; B @ = 120 ; b) 6^ A; Bh; ^C; Bh@ = 30 ; c) ^6B; C @; Ah = 10 .

6. E1 Ha az albbi trtek egyszersthetk, akkor mivel egyszersthetk? a) 2n - 3 ; b) 3n - 1 ; c) 10k + 1. n+2 2n + 6 4k - 2 a) Ha a trt d-vel egyszersthet, akkor 2n - 3 = rd s n + 2 = sd . A msodik egyenlet ktszerest az elsbl kivonva: 7 = d]2s - r g, ha teht a trt egyszersthet, akkor csak 7-tel egyszersthet. b) Ha a trt egyszersthet, akkor 20-szal (vagy annak valamelyik osztjval) egyszersthet. c) Ha a trt egyszersthet, akkor csak 3-mal egyszersthet. 7. K2 Egy kerkpros egy AB tvolsg els harmadt egy ra alatt tette meg, az t htralev rszt pedig 2,5 ra alatt. Sebessge mindkt szakaszon km/h-ban mrve egsz szm, melyek legkisebb kzs tbbszrse 120. Mekkora az AB tvolsg? Legyen v1, illetve v2 az t els, illetve msodik szakaszn a sebessg.
1S 3 2S 3

1 ra A
9. V F OLYAM

2,5 ra B

II. ALGEBRA S SZMELMLET

MATEMATIKA 31

S 2S S 3 3 Ekkor = 1 s = 2,5 . Az els egyenletbl = v1; ezzel a msodik egyenlet 3 v1 v2 v1 2,5 5 . 2v1 2,5 , azaz = = = v2 2 4 v2 Ezek szerint valamely n termszetes szmra v1 = 5n, v2 = 4n s 65n; 4n@ = 120 . Mivel 120 = 23 $ 3 $ 5 , ezrt az n prmtnyezs felbontsban az 5 nem szerepelhet, hiszen ha benne lenne, akkor a legkisebb kzs tbbszrsben mr 52 -nak kellene szerepelnie. Ugyanakkor az n prmfelbontsban a 3-nak benne kell lennie az els hatvnyon, valamint a 2-nek is szerepelnie kell (2-nek magasabb kitevj hatvnya mr nem szerepelhet, mert akkor v2 miatt mr 2 4 szerepelne a legkisebb kzs tbbszrsben). Teht csak n = 2 $ 3 = 6 lehetsges, gy v1 = 30, v2 = 24 . Az els rban megtett t 30 km, a msodik szakaszon megtett t 2,5 $ 24 = 60 km, vagyis az AB tvolsg 90 km. 8. E1 Az n s k pozitv egszek legnagyobb kzs osztja s legkisebb kzs tbbszrse: (n; k) = p2, [n; k] = p3q2, ahol p s q klnbz prmszmok. Hatrozzuk meg n s k prmtnyezs alakjt! n = p2, k = p3 q2 vagy n = p2 q2, k = p3 (termszetesen n s k szerepe felcserlhet). 9. E1 Fejtsk meg a keresztrejtvnyt! Vzsz.: 1. Eggyel cskken szmjegyek. 3. Egy ngyzetszm fordtottja. 4. A 40-nl kisebb prmszmok szma. 5. Oszthat 24-gyel. Fgg.: 1. Egy ikerprmpr nagyobbik tagja. 2. 9-cel oszthat palindrom-szm (azaz olyan szm, mely visszafel olvasva is ugyanaz). 3. A 40 s 50 kz es prmek sszegnek tszrse. 4. Egy kbszm negyede.

1 3 4 5

13. Osztk szma, ngyzetszmok (Emelt szint)


1. E1 Hatrozzuk meg az albbi szmok osztinak a szmt! a) 240; b) 500; c) 625; d) 1110. A megadott szmok prmtnyezs felbontsa alapjn: a) d]240g = 5 $ 2 $ 2 = 20 ; b) d]500g =12 ; c) d]625g = 5 ; d) d]1110g = 16 .

4 3 2 3 6 3 5 4 5 1 2 5 5 7 6

2. E1 Az A s B szmok prmtnyezs alakja: A = 23 5 73 11; B = 22 3 52 72. Mivel egyenl d(A B)? A $ B = 25 $ 3 $ 53 $ 75 $ 11, teht d] A $ Bg = 576 . 3. E2 Melyik az a legkisebb termszetes szm, mely oszthat 12-vel, s amelyre d(N) = 12? Ha d]N g =12, s N oszthat 12-vel, akkor N-nek legalbb kt klnbz prm osztja van. gy N prmtnyezs alakja az albbiak egyike: N = p $ q5, N = p2 $ q3, N = p $ q $ r2 . A legkisebb prmszmokat gyelembe vve a keresett szm: N = 60 . 4. E1 Valamely N termszetes szmra d(d(N)) = 3. Hny klnbz prm osztja lehet N-nek? Ha d ]d]N gg = 3, akkor d]N g = p2 . Ezek szerint vagy N = q p -1, vagy N = q p -1 $ r p -1. Teht az N prmtnyezs alakjban legfeljebb ktfle klnbz prmszm szerepelhet csak.
2

9 .

V F O LYA M

32 MATEMATIKA

II. ALGEBRA S SZMELMLET

5. E2 Legyen a = 23 3 5; b = 2 53 7. Hatrozzuk meg az albbi mennyisgeket! a) d(ab); b) d([a; b]); c) d((a; b)). a) d]abg = d^2 4 $ 3 $ 5 4 $ 7h =100 ; b) d ^6 a; b@h = 64 ; c) d ^^a; bhh = 4 .

6. E1 Az N termszetes szm prmtnyezs alakja N = 2x 3y. Az N hromszorosnak 4-gyel, az N ktszeresnek pedig 5-tel tbb osztja van, mint N-nek. Melyik ez az N szm? Egyrszt ] x +1g ^ y + 2h = ] x +1g ^ y +1h + 4 , ahonnan ] x + 2g ^ y +1h = ] x +1g ^ y +1h + 5 , ahonnan y = 4. msrszt pedig x = 3, 3 Teht N = 2 $ 3 4 = 648 .

7. E2 Melyik az a legkisebb termszetes szm, melynek 42 osztja van, s oszthat 42-vel? Ha az N termszetes szm oszthat 42-vel, akkor oszthat 2-vel, 3-mal s 7-tel. Ha 42 osztja van (s a legkisebb ilyet keressk), akkor prmtnyezs alakja: N = p16 $ p22 $ p3 . A szksges prm osztkat gy helyezzk el, hogy a legmagasabb kitevj legyen a legkisebb prm s gy tovbb. A keresett szm: N = 26 $ 32 $ 7 = 4032. 8. E2 Valamely N termszetes szmra d(d(N)) = 5. Legfeljebb hny darab klnbz prm osztja lehet az N szmnak? Ha d ]d]N gg = 5 , akkor d]N g = p 4 = p $ p $ p $ p . Teht N klnbz prm osztinak a szma legfeljebb 4. 9. E1 A kvetkez szmok kzl melyek ngyzetszmok? a) 27 32 76; b) 34 52 116; c) 22 510 136. a) Nem ngyzetszm;
2 b) ^32 $ 5 $ 113h ; 2 c) ^2 $ 55 $ 133h .

10. E2 Melyik az a legkisebb pozitv egsz szm, amellyel a 14 520-at meg kell szoroznunk ahhoz, hogy ngyzetszmot kapjunk? 14 520 = 23 $ 3 $ 5 $ 112 . A keresett szm: 2 $ 3 $ 5 = 30 . 11. E2 Igazoljuk, hogy a kvetkez szm nem lehet ngyzetszm! 20042004 + 20052005 + 20062006. 20042004 utols szmjegye 6, a msodik tag 5-re, a harmadik 6-ra vgzdik. Teht a hromtag sszeg utols szmjegye 6 + 5 + 6 = f7 , amire nem vgzdhet ngyzetszm.

14. Szmrendszerek
1. K1 rjuk fel az albbi szmokat a 10-es szmrendszerben! a) 12 0213; b) 30 5206; c) 50167. a) 12 0213 = 1 $ 3 4 + 2 $ 33 + 0 $ 32 + 2 $ 3 +1 =14210 ; b) 30 5206 = 3 $ 6 4 + 0 $ 63 + 5 $ 62 + 2 $ 6 + 0 $ 1 = 408010 ; c) 50167 = 5 $ 73 + 0 $ 72 + 1 $ 7 + 6 =172810 . 2. K1 rjuk fel a 10-es szmrendszerbeli 976 szmot a a) 2-es szmrendszerben; b) 3-as szmrendszerben! a) 97610 = 11110100002 ; b) 97610 = 11000113 .
9. V F OLYAM

II. ALGEBRA S SZMELMLET

MATEMATIKA 33

3. K2 Vgezzk el a kvetkez mveleteket, s adjuk meg az eredmny 10-es szmrendszerbeli alakjt! a) 110112 + 1011012 + 1111012; b) 21013 + 200213 + 12023. a) 110112 + 1011012 + 1111012 = 27 + 45 + 61 =13310 ; b) 21013 + 200213 +12023 = 64 +169 + 47 = 28010 . 4. E1 Igazoljuk, hogy egy 6-os szmrendszerbeli szm akkor s csak akkor oszthat 5-tel, ha szmjegyeinek sszege is oszthat 5-tel! Egy 6-os szmrendszerben felrt szm ltalnos alakja: cn $ 6 n + cn - 1 $ 6 n - 1 + cn - 2 $ 6 n - 2 + f + c1 $ 6 + c0 . Ezt mg gy is rhatjuk: cn $ ]5 +1gn + cn -1 $ ]5 +1gn -1 + cn - 2 $ ]5 +1gn - 2 + f + c1 $ ]5 +1g + c0 . Az itt szerepl kttag sszegek hatvnyaiban minden tag 5-nek hatvnya (teht oszthat 5-tel), kivve az utols tagokat, amelyek mindegyik esetben 1-esek. Teht az sszeg gy rhat valamilyen K egsz szmmal: 5K + cn + cn -1 + cn - 2 + f + c1 + c0 . Ez pedig akkor s csak akkor oszthat 5-tel, ha az utols tag, vagyis a felrt szm szmjegyeinek sszege oszthat 5-tel. 5. K2 Egy derkszg hromszg oldalai valamilyen x alap szmrendszerben 14x, 40x, 42x. Mekkork a hromszg oldalai a 10-es alap szmrendszerben?

] x + 4g2 + ]4x g2 = ]4x + 2g2 . Innen x1 = 6, x2 = 2. De x = 2 nem lehet, mert a 2-es szmrendszerben nincs 4-es szmjegy, gy csak x = 6 lehet. Ezzel a hromszg oldalai a 10-es szmrendszerben: 146 = 1010, 406 = 2410, 426 = 2610 .
6. E1 Bizonytsuk be, hogy a c alap szmrendszerben azok s csak azok a szmok oszthatk (c 1)-gyel, melyek szmjegyeinek sszege is oszthat (c 1)-gyel! A c alap szmrendszerben felrt szm ltalnos alakja: an c n + an - 1c n - 1 + an - 2 c n - 2 + f + a1c + a0 . rjuk t ezt a kvetkez alakban: an 6]c -1g +1@ n + an - 16]c -1g +1@ n - 1 + an - 2 6]c -1g +1@ n - 2 + f + a16]c -1g + 1@ + a0 . A kttag sszegek hatvnyait kifejtve minden tag oszthat ]c -1g-gyel, kivve az utols tagokat, melyek mindegyike 1. Teht azt kaptuk: ]c -1g $ K + an + an - 1 + an - 2 + f + a1 + a0 . Ez akkor s csak akkor oszthat ]c -1g-gyel, ha an + an - 1 + an - 2 + f + a1 + a0 , vagyis a szmjegyek sszege oszthat ]c -1g-gyel. 7. K2 Hny darab hatjegy szm van a 2-es szmrendszerben? Ezek kzl melyik a legkisebb s melyik a legnagyobb? A legkisebb: 1000002 = 32, a legnagyobb 1111112 = 63, teht a 2-es szmrendszer hatjegy szmainak a szma: 63 - 31 = 32.

9 .

V F O LYA M

III. FGGVNYEK, SOROZATOK

MATEMATIKA 35

III. Fggvnyek, sorozatok


1. Hozzrendelsek, fggvnyek
1. K1 Egy fggvny az A = {0, 2, 4, 6} halmaz mint rtelmezsi tartomny minden elemhez hozzrendeli a nla 1-gyel nagyobb szmot. Adjuk meg a fggvny rtkkszlett!
"1, 3, 5, 7, .

2. K1 Egy fggvny az rtelmezsi tartomny minden elemhez hozzrendeli az abszolt rtkt. Adjuk meg az rtkkszletet, ha az rtelmezsi tartomny a) a [0; 1] intervallum; b) a [1; 1[ intervallum; c) a {1; 1} halmaz; d) a {0; 1} halmaz! a) 60; 1@ ; b) 60; 1@ ; c) ! 1+ ; d) "0; 1, .

3. K1 Egy fggvny az rtelmezsi tartomny minden elemhez hozzrendeli az abszolt rtkt. Mi lehetett az rtelmezsi tartomny, ha az rtkkszlet a) a [0; 1] intervallum; b) a [1; 1[ intervallum; c) az {1} halmaz; d) a {0; 1} halmaz? Jellje A az rtelmezsi tartomnyt. a) Tbbfle megolds lehet. Pldkat mutatunk: A = 6-1; 1@ vagy A = 6-1; 0@ vagy A = ;-1; 1 E , ;0; 1 l . A = {a [1; 0] halmazbl a racionlis szmok, a 60; 1@ intervallumbl 2 2 azok az irracionlis szmok}. b) Az rtkkszlet nem tartalmazhat negatv szmot, ilyen rtelmezsi tartomny nem lehet. c) Lehetett A = !-1+ vagy A = ! 1+ vagy A = "-1; 1, . d) Lehetett A = "-1; 0, vagy A = "0; 1, vagy A = "-1; 0; 1, .

4. K1 Melyik tekinthet sorozatnak a kvetkez fggvnyek kzl? a) A termszetes szmokhoz hozzrendeljk az ellentettjket. b) Az egsz szmokhoz hozzrendeljk az ellentettjket. c) Minden nemnegatv vals szmhoz hozzrendeljk a nla 1-gyel kisebb szmot. d) Minden nemnegatv egsz szmhoz hozzrendeljk a nla 1-gyel kisebb szmot. e) Minden termszetes szmhoz hozzrendeljk a (2)-szerest. A sorozat olyan fggvny, amelynek az rtelmezsi tartomnya a nemnegatv egsz szmok, azaz a termszetes szmok halmaza. Eszerint sorozatnak tekinthet az a), a d) s az e) pontban adott fggvny.

9 .

V F O LYA M

36 MATEMATIKA

III. FGGVNYEK, SOROZATOK

5. K1 Mely fggvnyt szemlltettk az brkon? Adjuk meg az rtelmezsi tartomnyt, rtkkszlett s a hozzrendelst! a) b)

A 0 1 2 3 4 0 1 2

B 0,5 1,5 2,5 3

c)

y 6 5 4 3 2 1 0 1 2 3 4 5 6 7 8 x

d)

b) Az rtelmezsi tartomny: "0; 1; 2; 3; 4, ; az rtkkszlet: "1; 1, 5; 2; 2, 5; 3, ; egy lehetsges hozzrendels: f] ag = a + 1. c) Az rtelmezsi tartomny: "2; 4; 6; 8, ; az rtkkszlet: "2; 3; 4; 5, ; egy lehetsges hozzrendels: f]ag = a +1. 2 d) Az rtelmezsi tartomny: 6-1; 1@ ; az rtkkszlet: 6-1; 0@ ; egy lehetsges hozzrendels: f]ag = a -1. 6. K1 Melyik sknegyedbe esnek a kvetkez pontok? A(1; 1); B(2; 3); C(4; 1); D(1; 1); E(5; 1);

a) Az rtelmezsi tartomny: "-1; 0; 1, ; az rtkkszlet: "0; 1, ; egy lehetsges hozzrendels: f]ag = a2 , egy msik: f]ag = a .

F(5; 1).

Az I. negyedbe: C; a II. negyedbe: A, E; a III. negyedbe: D; a IV. negyedbe: B, F.

9.

V F OLYAM

III. FGGVNYEK, SOROZATOK

MATEMATIKA 37

2. Ponthalmazok a koordinta-rendszerben
1. K1 Milyen tulajdonsggal jellemezhetk a koordinta-rendszerben bejellt pontok? a)
y

b)

1 0 1 x

1 0 1 x

c)

d)

1 0 1 x

1 0 1 x

a) x = y ;

b) x = - y ;

c) x = 3;

d) y = -5 .

2. K1 Milyen tulajdonsggal jellemezhetk a koordinta-rendszerben bejellt pontok? a)


y

b)

1 0 1 x

1 0 1 x

c)

d)

1 0 1 x

1 0 1 x

a) x ! 60;16 ;

b) y ! @2; 56 ;

c) x 2 3;

d) y 1 - x .

9 .

V F O LYA M

38 MATEMATIKA

III. FGGVNYEK, SOROZATOK

3. K1 Jelljk a koordinta-rendszerben a kvetkez tulajdonsgokkal adott pontokat! a) x = 1; b) x < 1; c) x > 1; d) x $ 1; e) y = 3; f) y < 2; g) y # 1; h) y $ 2. a)


y

b)

1 0 1 x

1 0 1 x

c)

d)

1 0 1 x

1 0 1 x

e)

f)

1 0 1 x

1 0 1 x

g)

h)

1 0 1 x

1 0 1 x

9.

V F OLYAM

III. FGGVNYEK, SOROZATOK

MATEMATIKA 39

4. K1 Jelljk a koordinta-rendszerben a kvetkez tulajdonsgokkal megadott pontokat! a) x = y + 1; b) x < y; c) x ! [1; 2]; d) y > 2. a) b)

c)

d)

5. K1 Jelljk a koordinta-rendszerben a kvetkez tulajdonsgokkal megadott pontokat! a) x ! [2; 3], y ! ]1; 4[; b) x $ 3, y ! [1; 0[; c) x # 1, y $ 3. a) b) c)

9 .

V F O LYA M

40 MATEMATIKA

III. FGGVNYEK, SOROZATOK

3. A lineris fggvny
1. K1 Az albbiak kzl mely hozzrendelsek hatroznak meg lineris fggvnyt? rjuk fel ezek m meredeksgt! a) f : R " R ; f : x 7 x ; b) f : R " R ; f : x 7 x -1; c) f : R " R ; f : x 7 2] x -1g ; d) f : R " R ; f : x 7 2x -1; e) f : R " R ; f : x 7 x] x -1g ; f) f : R " R ; f : x 7 x + 2x ; g) f : R " R ; f : x 7 -1; h) f : R " R ; f : x 7 0 . a) lineris, m =1; c) lineris, m = 2; e) nem lineris, x] x -1g = x2 - x ; g) lineris, m = 0 ; b) lineris, m =1; d) lineris, m = 2; f) lineris, m = 3; h) lineris, m = 0 .

2. K1 brzoljuk a kvetkez lineris fggvnyeket kzs koordinta-rendszerben! a) f : x 7 1,5x ; g : x 7 1,5x -1; h : x 7 1,5x + 2; k : x 7 1,5x + 3. b) f : x 7 0,75x ; g : x 7 0,75x -1; h : x 7 0,75x + 2; k : x 7 0,75x + 3. c) f : x 7 x ; g : x 7 x -1; h : x 7 x + 2; k : x 7 x + 3.

a)

b)

c)

3. K1 brzoljuk a kvetkez lineris fggvnyeket kzs koordinta-rendszerben! a) f : x 7 1,5x ; g : x 7 0,5x ; h : x 7 0,75x ; k: x 7 x. b) f : x 7 1,5x + 2; g : x 7 0,5x + 2; h : x 7 0,75x + 2; k : x 7 x + 2. c) f : x 7 1,5x -1; g : x 7 0,5x -1; h : x 7 0,75x -1; k : x 7 x -1. a) b)

9.

V F OLYAM

III. FGGVNYEK, SOROZATOK

MATEMATIKA 41

c)

4. K2 Adjuk meg a kvetkez lineris fggvnyek meredeksgt s tengelymetszeteit, majd brzoljuk ket koordinta-rendszerben! a) f : x 7 2x - 3; b) f : x 7 1 x + 2; c) f : x 7 1 ] x - 1g; 2 2 d) f : x 7 0,75x + 3; e) f : x 7 -2] x -1g; f) f : x 7 -0,6x - 0,6 ; g) f : x 7 1,5x ; h) f : x 7 -3] x +1g; i) f : x 7 3x - 3. a) m = 2, a tengelymetszetek: ^0; -3h s b 3; 0l . 2 1 b) m = , a tengelymetszetek: ^0; 2h s ^-4; 0h . 2 c) m = 1 , a tengelymetszetek: b0; - 1 l s ^1; 0h . 2 2 d) m = 0,75 , a tengelymetszetek: ^0; 3h s ^-4; 0h . e) m = -2, a tengelymetszetek: ^0; 2h s ^1; 0h . f) m = -0,6 , a tengelymetszetek: ^0; -0,6h s ^-1; 0h . g) m = 1,5 , a tengelymetszetek: ^0; 0h . h) m = -3, a tengelymetszetek: ^0; -3h s ^-1; 0h . i) m = 3, a tengelymetszetek: ^0; -3h s ^1; 0h .

a)

b)

c)

d)

e)

f)

9 .

V F O LYA M

42 MATEMATIKA

III. FGGVNYEK, SOROZATOK

g)

h)

i)

1 0 1 x

5. K2 Hatrozzuk meg, hogy melyek azok a lineris fggvnyek, amelyek tengelymetszete: a) (0; 1) s (1; 0); b) (0; 2) s (1; 0); c) (0; 1) s (2; 0); d) (0; 1) s (2; 0)! a) m = -1, b = 1, f] x g = - x +1; c) m = 1, b = -1, f] x g = 1 x -1; 2 2 b) m = 2, b = 2, f] x g = 2x + 2; d) m = 1, b = 1, f] x g = 1 x +1. 2 2

6. K1 Mely sszefggsek hatroznak meg lineris fggvnyt? a) Egy idtartamhoz hozzrendeljk azt az utat, amelyet egy aut azalatt az id alatt megtesz. b) Egy idtartamhoz hozzrendeljk azt az utat, amelyet egy egyenletes sebessggel halad aut azalatt az id alatt megtesz. c) Egy adott ru darabszmhoz hozzrendeljk az rurt zetend sszeget. d) Egy adott vhez hozzrendeljk az abban az vben Magyarorszgon szletettek szmt. e) Egy adott naphoz hozzrendeljk az aznap Magyarorszgon mrt legmagasabb hmrskletet. f) Egy adott v minden egyes napjhoz hozzrendeljk az adott v napjai szmt. a) Nem lineris, mert nem felttlenl egyenletes az aut sebessge. b) Lineris, mert az aut sebessge egyenletes. Ez ppen azt jelenti, hogy a megtett t az eltelt idvel arnyosan vltozik. c) Lineris. A kifizetend sszeg az ru darabszmval arnyosan vltozik. d) Nem lineris. e) Nem lineris. f) Lineris, konstans. 7. K2 Milyen y, illetve x tengely irny eltolssal kaphatk a) az f : x 7 x fggvny grakonjbl a g : x 7 x +1; h : x 7 x - 3; k : x 7 x + 2; b) az f : x 7 2x fggvny grakonjbl a g : x 7 2x +1; h : x 7 2x - 3; k : x 7 2x + 2; c) az f : x 7 2x fggvny grakonjbl a g : x 7 2] x +1g; h : x 7 2] x - 3g; k : x 7 2] x + 2g; d) az f : x 7 -2x fggvny grakonjbl a g : x 7 -2x +1; h : x 7 -2x - 3; k : x 7 -2x + 2; e) az f : x 7 -2x fggvny grakonjbl a g : x 7 -2] x +1g; h : x 7 -2] x - 3g; k : x 7 -2] x + 2g fggvnyek grakonjai? Az rtkeket a tengelymetszetekbl olvashatjuk le: a) y tengellyel prhuzamosan 1; -3; 2, x tengellyel prhuzamosan -1; 3; -2. b) y tengellyel prhuzamosan 1; -3; 2, x tengellyel prhuzamosan - 1; 3; -1. 2 2 c) y tengellyel prhuzamosan 2; -6; 4 , x tengellyel prhuzamosan -1; 3; -2. d) y tengellyel prhuzamosan 1; -3; 2, x tengellyel prhuzamosan 1; - 3;1. 2 2 e) y tengellyel prhuzamosan -2; 6; -4 , x tengellyel prhuzamosan -1; 3; -2.

9.

V F OLYAM

III. FGGVNYEK, SOROZATOK

MATEMATIKA 43

4. Az abszoltrtk-fggvny
1. K1 brzoljuk koordinta-rendszerben a kvetkez abszoltrtk-fggvnyeket! Adjuk meg a kapott abszoltrtk-fggvnyek grakonjnak cscspontjt a koordintikkal! a) x 7 x ; b) x 7 x +1; c) x 7 x +1 ; d) x 7 2x ; e) x 7 2x +1; i) x 7 - 1 x -1 ; 2 f) x 7 2x +1 ; j) x 7 - 1 x ; 2 g) x 7 - 1 x ; 2 k) x 7 - 1 x - 3; 2 h) x 7 - 1 x -1; 2 l) x 7 - 1 x - 3 . 2

a)

b)

c)

d)

e)

f)

1 0 1 x

g)

h)

i)

j)

k)

l)

A cscspontok koordinti: a) (0; 0); g) (0; 0); b) (0; 1); h) (0; 1); c) (1; 0); i) (2; 0); d) (0; 0); j) (0; 0); e) (0; 1); k) (0; 3); f) b- 1; 0l ; 2 l) ^-6; 0h .

9 .

V F O LYA M

44 MATEMATIKA

III. FGGVNYEK, SOROZATOK

2. E1 Mely abszoltrtk-fggvnyek grakonjt lthatjuk az brkon?

a)

b)

c)

1 0 1 x

d)

e)

f)

a) x -1;

b) x -1 ;

c) 2 x ;

d) 2 x -1;

e) x - 2 ;

f) 2x - 2 .

3. K2 Milyen y vagy x tengely irny eltolssal kaphatk a) az f : x 7 x fggvny grakonjbl a g : x 7 x +1; h : x 7 x - 3; k : x 7 x + 2; b) az f : x 7 x fggvny grakonjbl a g : x 7 x +1 ; h : x 7 x - 3 ; k : x 7 x + 2 ; c) az f : x 7 2 x fggvny grakonjbl a g : x 7 2 x +1; h : x 7 2 x - 3; k : x 7 2 x + 2; d) az f : x 7 2x fggvny grakonjbl a g : x 7 2x +1 ; h : x 7 2x - 3 ; k : x 7 2x + 2 fggvnyek grakonjai? Adjuk meg a kapott abszoltrtk-fggvnyek grakonjainak cscspontjait a koordintikkal! a) A g fggvnyt y irny 1 egysgnyi eltolssal kapjuk; a cscspontja ^0; 1h . A h fggvnyt y irny -3 egysgnyi eltolssal kapjuk; a cscspontja ^0; -3h . A k fggvnyt y irny 2 egysgnyi eltolssal kapjuk; a cscspontja ^0; 2h . b) A g fggvnyt x irny -1 egysgnyi eltolssal kapjuk; a cscspontja ^-1; 0h . A h fggvnyt x irny 3 egysgnyi eltolssal kapjuk; a cscspontja ^3; 0h . A k fggvnyt x irny -2 egysgnyi eltolssal kapjuk; a cscspontja ^-2; 0h . c) A g fggvnyt y irny 1 egysgnyi eltolssal kapjuk; a cscspontja ^0; 1h . A h fggvnyt y irny -3 egysgnyi eltolssal kapjuk; a cscspontja ^0; -3h . A k fggvnyt y irny 2 egysgnyi eltolssal kapjuk; a cscspontja ^0; 2h . d) A g fggvnyt x irny - 1 egysgnyi eltolssal kapjuk; a cscspontja b- 1; 0l . 2 2 A h fggvnyt x irny 3 egysgnyi eltolssal kapjuk; a cscspontja b 3; 0l . 2 2 A k fggvnyt x irny -1 egysgnyi eltolssal kapjuk; a cscspontja ^-1; 0h . 4. K2 Milyen arny y, illetve x tengely irny nyjts (sszenyoms) viszi az f : x 7 x fggvny grakonjt a g : x 7 2x ; h : x 7 3x ; k : x 7 1 x fggvnyek grakonjba? 2 y irnyban 2, 3, 1 , x irnyban 1, 1, 2 arny nyjts. 2 2 3

9.

V F OLYAM

III. FGGVNYEK, SOROZATOK

MATEMATIKA 45

5. K2 Az abszoltrtk-fggvny grakonjn az albbi transzformcikat hajtottuk vgre. llaptsuk meg, hogy melyik fggvnyt kaptuk! a) Az x tengellyel prhuzamosan eltoltuk 2 egysggel, majd az x tengelytl szmtva, az y tengellyel prhuzamosan a 2-szeresre nyjtottuk. b) Az x tengelytl szmtva a felre nyomtuk ssze, majd az y tengellyel prhuzamosan eltoltuk 1 egysggel. a) Az els lps utn az x 7 x - 2 fggvnyt kaptuk, a msodik utn az x 7 2 x - 2 fggvnyt. b) Az els lpsben az x 7 1 x fggvnyt kaptuk, a msodik utn az x 7 1 x -1 fggvnyt. 2 2 6. E1 Ksztsnk rtktblzatot, majd brzoljuk a kvetkez abszoltrtk-fggvnyeket! a) a] x g = x + x ; 2 b) b] x g = c) c] x g = x +1 + x +1 ; 2 x +1 + x -1 . 2 3 0 0 3 2 0 0 2 1 0 0 1 0 0 1 1 1 1 2 1 2 2 3 2 3 3 4 3 4 4 5 4

x a(x) b(x) c(x)

4 0 0 4

a)

b)

c)

9 .

V F O LYA M

46 MATEMATIKA

III. FGGVNYEK, SOROZATOK

5. Az f: x 7 x2 fggvny
1. K1 brzoljuk a kvetkez fggvnyeket kzs koordinta-rendszerben! Fogalmazzuk meg, hogy a g, h, k fggvnyek grakonjt milyen transzformcival kaphatjuk meg az f fggvny grakonjbl! a) f : x 7 x2 ; h : x 7 0,1x2 ; k : x 7 -x 2 ; g : x 7 1 x2 ; 2 b) f : x 7 x2 ; g : x 7 b 1xl ; 2
2

h : x 7 ^0,1x h2 ;

k : x 7 ]-x g2 .

a) g: y tengely irny, 1 -szeresre vltoztats; h: y tengely irny, 0,1-szeresre vltoztats; 2 k: y tengely irny, -1-szeresre vltoztats, vagyis az x tengelyre trtn tkrzs. b) g: y tengely irny, 1 -szeresre vltoztats; h: y tengely irny, 0,01-szorosra vltoztats; 4 k: f-fel azonos.

a)

b)

2. K1 brzoljuk a kvetkez fggvnyeket kzs koordinta-rendszerben! Fogalmazzuk meg, hogy a g, h, k fggvnyek grakonjt milyen transzformcival kaphatjuk meg az f fggvny grakonjbl! a) f : x 7 x2 ; g : x 7 x2 -1; h : x 7 x2 +1; k : x 7 x2 + 3 ; 2 2 2 b) f : x 7 x ; g : x 7 ] x -1g ; h : x 7 ] x +1g ; k : x 7 ] x + 3g2 . a) g: y tengellyel prhuzamosan 1-gyel eltoljuk. h: y tengellyel prhuzamosan 1-gyel eltoljuk. k: y tengellyel prhuzamosan 3-mal eltoljuk. b) g: x tengellyel prhuzamosan 1-gyel eltoljuk. h: x tengellyel prhuzamosan 1-gyel eltoljuk. k: tengellyel prhuzamosan 3-mal eltoljuk. a) b)

9.

V F OLYAM

III. FGGVNYEK, SOROZATOK

MATEMATIKA 47

3. K2 Mely fggvnyek grakonjt kapjuk meg, ha az x2 fggvny grakonjn a kvetkez transzformcikat hajtjuk vgre? a) Az y tengely mentn + 3-mal eltoljuk. b) Az x tengely irnyban 3-szorosra nyjtjuk. c) Az x tengely mentn 2-vel eltoljuk. d) Az y tengely mentn 1 -szeresre sszenyomjuk. 2 a) x2 + 3;
2 b) b 1 x l ; 3

c) ] x + 2g2 ;

d) 1 x . 2

4. K1 Az x 7 x2 fggvny grakonjnak milyen eltolsval kaphatk a kvetkez fggvnyek grakonjai? Hatrozzuk meg a parabolk cscspontjainak koordintit! a) f(x) = x2 - 3; b) f(x) = x2 +1; c) f(x) = ] x + 3g2 ; d) f(x) = ] x - 2g2 ; 2 2 2 e) f(x) = ] x +1g -1; f) f(x) = ] x + 3g + 3; g) f(x) = ] x - 2g +1; h) f(x) = ] x - 4g2 -1. a) b) c) d) e) f) g) h) y y x x x x x x tengellyel tengellyel tengellyel tengellyel tengellyel tengellyel tengellyel tengellyel prhuzamosan prhuzamosan prhuzamosan prhuzamosan prhuzamosan prhuzamosan prhuzamosan prhuzamosan 3 egysggel ^0; -3h ; 1 egysggel ^0; 1h ; 3 egysggel ^-3; 0h ; 2 egysggel ^2; 0h ; 1, y tengellyel prhuzamosan 1 egysggel ^-1; -1h ; 3, y tengellyel prhuzamosan 3 egysggel ^-3; 3h ; 2, y tengellyel prhuzamosan 1 egysggel ^2; 1h ; 4, y tengellyel prhuzamosan 1 egysggel ^4; -1h .

6. A ngyzetreemelsfggvny sszetett msodfok fggvny sszetett transzformcii


1. K1 brzoljuk a kvetkez fggvnyeket kzs koordinta-rendszerben! Fogalmazzuk meg, hogy a g, h, k fggvnyek grakonjt milyen transzformcival kaphatjuk meg az f fggvny grakonjbl! a) f : x 7 x2 ; h : x 7 0,1x2 -1; k : x 7 -x 2 + 2 ; g : x 7 1 x2 +1; 2 b) f : x 7 x2 ; g : x 7 b 1 x l +1; 2
2

h : x 7 ^0,1x h2 -1;

k : x 7 ]-x g2 + 2.

a)

b)

y f

g k

1 0 1 h x

a) g: y tengely mentn 1 -szeresre vltoztatjuk s az y tengellyel prhuzamosan 1-gyel eltoljuk; 2 h: y tengely mentn 0,1-szeresre vltoztatjuk s az y tengellyel prhuzamosan 1-gyel eltoljuk;
9 . V F O LYA M

48 MATEMATIKA

III. FGGVNYEK, SOROZATOK

k: y tengely irnyban 1-szeresre vltoztatjuk, vagyis tkrzzk az x tengelyre, s az y tengellyel prhuzamosan 2-vel eltoljuk. b) g: y tengely irnyban 1 -szeresre vltoztatjuk s az y tengellyel prhuzamosan 1-gyel 4 eltoljuk; h: y tengely irnyban 0,01-szorosra vltoztatjuk s az y tengellyel prhuzamosan 1-gyel eltoljuk; k: y tengellyel prhuzamosan 2-vel eltoljuk. 2. K2 a) Hny y tengelymetszete lehet egy parabolnak? b) Hny y tengelymetszete lehet ltalban egy fggvnynek? Egy f fggvny y tengelymetszete a ^0; f]0gh pont. Mivel a 0-hoz egyetlen fggvnyrtk tartozik, ez egyetlen pont lehet. Ha a 0 nem eleme az rtelmezsi tartomnynak, akkor nincs tengelymetszet. (Pldul: minden pozitv szmhoz hozzrendeljk a nla 1-gyel kisebb szmot.) a) A parabolnak mindig egyetlen y tengelymetszete van. b) Egy f fggvnynek vagy 1 y tengelymetszete b) Egy f fggvnynek 0 vagy 1 0 y tengelymetszete lehet. lehet. 3. E1 brzoljuk a kvetkez fggvnyeket kzs koordinta-rendszerben! Fogalmazzuk meg, hogy a g, h, k fggvnyek grakonjt milyen transzformcival kaphatjuk meg az f fggvny grakonjbl! 2 a) f : x 7 x2 ; k : x 7 b 1 x l + 3; g : x 7 ]2x -1g2 -1; h : x 7 ]-x +1g2 +1; 2 1 2 2 2 b) f : x 7 x ; g : x 7 2] x -1g ; h : x 7 - ] x +1g ; k : x 7 ] x + 3g2 . 2 a) g: x tengellyel prhuzamosan 1 -del eltoljuk, y tengely irnyban 4-szeresre vltoztatjuk, 2 majd az y tengellyel prhuzamosan 1-gyel eltoljuk; h: x tengellyel prhuzamosan eltoljuk 1-gyel, majd az y tengellyel prhuzamosan 1-gyel eltoljuk; k: y tengely irnyban 1 -szeresre 4 vltoztatjuk, majd az y tengellyel prhuzamosan 3-mal eltoljuk. b) g: x tengellyel prhuzamosan 1-gyel eltoljuk, majd az y tengely irnyban 2-szeresre vltoztatjuk; h: x tengellyel prhuzamosan 1-gyel eltoljuk, az y tengely irnyban 1-szeresre vltoztatjuk; k: x tengellyel prhuzamosan 3-mal eltoljuk, az y tengely irnyban 1 -szeresre vltoztatjuk. 2

a)

b)

9.

V F OLYAM

III. FGGVNYEK, SOROZATOK

MATEMATIKA 49

4. E1 Adjuk meg a kvetkez msodfok fggvnyek grakonjnak cscspontjait koordintikkal, illetve a tengelymetszeteiket! a) f : x 7 ]2x -1g2 -1; b) f : x 7 ]-x -1g2 -1; c) f : x 7 1 x2 - 2; d) f : x 7 2] x -1g2 - 2; 2 e) f : x 7 -] x -1g2 ; f) f : x 7 1 ] x -1g2 + 2. 2 a) A cscspont koordinti: b 1; -1l ; az y tengelymetszet: ^0; 0h ; az x tengelymetszetek: ^0; 0h ; 2 ^1; 0h . b) A cscspont koordinti: ^-1; -1h ; az y tengelymetszet: ^0; 0h ; az x tengelymetszetek: ^0; 0h ; ^-2; 0h . c) A cscspont koordinti: ^0; -2h ; az y tengelymetszet: ^0; -2h ; az x tengelymetszetek: ^2; 0h ; ^-2; 0h . d) A cscspont koordinti: ^1; -2h ; az y tengelymetszet: ^0; -2h ; az x tengelymetszetek: ^0; 0h ; ^2; 0h . e) A cscspont koordinti: ^1; 0h ; az y tengelymetszet: ^0; -1h ; az x tengelymetszet: ^1; 0h . f) A cscspont koordinti: ^1; 2h ; az y tengelymetszet: ^0; 2,5h ; az x tengelyt nem metszi, mert a cscspontja az I. sknegyedbe esik, s felfele llnak a szrai.

7. Tovbbi fggvnyek
1. K1 brzoljuk koordinta-rendszerben a kvetkez hozzrendelseket! Ksztsnk rtktblzatokat! a) a : x 7 2x ; b) b : x 7 2 x ; c) c : x 7 1 x ; d) d : x 7 1 x . 2 2 Az x 7 x fggvny grakonjnak milyen transzformcijval kapjuk meg ezeknek a fggvnyeknek a grakonjt?

a)

b)

c)

d)

a) x tengely irny 1 -szeresre vltoztats. 2 c) y tengely irny 1 -szeresre vltoztats.

b) y tengely irny 2-szeresre vltoztats. d) x tengely irny 2-szeresre vltoztats.

9 .

V F O LYA M

50 MATEMATIKA

III. FGGVNYEK, SOROZATOK

2. K1 brzoljuk koordinta-rendszerben a kvetkez hozzrendelseket! Ksztsnk rtktblzatokat! a) a : x 7 x + 1; b) b : x 7 x + 1; c) c : x 7 x - 3; d) d : x 7 x - 3 . Mindegyik fggvny esetben adjuk meg a lehet legbvebb rtelmezsi tartomnyt! Az x 7 x fggvny grakonjnak milyen transzformcijval kapjuk meg ezeknek a fggvnyeknek a grakonjt?

a)

b)

c)

d)

a) b) c) d)

x $ 0 ; y tengely mentn 1 egysggel trtn eltols. x $ -1; x tengely mentn ]-1g egysggel trtn eltols. x $ 0 ; y tengely mentn ]-3g egysggel trtn eltols. x $ 3; x tengely mentn 3 egysggel trtn eltols.

3. E1 brzoljuk koordinta-rendszerben a kvetkez hozzrendelseket! Ksztsnk rtktblzatokat! a) a : x 7 2x + 1; b) b : x 7 2 x + 1; c) c : x 7 9x - 3; d) d : x 7 9 x - 3 . Mindegyik fggvny esetben adjuk meg a lehet legbvebb rtelmezsi tartomnyt! Az x 7 x fggvny grakonjnak milyen transzformcijval kapjuk meg ezeknek a fggvnyeknek a grakonjt?

a)

b)

9.

V F OLYAM

III. FGGVNYEK, SOROZATOK

MATEMATIKA 51

c)

d)

a) x $ 0 ; x tengely irnyban 1 -szeresre vltoztats (vagy y tengely irnyban 2

2 -szeresre

vltoztats, mert 2x = 2 $ x ), majd y tengely mentn 1 egysggel trtn eltols. b) x $ -1; x tengely mentn ]-1g egysggel trtn eltols, majd y tengely irnyban 2-szeresre vltoztats. c) x $ 0 ; x tengely irnyban 1 -szeresre val vltoztats (vagy y tengely irnyban 3-szorosra vltoztats), majd y tengely mentn ]-3g egysggel trtn eltols. d) x $ 3; x tengely mentn 3 egysggel trtn eltols, majd y tengely irnyban 9-szeresre trtn vltoztats. 4. K2 A g fggvny a t terlet ngyzet terlethez hozzrendeli a ngyzet tljnak hosszt. Ksztsnk rtktblzatot! brzoljuk a g fggvnyt koordinta-rendszerben! Mivel a ngyzet t terlete a d tlbl gy kaphat, hogy 2 t = d , d = 2t , a hozzrendels pedig a t 7 2t . 2

5. K1 Adjuk meg, hogy a kvetkez mennyisgprok kzl melyek llnak egymssal egyenes arnyban, melyek fordtott arnyban! Melyek azok, amelyek sem egyenes, sem fordtott arnyban nem llnak egymssal? a) Rgztett terlet tglalapok kt oldalnak a hossza. b) Rgztett kerlet tglalapok kt oldalnak a hossza. c) Rgztett terlet ngyzetek oldalnak s tljnak a hossza. d) Egy szm klnbz trt alakjainak szmllja s nevezje. e) Kt ikertestvr letkora. f) Kt ikertestvr tmege. Egyenes arnyossg: c), d), e). Fordtott arnyossg: a). Semelyik sem: b), f).

9 .

V F O LYA M

52 MATEMATIKA

III. FGGVNYEK, SOROZATOK

6. K1 Egy tglalap terlete 40 terletegysg. Szemlltessk koordinta-rendszerben azt a hozzrendelst, amely az egyik oldal fggvnyben megadja a msik oldalt! Milyen kapcsolat ll fenn a kt oldal kztt?

Fordtott arnyossg. 7. E1 brzoljuk koordinta-rendszerben a kvetkez fggvnyeket! Ksztsnk rtktblzatokat! a) a : x 7 52x ? ; b) b : x 7 2 5 x ? ; c) c : x 7 1 5 x ? ; d) d : x 7 ; 1 x E . 2 2 Hasonltsuk ssze a fggvnyek grakonjt az x 7 5 x ? hozzrendels grakonjval!

a)

b)

c)

d)

a) x tengely irny 1 -szeresre vltoztats. 2 c) y tengely irny 1 -szeresre vltoztats. 2

b) y tengely irny 2-szeresre vltoztats. d) x tengely irny 2-szeresre vltoztats.

9.

V F OLYAM

III. FGGVNYEK, SOROZATOK

MATEMATIKA 53

8. E1 brzoljuk koordinta-rendszerben a kvetkez fggvnyeket! Ksztsnk rtktblzatokat! a) a : x 7 !2x + ; b) b : x 7 2 ! x + ; c) c : x 7 1 ! x + ; d) d : x 7 & 1 x 0 . 2 2 Hasonltsuk ssze a fggvnyek grakonjt az x 7 ! x + hozzrendels grakonjval!

a)

b)

c)

d)

a) x tengely irny 1 -szeresre vltoztats. 2 c) y tengely irny 1 -szeresre vltoztats. 2

b) y tengely irny 2-szeresre vltoztats. d) x tengely irny 2-szeresre vltoztats.

9. E1 Az f fggvny minden vals szmhoz hozzrendeli az egszekre kerektett rtkt. brzoljuk koordinta-rendszerben a hozzrendelst! Ksztsnk rtktblzatot!

10. E2 Nevezzk egszrekerekts-fggvnynek azt az f hozzrendelst, amely minden vals szmhoz az egszre kerektett rtkt rendeli! Milyen fggvnytranszformci viszi az egszrsz-fggvnyt az egszrekerekts-fggvnybe? x tengely irny - 1 egysgnyi eltols. 2

9 .

V F O LYA M

MATEMATIKA 55

IV. Bevezets a geometriba


1. Pontok, egyenesek, skok
1. K1 Adott ngy pont egy skban, melyek kzl semelyik hrom nem illeszkedik egy egyenesre. Adott tovbb egy tdik pont, amely nem illeszkedik erre a skra. Hny skot hatroz meg az gy megadott t pont? Az els ngy pont (A, B, C, D) meghatroz egy skot. Mivel kzlk semelyik hrom pont nem illeszkedik egy egyenesre, ezrt brmelyik kett (AB, AC, AD, BC, BD, CD) az tdik E ponttal meghatroz egy skot. Ez sszesen hat. Vagyis a feladatban szerepl t pont ht skot hatroz meg. 2. K1 Az A, B, C, D, E pontok nincsenek egy skban, s semelyik hrom nincs egy egyenesen. Ezen pontok ltal meghatrozott egyenesek vagy skok szma a tbb? Az t pontbl brmelyik hrmat kivlasztva, azok meghatroznak egy skot. A sk meghatrozshoz ki nem vlasztott kt pont ugyanakkor meghatroz egy egyenest. (Pl.: ABD sk s CE egyenes.) Vagyis ugyanannyi egyenest hatroznak meg a feladatban szerepl pontok, mint skot. 3. K2 Hny egyenest hatroznak meg sszesen az brn lthat A, B, C, D, E, F, G, H pontok?

a)
H G

b)
H G

C D

C D

a) Az A, B, C, D, E, F pontok egy egyenesre illeszkednek, valamint a G s a H is meghatroz egy egyenest. Tovbbi egyenesek: HA, HB, , HF, ami hat darab, illetve GA, GB, , GF, ami szintn hat darab. Ez sszesen 14 egyenes. b) Az A, B, C, D, E, F pontok egy egyenesre illeszkednek, valamint az F, G, H pontok is. Tovbbi egyenesek: HA, HB, , HE, ami t darab, illetve GA, GB, , GE, ami szintn t darab. Ez szszesen 12 egyenes. 4. K1 Adott a trben t prhuzamos egyenes, melyek kzl brmely hrom nem esik egy skba. Hny skot hatroznak meg? Brmely kett meghatroz egy skot. Vagyis sszesen t skot hatroznak meg. 5. K2 Adott a trben hrom egyenes s rjuk nem illeszked ngy pont. Legfeljebb hny sklap illeszthet ezekre, ha minden sk egy egyenest s egy pontot tartalmaz az adottak kzl? Egy egyenes brmely r nem illeszked ponttal meghatroz egy skot. Vagyis legfeljebb 12 sk kpzelhet el. (Ha valamelyik pontegyenes pros ltal meghatrozott skra illeszkedik egy tovbbi pont vagy egyenes, akkor 12-nl kevesebb lesz a skok szma.)

9 .

V F O LYA M

56 MATEMATIKA

I V. B E V E Z E T S A G E O M E T R I B A

2. Szakasz, flegyenes, szg


1. K1 Az A, B, C hrom klnbz pont egy egyenesre illeszkedik. Hatrozzuk meg az AC szakasz hosszt, ha AB = BC = 19 cm! A feladat szvegbl kiderthet, hogy a B pont az AC szakasz felezpontja. Vagyis AC = 38 cm. 2. K2 Az A, B, C hrom klnbz pont egy egyenesre illeszkedik. Hatrozzuk meg az AC szakasz legkisebb s legnagyobb hosszt, ha AB = 19 cm s BC = 61 cm! Kt eset lehetsges: II. Az A pont illeszkedik a BC szakaszra.Ekkor a kt szakasz klnbsgvel egyenl a hossz: AC = 42 cm. II. Az A pont a BC szakasz B-n tli meghosszabbtsra illeszkedik. Ekkor a kt szakasz sszegvel egyenl a hossz: AC = 80 cm. 3. E1 Az A, B, C, D pontok ebben a sorrendben egy egyenesre illeszkednek. Igazoljuk, hogy AC $ CD + BC $ AB = AB $ BD + BC $ CD! Legyen AB = x, BC = y, CD = z . Ekkor a bizonytand llts a kvetkez alakban rhat: ^ x + y h z + yx = x ^ y + z h + yz . A zrjelek felbontsa utn lthat, hogy mindkt oldalon xy + yz + xz szerepel. 4. K1 Szmoljuk ki az a + b, a b + c s az a + c szgek nagysgt, ha a = 37 45 34, b = 28 54 48, c = 91 22 49! a + b = 37o 45l 34m + 28o 54l 48m = 65o 99l 82m = 66o 40l 22m . b + c = 28o 54l 48m + 91o 22l 49m = 119o 76l 97m = 120o 17l 37m . c + a = 91o 22l 49m + 37o 45l 34m = 128o 67l 83m = 129o 8l 23m . 5. K1 Szmoljuk ki az a s a b szgek nagysgt, ha a + b = 72 s a b = 14 32!
^a + bh + ^a - bh 72o + 14o 32l 86o 32l 43o 16l . = = = a , ezrt a = 2 2 2 ^a + bh - ^a - bh 72o - 14o 32l 57o 28l 28o 44l . Mivel = = = b , ezrt b = 2 2 2

Mivel

6. K2 Az a s a b egyms mellkszge. a) Mekkork ezek a szgek, ha az a szg 12 17-cel nagyobb, mint a b? b) Mekkork ezek a szgek, ha az arnyuk 2 : 7? c) Mekkork ezek a szgek, ha az a a b-nl b-val nagyobb?
o o o l l a) b = 180 - 12 17 = 167 43 = 83o 51l 30m , 2 2 a = 83o 51l 30m + 12o 17l = 95o 68l 30m = 96o 8l 30m . (Az a-t szmolhatnnk gy is: a = 180o - b = 180o - 83o 51l 30m = 96o 8l 30m .) o o b) a = 2 $ 180 = 40o , b = 7 $ 180 = 140o . 9 9

(A b-t szmolhatnnk gy is: b = 180o - a = 180o - 40o = 140o .) c) Vagyis a = 2b , ezrt a + b = 3b = 180o . Azaz b = 60o, a =120o . 7. K1 t szg egytt teljesszget alkot. Mindegyik szg az elznl 12-kal nagyobb. Szmtsuk ki a legkisebb szg nagysgt! Legyen a legkisebb szg a. Ekkor a + ^a + 12oh + ^a + 24oh + ^a + 36oh + ^a + 48oh = 360o . Az sszevonsok utn: 5a + 120o = 360o . Vagyis a = 48o .
9. V F OLYAM

I V. B E V E Z E T S A G E O M E T R I B A

MATEMATIKA 57

8. K2 Mekkora szget zr be az ra kt mutatja a) 4 rakor; b) 5 rakor; c) fl tkor; d) hromnegyed 12-kor? a) A teljesszg harmadt, azaz 120-ot. b) A teljesszg 5 -t, azaz 150-ot. 12 c) A teljesszg 3 -t ( 1 -t), azaz 45-ot. 24 8 d) A teljesszg 11 -t, azaz 82,5-ot. 48 a)
11 10 9 8 7 6 5 4 12

b)
1 2 3 9 8 7 10 11

12

c)
1 2 3 4 9 8 7 10 11

12

d)
1 2 3 4 9 8 7 10 11

12

1 2 3 4

9. K2 Fejezzk ki fok, perc, msodperc alakban a kvetkez szgeket: a) 62,5; b) 15,3; c) 31,45; d) 90,55! a) 62 30; b) 15 18; c) 31 27; d) 90 33.

10. K2 Fejezzk ki fokokban a kvetkez szgeket: a) 12 30; b) 65 45; c) 23 7 30; d) 51 11 15! a) 12,5; b) 65,75; c) 23,125; d) 51,1875!

11. K2 Egy forgsszg egyik szrt rgztjk, a msik szrt a cscspontja krl forgatjuk. Elszr az ramutat jrsval ellenttes irnyban 120 21-es szggel, aztn az ramutat jrsval megegyez irnyban 17 32-es szggel, vgl ismt az ramutat jrsval ellenttes irnyban 65 47-es szggel forgatjuk el. Mekkora a hrom forgsszg sszege? Felrjuk a mutat forgatst eljeles szgekkel: 120o 21l - 17o 32l + 65o 47l = 102o 49l + 65o 47l = 168o 36l . Vagyis a hrom forgsszg sszege: 168 36. 12. K1 Keressnk az brn nevezetes szgprokat!

9 .

V F O LYA M

58 MATEMATIKA

I V. B E V E Z E T S A G E O M E T R I B A

A tanult szgprokra egy-egy pldt adunk (termszetesen ennl tbb is lthat az brn):

Mellkszgek: a s b. Egylls szgek: c s d. Vltszgek: g s h. Cscsszgek: f s b. Ptszgek: n s h. Kiegszt szgek: d s m. Merleges szr konvex szgek: n s ].

3. Hromszgek
1. K1 Ltezik-e olyan hromszg, amelynek oldalhosszai: a) 17, 19, 35; b) 42, 11, 54; 7 9 1 5, 2? c) , , ; d) 8 , 3 4 12 5 3 15 Hasznljuk a hromszg-egyenltlensget! a) Mivel 17 1 19 + 35, 19 1 17 + 35, 35 1 17 + 19 , ezrt ilyen hromszg ltezik. (A harmadik egyenltlensg egyedl is igazolja a hromszg ltezst, mert a kt rvid oldal sszege nagyobb, mint a harmadik oldal.) b) Mivel 42 + 11 1 54 , ezrt ilyen hromszg nem ltezik. c) Mivel 7 = 9 + 1 , ezrt ilyen hromszg nem ltezik. 3 4 12 d) Mivel 8 1 5 + 2 , 5 1 8 + 2 , 2 1 5 + 8 , ezrt ilyen hromszg ltezik. 5 3 15 3 5 15 15 3 5 2. K2 Mutassuk meg, hogy minden hromszgben kivlaszthat kt olyan oldal, amelyeknek az sszege nem kisebb, mint a harmadik oldal ktszerese! A hromszg oldalai kzl legyen a az, amelyiknl nincs rvidebb, s legyen c az, amelyiknl nincs hosszabb: a # b # c . Ebbl kvetkezik, hogy 2a # 2b # b + c . 3. E1 Igazoljuk, hogy minden konvex ngyszgben a hosszabb tl hosszabb a legrvidebb oldalnl!

D d x A v

c z P y a

Legyen az ABCD konvex ngyszg tlinak metszspontja P. rjunk fel hromszg-egyenltlensget az ABP, BCP, CDP s DAP hromszgekre: a 1 x + y, b 1 y + z , c 1 z + v , d 1 v + x . A ngy egyenltlensg megfelel oldalait sszeadva kapjuk, hogy a + b + c + d 1 x + y + y + z + z + v + v + x. Mivel x + z az egyik tl hossza, y + v pedig a msik tl hossza, gy az egyenltlensg jobb oldaln a kt tl sszegnek ktszerese lthat. (Vagyis a + b + c + d 1 2e + 2f .) Az egyenltlensg bal oldaln ll a + b + c + d sszegnl nem nagyobb a legrvidebb oldal ngyszerese. (Ha a-nl nincs rvidebb oldal, akkor 4a # a + b + c + d .) Az egyenltlensg jobb oldaln ll kt tl sszegnek ktszeresnl nem kisebb a hosszabb tl ngyszerese. (Ha f-nl nincs hosszabb tl, akkor 2 ]e + f g # 4f .) Vagyis valban minden konvex ngyszgben a hosszabb tl hosszabb a legrvidebb oldalnl.

9.

V F OLYAM

I V. B E V E Z E T S A G E O M E T R I B A

MATEMATIKA 59

4. E2 Igazoljuk az ABC hromszg belsejben lv minden P pontra, hogy a PA + PB + PC sszeg nagyobb, mint a hromszg flkerlete! Tudjuk a hromszg-egyenltlensgek miatt, hogy AB 1 AP + BP, BC 1 BP + CP, CA 1 CP + AP . A hrom egyenltlensg megfelel oldalait sszeadjuk, majd elosztjuk 2-vel, s a bizonytand lltst kapjuk. 5. E2 Igazoljuk az ABCD tglalap belsejben lv minden P pontra, hogy a PA + PB + PC + PD sszeg nem kisebb, mint a tglalap a) flkerlete; b) tljnak ktszerese! a) Tudjuk a hromszg-egyenltlensgek miatt, hogy AB 1 AP + BP , BC 1 BP + CP , CD 1 CP + DP , DA 1 DP + AP . A ngy egyenltlensg megfelel oldalait sszeadjuk, majd elosztjuk 2-vel, s a bizonytand lltst kapjuk. b) Tudjuk a hromszg-egyenltlensgek miatt, hogy AC # AP + CP , DB = AC # BP + DP. A kt egyenltlensg megfelel oldalait sszeadjuk, s a bizonytand lltst kapjuk. 6. K1 Egy hromszg kt szgnek arnya 5 : 7, a harmadik szg 72-os. Mekkork a hinyz szgek? A hinyz kt szg sszege 108. Az egyik szg legyen 5x, ekkor a msik 7x. Vagyis 5x + 7x = 108o , amibl x = 9o . A hinyz kt szg: 45, 63. 7. K2 Egy hromszg egyik szge 54, tudjuk tovbb, hogy kt szgnek arnya 4 : 5. Mekkork a hinyz szgek? Hrom eset lehetsges. o III. eset: Ha a 4 : 5 arny kt szg kzl a kisebbik az 54-os, akkor a nagyobb 5 $ 54 = 67,5o . 4 Ekkor a harmadik szg: 180o - 54o - 67,5o = 58,5o . III. eset: Ha a 4 : 5 arny kt szg kzl a nagyobbik az 54-os, akkor a kisebb 4 $ 54 = 43,2o . 5 Ekkor a harmadik szg: 180o - 54o - 43,2o = 82,8o . III. eset: Ha a 4 : 5 arny kt szg kzl egyik sem egyenl 54-kal, akkor a hinyz kt szg sszege 126. Az egyik szg legyen 4x, ekkor a msik 5x. Vagyis 4x + 5x =126o , amibl x =14o . A hinyz kt szg: 56, 70. 8. K1 Egy hromszgben adott egy a bels s egy b kls szg. Mekkork a hinyz bels szgek? a) a = 24, b = 103; b) a = 104, b = 33. a) b = 180o - 103o = 77o , c = 180o - 24o - 77o = 79o ; b) b = 180o - 33o = 147o , c = 180o - 104o - 147o = -71o . Ilyen hromszg nem ltezik.
o

9 .

V F O LYA M

60 MATEMATIKA

I V. B E V E Z E T S A G E O M E T R I B A

4. Tovbbi sszefggsek a hromszg alapadatai kztt


1. K1 Rajzoltunk egy olyan egyenl szr hromszget, amelynek kt 58-os szge van. A szra vagy az alapja a hosszabb? Mivel a hromszgben a szgsszeg 180, ezrt a harmadik szg (a szrak ltal bezrt szg) 180o - 2 $ 58o , azaz 64. Mivel a szrszg a legnagyobb szg a hromszgben, ezrt az alap hosszabb, mint a szr. 2. K1 Egy hromszg kt szge 52-os s 64-os. Egyenl szr-e a hromszg? Mivel a hromszgben a szgsszeg 180, ezrt a harmadik szg 180o - 52o - 64o , azaz 64. Mivel van kt egyenl szge, ezrt egyenl szr a hromszg. 3. K2 Az ABC hromszgben az A-nl lv szg 51-os, a C-nl lv pedig 68-os. Rakjuk nveked sorrendbe az oldalakat! Mivel a hromszgben a szgsszeg 180, ezrt a harmadik szg 180o - 51o - 68o , azaz 61. Mivel nagyobb szggel szemben hosszabb oldal van, ezrt BC 1 CA 1 AB . 4. K2 Az ABC hromszgben AB = 8 cm, AC = 3,5 cm. Melyik llts igaz, melyik hamis? Az ABC hromszg a) C-nl lv bels szge a legnagyobb bels szg. b) B-nl lv bels szge kisebb, mint a C-nl lv. c) A-nl lv kls szge a legnagyobb kls szg. d) Brmelyik bels szge lehet 90-os. e) BC oldala nem a leghosszabb oldal. A harmadik oldal lehetsges hossza: 4,5 cm 1 BC 1 11,5 cm . Vagyis BC nem lehet a legrvidebb oldal, de a leghosszabb igen. Ezt gyelembe vve: a) hamis; b) igaz; c) hamis; d) hamis; e) hamis. 5. K2 Keressk meg a megkezdett mondat helyes befejezst! Ha egy hromszgben az egyik kls szg hegyesszg, akkor a) a hromszg lehet, hogy szablyos. b) a hromszg lehet, hogy derkszg. c) a vele szemkzti oldal a leghosszabb a hromszgben. d) a hromszg nem lehet egyenl szr. Ha a kls szg hegyesszg, akkor a bels tompaszg, vagyis nem lehet a hromszg szablyos. Mivel a hromszg egyik bels szge tompaszg, ezrt a hromszg derkszg sem lehet. (Ekkor a szgsszeg mr tbb lenne, mint 180.) A hromszg tompaszg, s ltezik tompaszg egyenl szr hromszg. Ezek szerint a mondat helyes befejezse nem lehet az a), b), d). A c) pedig valban j, hiszen a tompaszg a legnagyobb szg a hromszgben, ezrt a vele szemkzti oldal a leghosszabb.

9.

V F OLYAM

I V. B E V E Z E T S A G E O M E T R I B A

MATEMATIKA 61

5. sszefggs a derkszg hromszg oldalai kztt


1. K1 Adott egy derkszg hromszg kt oldala. Szmtsuk ki a harmadik oldalnak hosszt! (Az tfogt jelltk c-vel.) a) a = 33, b = 56; b) a = 7,2, b= 15,4; c) b = 80, c = 89; d) a = 16,5, c = 21,9. Hasznljuk a Pitagorasz-ttelt! Az a) s a b) esetben c = a2 + b2 , a c)-nl a = a d)-nl b = c2 - a2 . a) c = 65 ; b) c = 17 ; c) a = 39 ; d) b =14,4 . c2 - b2 ,

2. K1 Ltezik-e olyan hromszg, amelynek oldalhosszai a kvetkezk? Ha igen, akkor derkszg-e? a) a = 65, b = 72, c = 97; b) a = 12, b = 35, c = 41; c) a = 2, b = 3,75, c = 4,25; d) a = 0,425, b = 0,275, c = 0,725. a) Ltezik ilyen hromszg, mert minden oldalra teljesl a hromszg-egyenltlensg. A hromszg derkszg, mivel 652 + 722 = 9409 s 972 = 9409 . b) Ltezik ilyen hromszg, mert minden oldalra teljesl a hromszg-egyenltlensg. A hromszg nem derkszg, mivel 122 + 352 =1369 s 412 = 1681. c) Ltezik ilyen hromszg, mert minden oldalra teljesl a hromszg-egyenltlensg. A hromszg derkszg, mivel 22 + 3,752 = 18,0625 s 4,252 = 18,0625 . d) Nincs ilyen hromszg, mert a + b 1 c . 3. K1 Derkszg-e a PQR hromszg? a) P( 4; 1), Q(3; 3), R(5; 1); b) P(1; 2), Q(1; 5), R(6; 1). a) PQ = 12 + 42 = 17, QR = 82 + 22 = 68 , PR = 92 + 22 = 85 . A hromszg derkszg, mert PQ2 + QR2 = 17 + 68 = 85 s PR2 = 85 . b) PQ = 22 + 32 = 13, QR = 52 + 42 = 41, PR = 72 + 12 = 50 . A hromszg nem derkszg, mert PQ2 + QR2 = 13 + 41 = 54 s PR2 = 50 . 4. K1 Vzszintes talajon ll kt fggleges oszlop tvolsga 8 m. Az egyik oszlop 2 m-rel alacsonyabb a msiknl. Mekkora a kt oszlop tetejnek tvolsga? Jelljk a keresett tvolsgot h-val. Alkalmazzuk a Pitagorasz-ttelt a kt oszlop tvolsgra, a h-ra, s a kt oszlop magassgnak klnbsgre: h = 82 + 22 = 68 . 8,25 . Vagyis a kt oszlop tetejnek tvolsga kb. 8,25 mter. 5. E1 Az ABC derkszg hromszg BC befogjn vegyk fel a D pontot, az AC befogjn pedig az E pontot. Igazoljuk, hogy a) BA2 - BE2 = DA2 - DE2 ; b) BA - BE = DA + DE ! DA - DE BA + BE a) Alkalmazzuk a Pitagorasz-ttelt az EBC s az ABC derkszg hromszgre: BC2 + CE2 = BE2, BC2 + CA2 = BA2 . A msodik egyenlet megfelel oldalaibl vonjuk ki az els egyenlet megfelel oldalait, gy ezt kapjuk: CA2 - CE2 = BA2 - BE2 . (1) Most alkalmazzuk a Pitagorasz-ttelt az EDC s az ADC derkszg hromszgre: DC2 + CE2 = DE2, DC2 + CA2 = DA2 . A msodik egyenlet megfelel oldalaibl megint vonjuk ki az els egyenlet megfelel oldalait: CA2 - CE2 = DA2 - DE2 . (2) Az (1) s a (2) egyenletbl kapjuk a bizonytand lltst: BA2 - BE2 = DA2 - DE2 . b) Ha mindkt oldalt megszorozzuk ]DA - DE g ]BA + BE g-vel, s alkalmazzuk mindkt oldalon az ]a - bg ]a + bg = a2 - b2 azonossgot, akkor az a) feladat lltst kapjuk.
9 . V F O LYA M

62 MATEMATIKA

I V. B E V E Z E T S A G E O M E T R I B A

6. Geometriai szmtsok
1. K1 Adott egy egyenl szr hromszg a alapja s b szra. Szmtsuk ki a hromszg m magassgt! a) a = 4 cm, b = 3 cm; b) a = 17 cm, b = 12 cm. Alkalmazzuk a Pitagorasz-ttelt az a s m befogj, b tfogj derkszg hromszgre: 2 m= b2 - b a l . 2
2

a) m = 32 - 22 = 5 . 2,236 (cm);

b) m = 122 - 8,52 = 71,75 . 8,471 (cm).

2. K1 Az ABCD rombusz oldalainak hossza 6 cm, A-nl lv szge 120. Szmtsuk ki tlinak hosszt! Az ABCD rombuszt a rvidebb tlja kt szablyos hromszgre vgja. Tudjuk, hogy az a oldal szablyos hromszg magassga: m = 3 a . 2

D a a

Mivel most a = 6 cm, ezrt AC = 6 cm, BD = 6 3 cm . 10,392 cm.

60 a a

3. K2 Egy deltoidban a 90-os szggel szemben 60-os szg van. A cscsaikat sszekt tl hossza 10 cm. Mekkora a deltoid kerlete? Az ABC szablyos hromszgben: BE = 3 a. 2

D b A 60 a E 60 B a b C

Az ACD egyenl szr derkszg hromszgben: DE = a . 2 20 . 3+1 Az AED egyenl szr derkszg hromszgben: AD = b = a 2 = 2 Mivel BD = 10 , ezrt 20 10 $ 2 40 + 20 2 . 24,99 . + n= 3+1 3+1 3+1 Vagyis a deltoid kerlete kb. 24,99 cm. 4. K2 Mutassuk meg, hogy az m, a + b s c + m hosszsg szakaszok derkszg hromszget hatroznak meg, ahol a s b egy derkszg hromszg befoginak, c az tfogjnak, m pedig az tfogjhoz tartoz magassgnak a hossza! Mivel m2 + ]a + bg2 = m2 + a2 + 2ab + b2 = c2 + 2cm + m2 = ]c + mg2 , ezrt a Pitagorasz-ttel megfordtsa miatt az m, a + b s c + m hosszsg szakaszok derkszg hromszget hatroznak meg. Az talakts sorn felhasznltuk, hogy a2 + b2 = c2 s 2t = ab = cm . k = 2]a + bg = 2 d 3 a a 10 cm. Ebbl kapjuk: a + = = 2 2

10 2. 3+1

9.

V F OLYAM

I V. B E V E Z E T S A G E O M E T R I B A

MATEMATIKA 63

5. E1 Szmtsuk ki az a = 13 cm, b = 14 cm, c = 15 cm oldalhosszsg hromszg b oldalhoz tartoz magassgnak hosszt! Az bra jellseit hasznlva: m2 + x2 = 132 s m2 + ]14 - x g2 = 152 . A msodik egyenletben elvgezzk a ngyzetre emelst, majd az m2 + x2 helyre 132 -t helyettestnk: 132 + 142 - 28x = 152 .
2 2 2 Ebbl kapjuk, hogy x = 13 + 14 - 15 = 5 s m = 132 - x2 = 132 - 52 = 12. 28 Vagyis a hromszg b oldalhoz tartoz magassgnak hossza 12 cm.

13

15

14 x

6. E2 Hatrozzuk meg azokat a pitagoraszi hromszgeket, amelyeknek a kerlete s terlete azonos mrszm! A feladat szvege szerint: ab = a + b + c , azaz ab - 2a - 2b = 2 a2 + b2 ( a 2 0, b 2 0 , 2 egsz szmok). Ebbl kapjuk: a2 b2 - 4a2 b - 4ab2 + 8ab = 0 , amit ab-vel osztunk s a kvetkez alakban rhatunk: ]a - 4g ]b - 4g = 8 . A lehetsgeket tblzatban rgztettk: a4 b4 a b 8 1 4 3 4 2 0 2 2 4 2 0 1 8 3 4 1 8 5 12 2 4 6 8 4 2 8 6 8 1 12 5

Ktfle derkszg hromszget kaptunk. Az egyiknek a befogi: 6 s 8, ennek tfogja 10, a msiknak a befogi: 5 s 12, ennek tfogja 13. Mindkett pitagoraszi hromszg, gy mindkett megoldsa a feladatnak.

9 .

V F O LYA M

64 MATEMATIKA

I V. B E V E Z E T S A G E O M E T R I B A

7. Geometriai szerkesztsek
1. K1 Szerkessznk adott magassg szablyos hromszget! A vzlatrajz jellseit hasznljuk. Az adott TC magassg felezi a C-nl lv 60-os szget. Az ATC hromszgben ismert a TC oldal s a rajta fekv kt szg, ezrt ez a hromszg megszerkeszthet. Ugyanezt mondhatjuk a BTC hromszgrl is. m

30

2. K2 Szerkessznk egyenl szr hromszget, ha adott az alapja s a) az alaphoz tartoz magassga; b) a szrhoz tartoz magassga! C a) Adatok: c, mc . A vzlatrajz jellseit hasznljuk. Az adott TC magassg felezi az AB oldalt. Az ATC hromszgben ismert a TC s az AT oldal s a kzbezrt 90-os szg, ezrt ez a hromszg megszerkeszthet. Ugyanezt mondhatjuk a BTC hromszgrl is. mc

c 2

P ma

b) Adatok: c, ma. A vzlatrajz jellseit hasznljuk. Az APB derkszg hromszgben ismert az AB tfog s az AP befog. Ezt a hromszget szerkesztjk meg elszr, majd a C pont megszerkesztse kvetkezik. Az AP-re P-ben merleges egyenesbl az A kzppont c sugar kr metszi ki a B pontot. A PB egyenesnek s az AB szakasz felezmerleges egyenesnek metszspontja adja a C pontot.

B 3. K2 Szerkessznk egyenl szr hromszget, ha adott az alaphoz tartoz magassga s az alappal szemkzti szge! Adatok: m, c. A vzlatrajz jellseit hasznljuk. Az adott TC magassg felezi a C-nl lv c szget. Az ATC hromszgben ismert a TC oldal s a rajta fekv kt szg, ezrt ez a hromszg megszerkeszthet. Ugyanezt mondhatjuk a BTC hromszgrl is.

C
2

A
9. V F OLYAM

I V. B E V E Z E T S A G E O M E T R I B A

MATEMATIKA 65

4. K2 Szerkesszk meg az adott kerlet egyenl szr derkszg hromszget! Adat: k.

22,5 C1 A

45

45 B

22,5 C2

A vzlatrajzot gy ksztettk el, hogy AC1 = AC , BC2 = BC legyen, gy C1C2 az adott k kerlettel egyenl. A C1 s C2 cscsoknl 22,5-os szgek vannak. (Az AC1C egyenl szr hromszgben a CC1 alappal szemkzti kls szg 45. Ez a szg egyenl a nem mellette fekv kt bels szg sszegvel, amelyek most egyenlk.) Elszr megszerkesztjk a C1C2 C hromszget. Majd a CC1 szakasz felezmerlegese kimetszi a C1C2 szakaszbl az A pontot, a CC2 szakasz felezmerlegese kimetszi a C1C2 szakaszbl a B pontot. 5. E1 Szerkesszk meg az adott kerlet egyenl szr hromszget, ha adott az alappal szemkzti szge is! Adatok: k, c. c Az elz feladatban lthat mdon jrunk el. Most a C1 s C2 cscsoknl 45o - szgek van4 nak. 6. E1 Egy derkszg hromszgnek ismert az tfogja s a kt befog sszege. Szerkesszk meg a hromszget! Adatok: c, a + b .

c 45 P C A

A vzlatrajzot gy ksztettk el, hogy PC = BC teljesljn, s gy AP szakasz hossza a derkszg hromszg kt befogjnak hosszval legyen egyenl. Ennek felvtelvel kezdjk a szerkesztst. A BCP hromszg egyenl szr derkszg, gy P-nl 45-os szg van. Adott az AB szakasz hossza is. gy a B cscs megszerkeszthet. PA-hoz P-nl szerkesztnk egy 45-os e egyenest, az A krl pedig egy c sugar k krt. Metszspontjuk adja a B pontot. B-bl merlegest lltunk PA-ra, ami kimetszi a keresett C pontot is. Ha az e egyenesnek s a k krnek nincs kzs pontja, akkor nem kapunk megoldst. Ha az e egyenes rinti a k krt, akkor egy megolds lesz. Ha az e egyenes s a k kr metszi egymst, akkor kt megolds van. (A kt megolds alakra nem klnbzik egymstl.)

9 .

V F O LYA M

66 MATEMATIKA

I V. B E V E Z E T S A G E O M E T R I B A

8. Thalsz-ttel
1. K1 Egy AB tmrj flkrven felvettk a P s a Q pontot. Szerkesszk meg az ABC hromszget, ha tudjuk, hogy az A cscsbl indul magassg talppontja P, a B cscsbl indul magassg talppontja pedig Q! Az AQ s a BP egyenes metszspontja adja a C cscsot. (A Thalsz-ttel miatt APBB = 90o s AQBB = 90o .) 2. K2 Az ABC hromszgben az A cscsbl indul magassg talppontja legyen T1, a B cscsbl indul magassg talppontja legyen T2. Legyen tovbb az AB oldal felezpontja F. Igazoljuk, hogy T1FT2 egyenl szr hromszg! Tudjuk, hogy AT1BB = 90o s AT2 BB = 90o , ezrt az AB Thalsz-krre illeszkedik T1 s T2 pont is. Vagyis az AB szakasz F felezpontjtl mindkett sugrnyira tallhat. Ez pontosan a bizonytand llts. 3. K2 Egy krben megrajzoltunk egy AB tmrt s egy AC hrt. Felvettk tovbb a rajzunkon a D pontot gy, hogy AD szakasz felezpontja C. Mutassuk meg, hogy a BD szakasz hossza a kr tmrjvel egyenl! Az ABD hromszgben BC merlegesen felezi az AD oldalt. Vagyis AB = DB , s ezt kellett megmutatni. 4. K2 Egy egyenes t mellett a mezn ll kt fa. Ksztsnk trkpvzlatot, majd szerkesztssel hatrozzuk meg az ttestnek azt a pontjt, ahonnan a kt fa derkszgben ltszik! Vzlatunkon a kt fa F1 s F2 , az t az e egyenes.

F2 F1 e
Az F1F2 Thalsz-krnek minden pontjbl derkszgben ltszik F1F2 . A keresett pontok a Thalsz-kr s az e egyenes kzs pontjai lesznek. Ha a Thalsz-kr metszi az e egyenest, akkor kt pont is megfelel. Ha a Thalsz-kr rinti az e egyenest, akkor egy megfelel pont van. Egybknt nincs megfelel pont. 5. K2 Szerkessznk hromszget egy oldalbl s kt magassgbl! Kt esetet klnbztetnk meg.

T2

m1

T1

m2 c

1. eset: A kt adott magassg egyike sem tartozik az adott oldalhoz. Adatok: c, m1, m2 . A vzlatrajz jellseit hasznljuk. Megszerkesztjk az AB szakasz Thalsz-krt, amire illeszkedik a T1 s a T2 pont. Az A kzppont, m1 sugar s a B kzppont, m2 sugar kr kimetszi a Thalszkrbl a T1, illetve a T2 pontot. Az AT2 s a BT1 egyenesek metszspontjaknt kapjuk a C pontot. A szerkeszthetsg felttele: m1 1 c s m2 1 c .

9.

V F OLYAM

I V. B E V E Z E T S A G E O M E T R I B A

MATEMATIKA 67
C

2. eset: A kt adott magassg egyike az adott oldalhoz tartozik. Adatok: c, m2 , m3. Hasznljuk a vzlatrajz jellseit! Megszerkesztjk az AB szakasz Thalsz-krt, amire illeszkedik a T2 pont. A B kzppont, m2 sugar kr kimetszi a Thalsz-krbl a T2 pontot. Az AB egyenessel m3 tvolsgra hzott prhuzamos egyenes s az AT2 egyenes metszspontja lesz a C pont. A szerkeszthetsg felttele: m2 1 c .

m3 T2 m2 c

A
6. E1 Az e s f egyenesek merlegesek egymsra. Tudjuk, hogy PQ szakasz hossza lland, tovbb P az e egyenesre, Q az f egyenesre illeszkedik. A PQ szakasz sszes lehetsges helyzetben jelljk meg a szakasz F felezpontjt! Milyen ponthalmazt kapunk? Az brn a feladat feltteleinek megfelel helyzetben lv PQ szakaszt ltunk. A PQ Thalsz-krre illeszkedik e s f egyenes C metszspontja. Ezrt a PQ szakasz F felezpontja a C-tl mindig a PQ szakasz hossznak felre tallhat. Ez egy lland, gy a keresett ponthalmaz egy kr, amelynek C a kzppontja, sugara pedig a PQ hossznak felvel egyenl.

T3

e P

9. A hromszg oldalfelez merlegesei s kr rt kre


1. K1 Vegynk fel egy AB szakaszt! Szerkessznk hrom klnbz krt, amelyek mindegyiknek az AB szakasz egy hrja! Az AB szakasz f felezmerleges egyenesn brhol vlaszthatunk egy pontot, az j lesz a kr kzppontjnak.

f B

9 .

V F O LYA M

68 MATEMATIKA

I V. B E V E Z E T S A G E O M E T R I B A

2. K1 Szerkessznk egy hegyesszg, egy derkszg s egy tompaszg hromszget, amelyeknek a kr rt kre ugyanaz az elre adott kr! A hrom bra mutat egy-egy megoldst.

3. K2 Szerkessznk egy 2,5 cm oldalhosszsg szablyos hromszget! Szerkesszk meg a hromszg kr rt krt is! Az ABC szablyos hromszg megszerkesztse utn elg kt oldalfelez merleges egyenest megszerkeszteni, ezek metszspontja megadja a kr rt kr K kzppontjt. Ezutn pldul a KA-t krznylsba vesszk, s megrajzoljuk a kvnt krt.
C

4. K2 Szerkessznk hromszget, ha adott az egyik oldala, a kr rt kr sugara s az adott oldalon fekv egyik szge! Adatok: c, r, a. A vzlatrajz jellseit hasznljuk. Az AB szakasz felvtele utn megszerkesztjk az O pontot. Ekkor az O kzppont r sugar kr s az AB-hez A-ban a szgben hajl egyenes metszspontja lesz a C.
B

r A c

O r

5. K2 Szerkessznk egyenl szr hromszget, ha adott a) az alapja s a kr rt kr sugara; b) a szra s a kr rt kr sugara; c) a szrak ltal bezrt szg s a kr rt kr sugara!

C1

O r A
c 2

r
c 2

a) Adatok: c, r. A vzlatrajz jellseit hasznljuk. Az AB szakasz felvtele utn megszerkesztjk az O pontot. Ekkor az O kzppont r sugar kr s az AB felezmerlegesnek metszspontja lesz a hromszg harmadik cscsa. Mivel kt metszspontot kapunk, ezrt kt megfelel hromszg szerkeszthet: ABC1 s ABC2. A megoldhatsg felttele: c # 2r . ( c = 2r esetn a kapott kt hromszg egybevg.)

C2

9.

V F OLYAM

I V. B E V E Z E T S A G E O M E T R I B A

MATEMATIKA 69
C r b r A O r B b

b) Adatok: b, r. A vzlatrajz jellseit hasznljuk. Az AC szakasz felvtele utn megszerkesztjk az O pontot. Ekkor az O kzppont r sugar kr s a C kzppont, b sugar kr metszspontja lesz a B cscs. (A kt krnek kt metszspontja van, a msik az A.) A megoldhatsg felttele: b 1 2r .

c) Adatok: r, c. A vzlatrajz jellseit hasznljuk. Az AO szakasz felvtele utn megszerkeszthet a C pont. (Mivel ACO hromszg egyenl c szr, ezrt A-nl is szg van, tovbb AOCB = 180o - c .) Ekkor az O kzppont r su2 gar kr s az AC-hez C-nl c szgben hajl egyenes metszspontja lesz a B cscs. A megoldhatsg felttele: 0 1 c 1 180o .

C
2 2

r r A O r B

6. K2 Szerkesszk meg egy adott kr ismeretlen kzppontjt! Mivel minden hrfelez merleges egyenesre illeszkedik a kr kzppontja, ezrt kt tetszleges (de nem prhuzamos) hr felezmerlegesnek metszspontja lesz a kr kzppontja. 7. K1 Az ABCDE szablyos tszg cscsai kzl minden lehetsges mdon kivlasztunk hrmat. Az gy kapott hromszgek mindegyiknek megszerkesztjk a kr rt krt. Hny klnbz hromszget s hny klnbz krt kapunk? A szablyos tszg kr rt kr minden ilyen hromszgnek a kr rt kre lesz. Vagyis egy krt kapunk. A hromszgeket felsorols utn meg tudjuk szmolni: ABC, ABD, ABE, ACD, ACE, ADE, BCD, BCE, BDE, CDE. Vagyis 10 klnbz hromszget kapunk. 8. E1 Adott egy ngyszg. Szerkessznk olyan krket, amelyek a ngy cscstl egyenl tvolsgban haladnak! Legyen a ngy pont: A, B, C, D. Megszerkesztjk pldul az ABC hromszg kr rt krt. A kapott kr kzppontja K, sugara r. A K kzppont, r + KD sugar kr megfelel a feladat feltteleinek, ugyanis ez 2 mind a ngy ponttl r - KD tvolsgra halad. 2 Ha kivlasztott hrom pont kr rt krn a negyedik pont is rajta van, akkor brmely K kzppont kr megfelel lesz. Mivel ngy pont kzl hrmat kivlasztani ngyflekppen lehet, ezrt ltalnos helyzetben ngy megoldsa van a feladatnak. Ez lthat az brn.

A B

9 .

V F O LYA M

70 MATEMATIKA

I V. B E V E Z E T S A G E O M E T R I B A

9. E2 Apollniosz-fle feladatnak nevezzk a kvetkezt: Szerkessznk hrom adott krhz egy negyediket, amely mindhrom adott krt rinti! Azokat a feladatokat is gy nevezzk, amikor a hrom adott kr brmelyike helyett pont vagy egyenes van adva. Ezek a pontok, egyenesek, krk lesznek az gynevezett kiindul elemek. Tervezznk meg egy vletlenszer sorsolst, amelynek segtsgvel egy Apollniosz-fle feladat kiindul elemeit kivlaszthatjuk! Hny kimenetele lehet a sorsolsnak? Tegynk egy kartondobozba pldul hrom piros, hrom kk s hrom rzsaszn kupakot (de brmi lehet, ami alakra egyforma, csak sznben klnbz). Kevers utn vegynk ki hrmat. A piros (P) jelentsen pontot, a kk (K) krt, a rzsaszn (R) pedig egyenest. gy valban egy Apollniusz-feladatot sorsoltunk ki. Lehet hogy mindhrom kihzott kupak azonos szn: PPP, KKK, RRR. Lehet, hogy kett azonos szn: PPK, PPR, KKP, KKR, RRP, RRK. Lehet, hogy mind klnbz szn: PKR. Vagyis 10 eset lehetsges.

10. A hromszg szgfelezi, bert s hozzrt krei


1. K1 Egy hromszgben egy-egy szg 64-os s 48-os. Mekkora szget zr be egymssal a kt szg bels szgfelezje? A kt szg cscsa s a szgfelezk metszspontja egy hromszget hatroz meg. Ennek a hromszgnek kt szge: 32 s 24. Ezrt a harmadik szg: 124. Ezek alapjn a kt szgfelez hajlsszge a 124 kiegszt szge lesz, vagyis: 56. 2. K2 Mekkora szget zr be egymssal a hromszg kt szgfelezje, ha tudjuk, hogy a harmadik szg 76-os? A kt szg cscsa s a szgfelezk metszspontja egy hromszget hatroz meg. Ebben a hromszgben kt szg sszege: ^180o - 76oh : 2, azaz 52. Ezrt a harmadik szg: 128. Ezek alapjn a kt szgfelez hajlsszge a 128 kiegszt szge lesz, vagyis: 52. 3. K2 Mekkora szget zr be egymssal a hromszg valamely cscsbl indul bels s kls szgfelez? Mivel az egy cscsnl fekv bels s kls szgfelez sszege 180, ezrt az egy cscsbl indul bels s kls szgfelez 90-os szget zr be egymssal. 4. K2 Mekkora a bert s a kr rt kr sugara az a) a = 6 cm, b = 8 cm; b) a = 15 cm, b = 36 cm befogj derkszg hromszgben? a) Mivel derkszg a hromszg, ezrt a kr rt kr a Thalsz-kr lesz. Vagyis ennek a krnek az r sugara az tmr felvel egyenl hosszsg. Pitagorasz-ttellel: c = 10 cm, azaz r = 5 cm. A derkszg hromszg terlete: t = ab = 6 $ 8 = 24 (cm2), kerlete: 2 2 k = 6 + 8 + 10 = 24 (cm), ezrt a kerletnek a fele: s = 12 cm. Hasznljuk a t = ts terletkpletet (ahol t a hromszg bert krnek sugara): t = t = 24 = 2 (cm). s 12 b) Az elz gondolatmenetet kvetve: r = 19,5 cm, t = 6 cm.

9.

V F OLYAM

I V. B E V E Z E T S A G E O M E T R I B A

MATEMATIKA 71

5. E1 Igazoljuk, hogy CaCb = a + b, ahol a s b a hromszg oldalai a szoksos jells szerint, Ca a c oldalegyenes s az a oldalhoz hozzrt kr rintsi pontja, Cb a c oldalegyenes s a b oldalhoz hozzrt kr rintsi pontja! Hasznljuk az bra jellseit!

Oa C Ob Bb Cb A Aa B Ca

Tudjuk, hogy ACa = s, BCb = s, AB = c . Vagyis Ca Cb = 2s - c = a + b . 6. E2 Igazoljuk, hogy CcCa = b, ahol a a hromszg oldala a szoksos jells szerint, Cc a c oldal s a c oldalhoz hozzrt kr rintsi pontja, Ca a c oldalegyenes s az a oldalhoz hozzrt kr rintsi pontja! Hasznljuk az bra jellseit!

Oa C

Aa Cc Cb A

Ca

Oc

Mivel ACa = s, AB = c , ezrt BCa = s - c . Mivel CAc = s, BC = a , ezrt BAc = s - a . Mivel BCc = BAc , ezrt BCc = s - a . Ezek alapjn Cc Ca = BCa + BCc = s - c + s - a = b . Megjegyzsek: 1. Az elz feladatban lttuk, hogy Ca Cb = a + b . Most bizonytottuk, hogy Cc Ca = b . Ezekbl kvetkezik, hogy Cb Cc = a . 2. Megmutathat, hogy AB oldal felezmerlegesre BCc szakasz tkrs, s a tkrkpe az ACo szakasz, vagyis ACo = s - a . Mivel BCb = s, AB = c , ezrt ACb = s - c . Ezek alapjn Cb C0 = s - c + s - a = b . 3. Az eddigi eredmnyeinket felhasznlva kiszmthatjuk a Co Cc szakasz hosszt: Co Cc = AB - 2 $ BCc = c - 2]s - ag = c - a - b - c + 2a = a - b . Felhasznltuk a rajzunkrl, hogy a 2 b . Ha ezt nem tudjuk, akkor Co Cc = a - b .
9 . V F O LYA M

72 MATEMATIKA

I V. B E V E Z E T S A G E O M E T R I B A

11. Sokszgek
1. K1 Szmtsuk ki a a) 11; b) 23; c) 108; d) 1000 oldal konvex sokszg bels szgeinek sszegt! Tudjuk, hogy az n oldal konvex sokszg bels szgeinek sszege ]n - 2g $ 180c . a) ]11 - 2g $ 180o = 1620o ; b) ]23 - 2g $ 180o = 3780o ; c) ]108 - 2g $ 180o = 19 080o ; d) ]1000 - 2g $ 180o = 179 640o .

2. K1 Szmtsuk ki a a) 13; b) 21; c) 132; d) 500 oldal konvex sokszg tlinak szmt!

] g Tudjuk, hogy az n oldal konvex sokszgben n n - 3 tl van. 2 ] g ] g a) 13 13 - 3 = 65 ; b) 21 21 - 3 =189 ; 2 2 ] g ] g c) 132 132 - 3 = 8514 ; d) 500 500 - 3 = 124 250 . 2 2
3. K2 Igazoljuk, hogy az brn lthat konkv sokszgek bels szgeinek sszege (n 2) 180, ahol n az oldalak szmt jelenti! a) b)

a)

b)

a) Az brn lthat mdon az tszget hrom hromszgre vgjuk. Ezrt a bels szgek sszege valban ]5 - 2g $ 180o = 540o . b) Az brn lthat mdon a htszget t hromszgre vgjuk. Ezrt a bels szgek sszege valban ]7 - 2g $ 180o = 900o . 4. K2 Hny oldal lehet az a sokszg, melyben minden szg hegyesszg? Tudjuk, hogy a konvex sokszgek bels szgeinek sszege ]n - 2g $ 180o . Ha egy n oldal sokszgben minden szg kisebb, mint 90, akkor ]n - 2g $ 180o 1 n $ 90o . Azaz n 1 4 . Vagyis csak hromszgek esetn kpzelhet el az, hogy egy sokszg minden szge hegyesszg.

9.

V F OLYAM

I V. B E V E Z E T S A G E O M E T R I B A

MATEMATIKA 73

5. K2 Egy konvex sokszg hrom cscst az brn lthat mdon egy-egy egyenes mentn levgtuk. Hogyan vltozik ekkor a bels szgek sszege? Ha eredetileg a sokszgnek n oldala volt, akkor a vgs utn n + 3 oldala lett. Azaz a bels szgek sszege ]n - 2g $ 180o -rl ]n + 1g $ 180o -ra ntt. Vagyis 540-kal ntt a bels szgek sszege.

6. E2 a) Egy n oldal sokszg oldalainak szmt megduplzzuk. Az oldalszm f(n) fggvnyeknt fejezzk ki, hogy hnyszorosra vltozik a sokszg bels szgeinek sszege! b) Mely egsz szmokat veszi fel az a) rszben szerepl f(n) fggvny? c) Adjuk meg azt a legszkebb intervallumot, amelyben az a) rszben szerepl f(n) fggvny minden fggvnyrtke megtallhat! a) Az eredeti sokszg bels szgeinek sszege: ]n - 2g $ 180o , az j sokszgben a szgsszeg: o 2n - 2 -szeresre n a sokszg szgsszege ]2n - 2g $ 180o . Vagyis f ]ng = ]2n - 2g $ 180 o = n-2 ]n - 2g $ 180 ( n $ 3 egsz szm). o ] g 2n - 2 2]n - 2g + 2 2 2 b) Mivel f ]ng = 2n - 2 $ 180 s n $ 3 egsz szm, = = = + n-2 n-2 n-2 ]n - 2g $ 180o ezrt kt egsz szmot vehet fel az f ]ng. Ezek a kvetkezk: f ]3g = 4, f ]4g = 3. c) Tudjuk, hogy f ]ng = 2 + 2 s n $ 3 egsz szm. Az n nvekedsvel a 2 cskken, n-2 n-2 ] g @ @ de pozitv marad. Ezeket gyelembe vve: f n ! 2; 4 . Megjegyzs: Minden megengedett n esetn f ]ng ! @2; 4 @ , de ha y ! @2; 4 @ , akkor nem felttlenl ltezik y-hoz megfelel n szm. 7. E1 Egy sokszgnek szeretnnk megduplzni a bels szgeinek sszegt. Hogyan kell vltoztatni az oldalak szmt? Az eredeti sokszg bels szgeinek sszege: ]n - 2g $ 180o , az j sokszgben a szgsszeg: ]m - 2g $ 180o . A feladat felttele szerint: 2]n - 2g $ 180o = ]m - 2g $ 180o , amibl m = 2n - 2. Vagyis az eredeti sokszg oldalszmnak ktszeresnl kettvel kevesebb oldala legyen az j sokszgnek. 8. E1 Adjuk meg az sszes olyan egsz szmot, amely lehet egy szablyos sokszg bels szgnek fokban kifejezett mrszma! A tanknyvi pldban mr lttuk, hogy 22-fle olyan szablyos sokszg van, amelyben a bels szgek fokokban mrt mrszma egsz szm. Megllaptottuk, hogy az oldalak szma a kvetkez 22 szm lehet: 3, 4, 5, 6, 8, 9, 10, 12, 15, 18, 20, 24, 30, 36, 40, 45, 60, 72, 90, 120, 180, 360. o Az n oldalszm ismeretben a 180o - 360 kplettel kiszmtjuk a megfelel szablyos sokszg n egy szgnek mrszmt. A kvetkez szmokat kapjuk: 60, 90, 108, 120, 135, 140, 144, 150, 156, 160, 162, 165, 168, 170, 171, 172, 174, 175, 176, 177, 178, 179.

9 .

V F O LYA M

74 MATEMATIKA

I V. B E V E Z E T S A G E O M E T R I B A

9. E2 Melyek azok a szablyos sokszgprok, amelyek esetn az egyik sokszg bels szge a msik sokszg kls szgvel egyenl?
o ] g Az n oldal szablyos sokszg minden szgnek nagysga n - 2 $ 180 . Tudjuk, hogy a kls n szgek sszege minden konvex sokszgben 360, ezrt az m oldal szablyos sokszg minden o o o ] g kls szgnek nagysga 360 . A feladat szvege szerint: n - 2 $ 180 = 360 , azaz trenm n m dezve: mn - 2n - 2m = 0 . Ezt ]n - 2g ]m - 2g = 4 alakban is felrhatjuk. Mivel n s m 2-nl nagyobb egsz, ezrt csak a ^6; 3h, ^4; 4h, ^3; 6h szmprok adjk a megoldst. Vagyis a szablyos hromszgszablyos hatszg s a ngyzetngyzet prosts tesz eleget a feladat feltteleinek.

9.

V F OLYAM

V. E G Y E N L E T E K , E G Y E N L E T R E N D S Z E R E K

MATEMATIKA 75

V. Egyenletek, egyenletrendszerek
1. Elsfok egyismeretlenes egyenletek
Oldjuk meg a kvetkez egyenleteket a vals szmok halmazn! 1. K1 1 $ ;2x - 2 b x + 1 lE = 4x - 3] x + 2g. 2 3 4 x - 1 x - 1 = x - 6 , azaz 12x - 4x -1 = 12x - 72, ahonnan x = 71. 3 12 4 2. K1 3x + 2 + 2x + 5 = x - 5 - x . 4 6 3 9x + 6 + 4x + 10 = 12x - 20 + 4x , ahonnan x = 12. 3. K1 2x - 3 = 2] x - 3g . 2+x 6+x

]2x - 3g ] x + 6g = ]2x - 6g ] x + 2g, azaz 2x2 + 9x - 18 = 2x2 - 2x - 12, ahonnan x = 6 . 11


4. K1 ] x + 6g ] x - 5g - ] x - 3g ]2 - x g = ]2x - 1g ] x + 2g. x2 + x - 30 - 2x + 6 + x2 - 3x = 2x2 + 3x - 2, ahonnan x = - 22 . 7 5. K2 x - 5 - x + 5 + 220x = 0 . x + 5 x - 5 x - 25 x2 -10x + 25 - x2 -10x - 25 + 20x 0 , azaz 0 = = 0 . Az egyenlet megoldsa x ! 5 x2 - 25 x2 - 25 mellett minden vals szm. 6. K1 6x + 12 = 8x - 24 . 7. K1 2 x - 1 = 5 x + 1 . 3 4 4 3 8. K1 6] x - 3g - 3] x - 4g = 2] x + 6g. 9. K1 x - 3 - 2x - 4 = x + 1 - x . 3 5 10 10. K1 4x - x + 2 - 3x - 2 = 2 - 4x - 1. 3 2 3 11. K1 x - 62]3x - 2g + 2x + 3@ = 4] x + 6g. 12. K2 5x - 3 "4x - 2 6 4x - 3]5x - 2g@, = 182. x =18 . x = -1. x =18 . x= 9 . 25 x = 4. 7 x = - 23 . 11

x = -2.

9 .

V F O LYA M

76 MATEMATIKA

V. E G Y E N L E T E K , E G Y E N L E T R E N D S Z E R E K

2. Szveges feladatok megoldsa egyenletekkel


1. K1 Egy hromjegy szmban a 10-esek helyn ll szmjegy kettvel nagyobb az egyesek helyn llnl, s eggyel kisebb a szzasok helyn ll szmjegynl. Ha a szmhoz hozzadjuk a fordtottjt, azaz a szmjegyeinek fordtott sorrendjben felrt szmot, eredmnyl 1453-at kapunk. Mi volt az eredeti hromjegy szm? abc + cba = 101a + 20b + 101c = 1453, ahol b = c + 2 s a = c + 3. Ezek szerint 101]c + 3g + 20]c + 2g + 101c = 1453, azaz 222c + 343 = 1453, ahonnan c = 5 , s gy b = 7, a = 8 . A keresett szm a 875. km 2. K2 Budapestrl elindul egy aut Mnchenbe reggel 8 rakor lland 80 h sebessggel. km Fl 10-kor elindul egy msik aut ugyanazon az tvonalon lland 110 h sebessggel. Mikor lesz a kt aut kztt a tvolsg 60 km? A kt aut kztt a tvolsg ktszer lesz 60 km. Elszr akkor, amikor a msodik aut mg az els mgtt van, msodszor pedig amikor a msodik aut mr az elst megelzve eltte van. Ha t az els aut menetideje, akkor az albbi egyenleteket rhatjuk fel: 80t =110^t -1,5h + 60 , illetve 80t =110^t -1,5h - 60 . Els esetben t = 3,5 , a msodik esetben t = 7,5 . Teht a kt aut kztt a tvolsg 11 ra 30kor, illetve 15 ra 30-kor lesz 60 km. 3. K2 Egy nagy eszs utn vzzel elrasztott pincbl hrom szivattyval akarjk a vizet kiemelni. Az els s a msodik szivatty egyedl 6 ra alatt tudn kiszivattyzni a vizet a pincbl. A hrom szivattyt dleltt 11-kor kapcsoltk be, s dlutn 13 ra 15 percre kiszivattyztk az sszes vizet. Mennyi id alatt vgezne egyedl a szivattyzssal a harmadik szivatty? Ha a harmadik szivatty egyedl x ra alatt szvja ki a vizet a pincbl, akkor 2,25 2,25 2,25 + + = 1, azaz 2,25x + 2,25x +13,5 = 6x . 6 6 x Innen x = 9 . Teht a 3. szivatty egyedl 9 ra alatt vgezne a munkval. 4. K2 Egy kzpkori feladat: Lszen egy hromszeglemny, melliknek is kt gyeplnii azonos mrtkek vala. Emezekkel szemkesztes kenyeki kt tag naturalis numerandusok vala. Mgnem az harmadik kenyek emezen numerandusok fordtottja vala. Mekkork az fentebb forgand triangulum kenyeki? Vagyis: Egy egyenl szr hromszg alapjn fekv szgei fokokban mrve ktjegy egsz szmok. A szrszge e ktjegy szmok fordtottja (vagyis a szmjegyek felcserlsvel kapott ktjegy szm). Mekkork a hromszg szgei? A felttelek szerint 2 $ ab + ba =180 . Rszletesen kirva 21a +12b =180 , azaz 7a + 4b = 60 . Mivel a jobb oldal 4-gyel oszthat, ezrt a bal oldalnak is 4-gyel oszthatnak kell lennie, azaz 7a oszthat 4-gyel, vagyis a oszthat 4-gyel. Teht a = 4 , vagy a = 8 . Ha a = 4 , akkor b = 8 , ha pedig a = 8 , akkor b =1. A hromszg szgei: 48, 48, 84 vagy 81, 81, 18. 5. K2 Andrs s Bla egytt 36 vesek. Amikor Andrs annyi ids lesz, mint Bla most, akkor Bla veinek szma 16-tal lesz tbb, mint Andrs veinek mostani szma. Hny vesek most? Ha Andrs most a ves, akkor Bla 36 - a ves. Amg Andrs annyi ids lesz, mint Bla most, azaz 36 - a ves, addig 36 - a - a = 36 - 2a v telik el. Ennyi v mlva Bla 36 - a + 36 - 2a = 72 - 3a ves lesz. A felttelek szerint ahonnan 72 - 3a -16 = a , a =14 . Teht Andrs jelenleg 14 ves, Bla pedig 22 ves.
9. V F OLYAM

V. E G Y E N L E T E K , E G Y E N L E T R E N D S Z E R E K

MATEMATIKA 77

6. K1 1000 Ft-ot felvltottunk 20 s 50 Ft-os rmkre. sszesen 32 db rmt kaptunk. Hny db 20 s hny db 50 Ft-os rmnk lett? Ha x db 20 Ft-os van, akkor a megoldand egyenlet: 20x + 50]32 - x g =1000 . x = 20 , teht 20 db 20 Ft-os s 12 db 50 Ft-os rmnk van. 7. K1 Hrom testvr kzl a kzps 11 ves, a legidsebb tszr annyi ids, mint a legatalabb. A hrom testvr egyttes letkora eggyel kevesebb, mint amennyi ids lesz a legidsebb akkor, amikor ktszer annyi ids lesz, mint most. Hny vesek a testvrek? Legyen a legatalabb x ves. Ekkor x +11 + 5x +1 =10x , ahonnan x = 3. A testvrek letkora: 3, 11 s 15 v. 8. K2 Az A s B helysgek kztt a tvolsg 240 km. A-bl B-be elindul reggel 6 rakor egy km km vonat 64 h sebessggel. 7-kor B-bl indul egy vonat A-ba 80 h sebessggel. Mikor lesz a tvolsg kzttk 40 km? Ktszer lesz a kt vonat kztt a tvolsg 40 km. Ha t az A helysgbl indul vonat menetideje, akkor egyms fel kzeledve: 64t + 80]t -1g + 40 = 240 , ahonnan t =1,94 ra. Egymstl tvolodva: 64t + 80]t -1g - 40 = 240 , ahonnan t = 2,5 ra. A kt vonat kb. 7 ra 57 perckor, illetve 8 ra 30 perckor lesz egymstl 40 km tvolsgra. 9. E1 Csaba reggel 9 rakor elindult kerkprral, hogy a vrosban lak nagymamjt megkm ltogassa. lland 20 h sebessggel haladt, s msfl rt tlttt a nagyinl. Visszafel ugyancsak lland sebessggel haladva mr sietett haza, sebessgt 10%-kal nvelve
3 3 eltt 3 perccel rt haza. Milyen tvol lakik a nagymama? 4

Csaba sszesen 5,7 rt tlttt tvol. Ebbl 1,5 rt volt nagymamjnl, 4,2 rt volt ton. Ha a krdses tvolsg S, akkor S + S = 4,2, ahonnan S = 924 = 44 km. 20 22 21 10. E1 Egy kocsmros a 12 liter 40%-os plinkjt hogy nagyobb nyeresgre tegyen szert hgtani akarta. Ismert, hogy 10%-nl nagyobb tmnysg esetn 2%-os eltrst mszer nlkl mg nem lehet szrevenni. Ezt tudja a kocsmros is. Legfeljebb hny dl vzzel hgthatja plinkjt, hogy mg ne vegyk szre a csalst a fogyasztk? A plinka tisztaszesz-tartalma 4,8 liter. A hgts utn legalbb 38%-osnak kell lennie. 4, 8 $ 100 $ 38 , ahonnan x # 0,6316 l. 12 + x Teht legfeljebb 6,3 dl vzzel hgthatja plinkjt a kocsmros.

9 .

V F O LYA M

78 MATEMATIKA

V. E G Y E N L E T E K , E G Y E N L E T R E N D S Z E R E K

3. Egyenletek megoldsi mdszerei


1. K2 Oldjuk meg a kvetkez egyenleteket a vals szmok halmazn! a) ]2x +1g ] x - 4g + 3 ]2x +1g = 6]2x + 1g; b) ]4 - x g -12 + 3x = 2x - 8 . a) Rendezzk 0-ra az egyenletet, majd emeljnk ki ]2x +1g-et. ]2x +1g ] x - 4g + 3]2x +1g - 6]2x +1g = 0 , azaz ]2x +1g ] x - 7g = 0 . Innen x1 = - 1 , x2 = 7 . 2 b) Az egyenlet gy alakthat: ]4 - x g - 3]4 - x g + 2]4 - x g = 0 , azaz ]4 - x g ]3 - 3g = 0 . Azonossghoz jutottunk, vagyis az egyenletet minden vals szm kielgti. 2. K2 Oldjuk meg grakusan az albbi egyenleteket! a) 2x - 2 = - 1 x + 3; b) x - 3 = 1 x ; c) x - 4 = - x + 2. 2 2 2

a)
1 2x + 3

b)
|x 3|

1 1 x

1 1 x

1 2x

2x 2

Az bra alapjn az egyenlet megoldsa x = 2. c)


y

Az bra alapjn az egyenlet megoldsa x1 = 2, x2 = 6 .

x + 2

1 1
1 2x

Az egyenlet megoldsa az bra alapjn x = 8 , amit helyettestssel ellenrizhetnk. 3

3. E1 Mely x, y vals szmok elgtik ki az albbi egyenlsgeket? a) x2 - 6x + 9 + y2 - 2y +1 = 0 ; 2x - 4 + 6 - 3x + 2xy = x2 - 4y + 3; 2y2 + 12y + 19 c) 2 1 . = 2 x - 4x + 6 b)


9. V F OLYAM

V. E G Y E N L E T E K , E G Y E N L E T R E N D S Z E R E K

MATEMATIKA 79

a) Vegyk szre, hogy az egyenlet bal oldaln teljes ngyzetek szerepelnek. ] x - 3g2 + ^ y -1h2 = 0 . Ez akkor s csak akkor teljesl, ha a bal oldal mindkt tagja 0, vagyis x = 3, y =1. b) A ngyzetgykk miatt 2x - 4 $ 0 , azaz x $ 2 s 6 - 3x $ 0 , azaz x # 2. Ezek szerint csak x = 2 lehetsges. Ezzel az egyenlet gy alakul: 4y = y - 4y + 3, ahonnan y = 3 . 7 1 c) Az egyenlet bal oldala: . Mivel a nevez rtke legalbb 2, gy a trt rtke leg] x - 2g2 + 2 feljebb 1 . Az egyenlet jobb oldala: 2 2 y + 6y + 19 = ^ y + 3h2 - 9 + 19 = ^ y + 3h2 + 1 $ 1 . 2 2 2 2 1 Mivel a bal oldal legfeljebb , a jobb oldal legalbb 1 , gy az egyenlsg csak akkor llhat 2 2 1 fenn, ha mindkt oldal rtke . Ekkor x = 2, y = -3. 2 4. E1 Oldjuk meg grakusan az albbi egyenleteket! a) 2 - x + 2 = 1 x ; b) x - 3 + x + 2 = x + 3; 5

c)

x -1 = 3 - x .

a)

b)

1 1 x

1 1 x

x1 = -5, x2 = 0 . c)
y

x1 = 2, x2 = 4 .

1 1 x

x = ! 2.

9 .

V F O LYA M

80 MATEMATIKA

V. E G Y E N L E T E K , E G Y E N L E T R E N D S Z E R E K

4. Egyenltlensgek
1. K1 Oldjuk meg az albbi egyenltlensgeket a vals szmok halmazn! a) 4x + 2 # x + 4 ; b) x +1 - x -1 # 2x - 2; c) 3 6 x - 2] x + 3g@ 2 2 6 x - 3] x - 3g@ . 3 2 3 a) x # 10 ; b) x $ 17 ; 11 c) x 2 36 .

2. K2 Oldjuk meg a kvetkez egyenltlensgeket! a) 2x + 3 # 0 ; b) x + 3 2 1; c) ] x + 3g ]4x - 6g 2 0 . x-4 2x - 4 a) Egy trt rtke akkor negatv, ha a szmll s a nevez klnbz eljel. vagy 2x + 3 $ 0 s x - 4 1 0 2x + 3 # 0 s x - 4 2 0 . Els esetben x $ - 3 s x 1 4 . A msodik esetben pedig x # - 3 s x 2 4 , ami nyilvn le2 2 hetetlen. Teht az eredeti egyenltlensget kielgt vals szmok: - 3 # x 1 4 .

b) Vegynk el mindkt oldalbl 1-et, majd vonjuk ssze a kapott kt tagot. x + 3 1 2 0, x+3 2x 4 -x + 7 2 0. teht ahonnan - - 2 0, 2x - 4 2x - 4 2x - 4 2x - 4 Ez utbbi egyenltlensg akkor teljesl, ha a szmll s a nevez azonos eljelek. Az eredeti egyenltlensget kielgt vals szmok: 2 1 x 1 7 . c) Egy kttnyezs szorzat akkor s csak akkor pozitv, ha tnyezi azonos eljelek. Az eredeti egyenltlensget kielgt vals szmok: x 1 -3 vagy x 2 3 . 2 3. K1 brzoljuk a szmegyenesen azokat az x vals szmokat, melyek mindkt egyenltlensget kielgtik! 2x + 4 # x 2, x - 2 1 x +1. + 3 3 Az els egyenltlensg megoldsa: -2 # x , a msodik megoldsa: x 1 7 . Teht a mindkt 2 egyenltlensget kielgt vals szmok a szmegyenesen:

x 2 0 3,5

4. E1 Hatrozzuk meg a p > 0 paramter rtkt gy, hogy az albbi egyenlet megoldsa pozitv legyen! x+p px + 3 . -x = 2 3 Elszr oldjuk meg az egyenletet. Mindkt oldalt 6-tal beszorozva, majd rendezve, azt kapjuk: azaz 3x + 3p - 6x = 2px + 6 , x ^2p + 3h = 3p - 6 . 3p - 6 Mivel p 2 0 , ezrt 2p + 3 ! 0 , gy az egyenlet megoldsa: x = . Meg kell oldanunk a 2p + 3 3p - 6 2 0 egyenltlensget. A nevez pozitv, gy a trt rtke akkor s csak akkor pozitv, ha 2p + 3 a szmll is pozitv, vagyis 3p - 6 2 0 , ahonnan p 2 2. 5. K1 Oldjuk meg a kvetkez egyenltlensgeket! a) x + 3 + x +1 # x - 2 ; b) 2] x -1g + 3]2x + 6g 2 8] x - 3g. 4 2 a) 13 # x ;
9. V F OLYAM

b) Az egyenltlensget minden vals szm kielgti.

V. E G Y E N L E T E K , E G Y E N L E T R E N D S Z E R E K

MATEMATIKA 81

6. K2 brzoljuk a szmegyenesen azokat az x vals szmokat, amelyek egyszerre mindkt egyenltlensget kielgtik! x 1 1 2x ; a) x - 3 + 2x - 2 2 6] x + 2g, 5 2 4 2x - 4 2x 1 4 x . b) x] x -1g - x] x + 3g # 2x + 3, + 3 a) Nincs olyan vals szm, mely mindkt egyenltlensgnek megfelel.

x 17 6
b) - 1 # x 1 16 . 2 11

4 7

x 0 1 2
16 11

7. E1 Mekkora legyen az a paramter rtke, hogy az albbi egyenlet megoldsa legalbb 1 legyen? ax + 2 a x 2x - a 2. + + = + 3 4 Szorozzuk meg az egyenlet mindkt oldalt 12-vel! 4ax + 8 + 12a + 12x = 6x - 3a + 24 , azaz 4ax + 6x = 16 - 15a , x]4a + 6g = 16 - 15a . 3 Ha a = - , akkor visszahelyettestve ezt az eredeti egyenletbe ellentmondshoz jutunk, 2 teht ez esetben nincs megolds. Ha a ! - 3 , akkor az egyenlet megoldsa x = 16 - 15a . 2 2]2a + 3g Ez (a 2. b) feladat megoldsban kzlt gondolatmenetet kvetve) akkor s csak akkor lesz legalbb 1, ha - 3 1 a # 10 . 2 19 8. E2 Mekkora legyen a p paramter rtke, hogy a kvetkez egyenlet megoldsa pozitv egsz szm legyen? 2x + p px -1 . -1 = 2 3 A kzs nevezvel szorozva azt kapjuk: azaz 6x + 3p - 6 = 2px - 2, 2x ^ p - 3h = 3p - 4 . Ha p = 3 , akkor az egyenletnek nincs megoldsa. Ha p ! 3 , akkor az egyenlet megoldsa 3p - 4 . x= 2p - 6 Vgezzk el a kvetkez talaktst: 3p - 4 2p - 6 p+2 p+2 . + = = 1+ 2p - 6 2p - 6 2p - 6 2p - 6 p+2 Ez akkor s csak akkor lesz pozitv egsz, ha nemnegatv egsz. Ekkor egyrszt p 2 3 2p - 6 kell hogy legyen, msrszt p + 2 $ 2p - 6 , ahonnan p # 8 . Mivel a nevez pros, gy a szmllnak is prosnak kell lennie, teht p = 8 , 6 vagy 4 lehet csak. De p = 6 esetn a trt rtke nem egsz, mg p = 8 s p = 4 esetn igen, gy az eredeti egyenlet megoldsa akkor s csak akkor lesz pozitv egsz, ha p = 4 vagy p = 8 .

9 .

V F O LYA M

82 MATEMATIKA

V. E G Y E N L E T E K , E G Y E N L E T R E N D S Z E R E K

5. Abszolt rtket tartalmaz egyenletek, egyenltlensgek


1. K1 Oldjuk meg a kvetkez egyenleteket a vals szmok halmazn! a) 2x + 6 = 4 ; b) 3 - 2x = 2 ; c) x +1 = 2x -1. a) x1 = -1, x2 = -5 ; b) x1 = 1, x2 = 5 ; 2 2 c) x = 2.

2. K1 Oldjuk meg az albbi egyenletet, egyenltlensgeket a vals szmok halmazn! a) 2x - 3 =10 ; b) 6x + 3 1 1; c) 5x -1 $ 4 . a) x1 = 13, x2 = - 7 ; b) - 2 1 x 1 - 1 ; 2 2 3 3 c) x # - 3 5 vagy 1 # x .

3. K2 Oldjuk meg az albbi egyenleteket a vals szmok halmazn! a) x + 3 + 2x - 4 = 2] x - 2g ; b) x -1 = 2 ; c) x - 2 + 4 = 6 . a) Ha x $ -3, akkor az eredeti egyenlet gy rhat: ahonnan x + 3 + 2x - 4 = 2x - 4 , x = -3. Ha x 1 -3, akkor az eredeti egyenlet - x - 3 + 2x - 4 = 2x - 4 , ahonnan x = -3. Mivel esetnkben x 1 -3, gy nem kapunk j megoldst. Teht az eredeti egyenlet egyedli megoldsa: x = -3. b) Az egyenletbl x -1 = ! 2, azaz x = 3 vagy x = -1. Ez utbbi nyilvn lehetetlen, gy az eredeti egyenlet megoldsa: x = ! 3. c) Az egyenletbl x - 2 + 4 = ! 6 , ahonnan x - 2 = 2 vagy x - 2 = -10 . Ez utbbi nem lehetsges, gy x - 2 = 2, azaz x - 2 = ! 2, ahonnan az egyenlet megoldsa: x1 = 0, x2 = 4 . 4. E1 Mely vals szmok elgtik ki a kvetkez egyenltlensgeket? a) x - 4 + x $ 4 ; b) x - 2 - x 1 1. a) brzoljuk grafikusan az egyenltlensg bal oldaln szerepl kifejezst! Ha x $ 4 , akkor x - 4 + x = x - 4 + x = 2x - 4 . Ha 0 # x 1 4 , akkor x - 4 + x = -x + 4 + x = 4 . Ha x 1 0 , akkor x - 4 + x = - x + 4 - x = -2x + 4 .
y

1 1 x

Az egyenltlensg megoldsa: minden vals szm.

9.

V F OLYAM

V. E G Y E N L E T E K , E G Y E N L E T R E N D S Z E R E K

MATEMATIKA 83

b) brzoljuk grakusan az egyenltlensg bal oldaln szerepl kifejezst! Ha x $ 2, akkor x - 2 - x = x - 2 - x = -2. Ha 0 # x 1 2, akkor x - 2 - x = - x + 2 - x = -2x + 2. Ha x 1 0 , akkor x - 2 - x = - x + 2 + x = 2.
y

1 1 x

Az eredeti egyenltlensg megoldsa: x 2 1 . 2 5. E2 Mekkora legyen az a s b paramterek rtke, hogy a kvetkez egyenletnek vgtelen sok megoldsa legyen? x - 2 + x - 4 = ax + b . brzoljuk grakusan az egyenltlensg bal oldaln szerepl kls abszolt rtken belli kifejezst, vagyis az f ] x g = x - 2 + x - 4 fggvny grakonjt! Ha x $ 2, akkor x - 2 + x - 4 = x - 2 + x - 4 = 2x - 6 . Ha 0 # x 1 2, akkor x - 2 + x - 4 = - x + 2 + x - 4 = -2. Ha x 1 0 , akkor x - 2 + x - 4 = - x + 2 - x - 4 = -2x - 2. Az f ] x g = x - 2 + x - 4 fggvny grakus kpt a bal oldali brn szemlltettk. Ebbl gy kapjuk meg az eredeti egyenlet bal oldalnak grakonjt, hogy a negatv rtkeket, vagyis az x tengely alatti rszt tengelyesen tkrzzk az x tengelyre (jobb oldali bra).
y y

1 1 x

1 1 x

Az eredeti egyenlet jobb oldala: g] x g = ax + b lineris; grakus kpe egyenes. Az egyenletnek akkor van vgtelen sok megoldsa, ha a jobb oldal grakus kpnek vgtelen sok kzs pontja van a bal oldali kifejezs grakus kpvel. Az a s b paramterek lehetsges rtkei: a = 2, b = -6 . a = -2, b = 6 . a = 0 , b = 2. a = 2, b = 2. a = -2, b = -2.
9 . V F O LYA M

84 MATEMATIKA

V. E G Y E N L E T E K , E G Y E N L E T R E N D S Z E R E K

6. E2 brzoljuk az m]k g fggvnyt, ahol m a kvetkez egyenlet megoldsainak a szma! x -1 -1 = k . Az egyenlet bal oldalnak grakus kpt az brn ltjuk.
y

1 1 x

A jobb oldal grakus kpe egy x tengellyel prhuzamos egyenes. A megoldsok szma a kt grakon metszspontjainak a szma. Ha k 1 0 , a megoldsok (mo) szma 0. Ha k = 0 , mo: 3, ha 0 1 k 1 1; mo: 6, ha k =1, mo: 4, vgl ha 1 1 k , mo: 2. Ennek megfelelen az m]k g fggvny grakonja az albbi:
y

1 1 x

6. Elsfok ktismeretlenes egyenletrendszerek s megoldsuk behelyettest mdszerrel


1. K1 Oldjuk meg a kvetkez egyenletrendszereket a vals szmprok halmazn! a) 4x + y =10 , 3x - 4y =17 ; b) 6x + 5y = 8 , 2x -10y = - 20 . 3 a) x = 3, y = -2; b) x = 2, y = 4 . 3 5

2. K1 Oldjuk meg a kvetkez egyenletrendszert a vals szmprok halmazn! x+y 2] x - 4g + 4y = 3 ^2y + 2h ; = 3 ^6x + y h -1. 5 A megadott egyenletrendszer egyszerbb alakja: x - y = 7, 89x + 14y = 5 . Az egyenletrendszer megoldsa: x =1, y = -6 . 3. K1 Oldjuk meg a kvetkez egyenletrendszert a vals szmprok halmazn! x - 2y 4y - x x + 1 y 5; 3 + =- . - = 3 2 2 2 2 Az egyenletrendszer egyszerbb alakja: x - 2y = 4, - x + 8y = -9 . Az egyenletrendszer megoldsa: x = 7, y = - 5 . 3 6
9. V F OLYAM

V. E G Y E N L E T E K , E G Y E N L E T R E N D S Z E R E K

MATEMATIKA 85

4. K1 Mekkora legyen a p s q paramterek rtke, hogy az albbi egyenletrendszernek a) ne legyen megoldsa; b) vgtelen sok megoldsa legyen; c) egyetlen megoldsa legyen? 3x - 4y = 11, 2x + py = q. Ha az els egyenlet bal oldala a msodiknak msflszerese, akkor vagy nincs megolds, vagy vgtelen sok megolds van. Ha 3 p = -4 , akkor p = - 8 . Ha 3 q =11, akkor q = 22 . 2 3 2 3 a) p = - 8 , q ! 22 ; b) p = - 8 , q = 8 ; c) p ! - 8 , q tetszleges. 3 3 3 3 3 5. K1 Oldjuk meg a kvetkez egyenletrendszereket a vals szmprok halmazn! a) 2x + 3y = 8 , x - 2y = -10 ; b) 2x - 5y =10 , - 4x +10y =19 . a) x = -2, y = 4 ; b) nincs megolds.

6. K1 Oldjuk meg a kvetkez egyenletrendszereket a vals szmprok halmazn! a) 2 x - 1 y = 1 , x + 5y = 6 ; b) 2 x + 5 y = - 1 , x + y = -2. 3 2 6 5 2 10 a) Az egyenletrendszer egyszerbb alakja: 4x - 3y = 1, x + 5y = 6 . Az egyenletrendszer megoldsa: x = y =1. b) Az egyenletrendszer egyszerbb alakja: 4x + 25y = -1, x + y = -2. Az egyenletrendszer megoldsa: x = - 7, y = 1 , 3 3

7. Elsfok ktismeretlenes egyenletrendszerek megoldsa egyenl egytthatk mdszervel


1. K1 Oldjuk meg a kvetkez egyenletrendszereket a vals szmprok halmazn az egyenl egytthatk mdszervel! a) 2x + y =13, 3x - y = 2; b) 7x - 3y =15 , 5x + 6y = 27 ; c) 4x + 3y = 6 , 6x + 5y = -7 . a) x = 3, y = 7 ; b) x = 3, y = 2; c) x = 51, y = -32. 2

2. K1 Oldjuk meg a kvetkez egyenletrendszert a vals szmprok halmazn az egyenl egytthatk mdszervel! x+y-2 x-y-1 8 + =- , 5 3 15 x+ 3y 5 = . 2 4

A megadott egyenletrendszer egyszerbb alakja: 8x - 2y = 3, 4x + 6y = 5 . Az egyenletrendszer megoldsa: x = y = 1 . 2 3. K1 Oldjuk meg a kvetkez egyenletrendszereket a vals szmprok halmazn! 6x - 2y 4x - 3y 8x - 3y 10 a) + = y +1; - 2x = y - x +1, 5 3 3 2 2x - 2y y+2 x - 3 2y y - 3 x . b) x +1 , = - = 3 5 4 4 3 a) Az egyenletrendszer egyszerbb alakja: 38x - 21y = 15, 32x - 21y = 6 . Az egyenletrendszer megoldsa: x = 3, y = 2. 2 b) Az egyenletrendszer egyszerbb alakja: -4x + 9y =10, 15x - 28y = -3. Az egyenletrendszer megoldsa: x = 11, y = 6 .
9 . V F O LYA M

86 MATEMATIKA

V. E G Y E N L E T E K , E G Y E N L E T R E N D S Z E R E K

4. K2 Hatrozzuk meg az a paramter rtkt gy, hogy az albbi egyenletrendszereknek ne legyen megoldsa! a) 2x + 7y =11,7 , -4x + ay = 5,6 ; b) 3x - 2 y = 4 , ax + ]a +1gy = 2. 7 a) Az egyenletrendszernek akkor nincs megoldsa, ha az egyik egyenlet bal oldala a msik egyenlet bal oldalnak valamilyen szmszorosa, de a jobb oldala a msik egyenlet jobb oldalnak nem ugyanannyiszorosa. -4 = a , ahonnan a = -14 . Ezzel a msodik egyenlet 2 7 bal oldala az els egyenlet bal oldalnak -2-szerese; de a jobb oldala az els egyenlet jobb oldalnak nem -2-szerese. Teht az egyenletrendszernek akkor nincs megoldsa, ha a = -14 . b) a = - a + 1, ahonnan a = - 21 . 3 2 23 7 Az egyenletrendszernek akkor nincs megoldsa, ha a = - 21 . 23

8. Elsfok ktismeretlenes egyenletrendszerek megoldsa grakus mdszerrel


1. K1 Oldjuk meg grakusan az albbi ktismeretlenes elsfok egyenletrendszereket! a) 3x + y = 0 , -3x + 2y = 9 ; b) -x + 2y = 4 , x + 4y = 20 .
y

a)

b)
y = 1 4x + 5

1 0 y= 1 2x + 2 1 x

Az egyenletrendszer megoldsa a grafikon alapjn: x = -1, y = 3.

Az egyenletrendszer megoldsa a grafikon alapjn: x = y = 4 .

2. K1 Adjunk meg olyan ktismeretlenes elsfok egyenletrendszert, amelynek megoldsa: a) x = -4 , y = 5 ; b) x = 3 , y = 2 ! 2 3 a) Pl.: 2x + y = -3, 3x + 2y = -2; b) Pl.: 2x + 3y = 5, 4x + 9y =12.

3. K2 Adott egy egyenletrendszer: 3x - 5y = 14 , 2x + ay = -2. Hatrozzuk meg az a paramter rtkt gy, hogy az egyenletrendszernek ne legyen megoldsa! Az egyenletrendszernek nincs megoldsa, ha az egyik egyenlet bal oldala a msik egyenlet bal oldalnak valamilyen szmszorosa, de a jobb oldala nem ugyanannyiszorosa. 3 a = -5 , ahonnan 2 10 3 a = - . Mivel 14 ! $ ]-2g, ezrt az a-ra kapott rtk esetn valban nincs megoldsa az 3 2 egyenletrendszernek.
9. V F OLYAM

V. E G Y E N L E T E K , E G Y E N L E T R E N D S Z E R E K

MATEMATIKA 87

4. K1 Adjunk meg olyan ktismeretlenes elsfok egyenletrendszert, melynek gykei: x = 1 , y = 7. 5 Pl.: 10x + 3y = 23, 5x - 2y = -13.

9. Egyenletrendszerrel megoldhat szveges feladatok


1. K1 Ha egy trt szmlljhoz 1-et hozzadunk, akkor a trt rtke 1 lesz. Ha a trt nevezjhez adunk 5-t, akkor a trt rtke 1 lesz. Melyik ez a trt? 2 A feladat szvege alapjn az albbi egyenletrendszert rhatjuk fel: x + 1 1, x 1 = = . y y+5 2 Az egyenletrendszer megoldsa: x = 6, y = 7 , teht a keresett trt: 6 . 7 2. K1 Egy ktjegy szmhoz hozzadtuk a fordtottjt, azaz a szmjegyeik felcserlsvel add ktjegy szmot, s eredmnyl 77-et kaptunk. Ha az eredeti szmot elosztjuk a fordtottjval, akkor a maradk is s a hnyados is 2. Melyik ktjegy szmrl van sz? A feladat szvegbl az albbi egyenletrendszer addik: ab + ba = 77, ab - 2 = 2. ba Az els egyenletbl a = 7 - b ; ezzel a msodik egyenletbl 54 = 27b addik, vagyis b = 2, a = 5 . A keresett szm: 52. 3. K1 Egy hromszg legnagyobb szge a msik kt szg szmtani kzepnl 33-kal nagyobb. A nagysg szerint kzps szg annyival nagyobb a legkisebb szgnl, amennyivel kisebb a legnagyobb szgnl. Mekkork a hromszg szgei? Legyenek a hromszg szgei: a $ b $ c . A felttelek szerint b c a = + + 33, a - b = b - c s a + b + c =180 . 2 a c A msodik egyenletbl b = + . Innen az a + c = 2b kifejezst a harmadik egyenletbe he2 lyettestve b = 60 . gy a + c =120 , azaz c =120 - a . Ezzel az els egyenlet gy alakul: ahonnan a = 82. a = 60 + 120 - a + 33, 2 A hromszg szgei: a = 82o, b = 60o, c = 38o . 4. K1 Egy farmon csirkket s nyulakat tartanak. sszes fejeik szma 46, sszes lbaik szma 160. Hny csirke s hny nyl van a farmon? Legyen c a csirkk, n a nyulak szma. Ekkor c + n = 46, 2c + 4n =160 . Az egyenletrendszer megoldsa: c =12, n = 34 .

9 .

V F O LYA M

88 MATEMATIKA

V. E G Y E N L E T E K , E G Y E N L E T R E N D S Z E R E K

5. K2 Egy keresked 50 kg narancsot vsrolt 12 000 Ft-rt. A narancsokat kt rszre vlogatta szt; a szebbik rszt 15%-os haszonnal, a kevsb szpeket 5%-os vesztesggel adta el, gy sszesen 936 Ft haszonra tett szert. Hny kg narancsot adott el nyeresggel, s mennyit vesztesggel? A keresked a narancs kiljt 240 Ft-rt vsrolta. Legyen x kg az 50 kg narancs szebbik rsze, y kg a kevsb szp rsze. Ekkor s x + y = 50 240 $ 1,15x + 240 $ 0,95 y =12 936 . Az egyenletrendszer megoldsa: x = 32, y =18 . 6. E1 Kt testvr letkornak sszege 32 v. Amikor letkoraik sszege ennek duplja lesz, akkor a atalabb testvr letkora annyi lesz, mint az idsebb letkora most. Hny vesek a testvrek? Legyenek az letkorok a 1 b . Ekkor a + b = 32 s 3b - a = 64 . Az egyenletrendszer megoldsa: a = 8, b = 24 . 7. E1 Andrs adott Blnak annyi pnzt, amennyi pnze Blnak ppen volt. gy a kt nak ugyanannyi pnze lett. Ezutn Bla adott Andrsnak feleannyi pnzt, amennyi pnze Andrsnak eredetileg volt, gy Andrsnak 66 Ft-tal tbb pnze lett, mint Blnak. Hny forintjuk volt eredetileg? Ha Andrsnak a, Blnak b forintja volt eredetileg, akkor s a - b = 2b a - b + a = 2b - a + 66 . 2 2 Az egyenletrendszer megoldsa: a = 66, b = 22. 8. E1 Egy motorcsnak 25 km-t tett meg flfel a Dunn, majd visszafordult s visszament kiindulsi helyre. Az egsz utat sszesen 5 ra alatt tette meg. Egy msik alkalommal ugyanakkora sebessggel haladva 10 km-t ment felfel a Dunn, majd visszafordult s 4 km-t tett meg lefel; sszesen 1,5 ra alatt. Mekkora a foly sebessge? Legyen vc a csnak, vD a Duna sebessge. Ekkor 10 4 3 25 25 s + + =5 = . vc - vD vc + vD 2 vc - vD vc + vD Bevezetve az a = 1 , b = 1 j ismeretleneket, arra jutunk, hogy a = 7 , b = 1 , vc - vD vc + vD 60 12 60 24 teht vc + vD =12 s vc - vD = . A kt egyenlet klnbsgbl 2vD = . Innen a Duna se7 7 bessge: vD = 12 . 1,71 km . 7 h

9.

V F OLYAM

MATEMATIKA 89

VI. Geometriai transzformcik


1. Nhny geometriai transzformci
1. K1 Az brn bejelltnk nhny rszletet sznessel. Keressk meg a jobb oldali brn is ezeket a rszleteket!

2. K1 Rajzoltunk egy t egyenest. A t tengelyen lv pontok helyben maradnak, a rajzunk skjban lv sszes tbbi pont megvltoztatja a helyt. Minden pont ktszer olyan messzire kerl a t egyenestl, mint eddig volt, de az egyenes ltal meghatrozott msik flskban. A pontbl s a kpbl a t tengelyre lltott egy-egy merleges egyenes talppontja egybeesik. Hogyan vltozik a kpen lthat alakzatok kpe? Rajzoljuk meg! A rajz mutatja a megoldst:

3. K2 A koordintaskon egy transzformci minden pont els koordintjt az ellentettjre vltoztatja, a msodik koordintjt pedig megfelezi. a) Adjuk meg a kvetkez pontok kpt: A^-2; 6h , B^-5; 2h , C^-2; 5h ! b) Adjuk meg azokat a pontokat, amelyek kpeknt a kvetkez pontokat kaptuk: K l^1; 2h , Ll^-3; -1h , M l^5; 1,5h ! a) Az j pontok koordinti: Al^2; 3h, Bl^5; 1h, C lb2; 5 l . 2 b) Az eredeti pontok koordinti: K^-1; 4h, L^3; -2h, M^-5; 3h .

9 .

V F O LYA M

90 MATEMATIKA

VI. GEOMETRIAI TRANSZFORMCIK

4. E1 A koordintaskon legyen az f transzformci az x tengelyre trtn tkrzs, a g transzformci pedig a K^1; 2h pontra tkrzs. Adottak az A^1; 4h , B^-2; 3h , C^4; 0h pontok. Rajzoljuk le az adott pontokat, s rjuk fel koordintikat az a) f; b) g; c) gf; d) fg transzformcik utn! a) b) c) d) Az f transzformci utn kapjuk: A1^1; -4h, B1^-2; -3h, C1^4; 0h . A g transzformci utn kapjuk: A2 ^1; 0h, B2 ^4; 1h, C2 ^-2; 4h . A gf transzformci utn kapjuk: A3 ^1; 8h, B3 ^4; 7h, C3 ^-2; 4h . Az fg transzformci utn kapjuk: A4 ^1; 0h, B4 ^4; -1h, C4 ^-2; -4h .

5. E2 Rajzoljunk skidomokat, amelyekhez tallhat olyan a) tengely; b) pont, amelyre trtn tkrzs esetn a skidom invarins alakzat! a) Pldul:

b) Pldul:

9.

V F OLYAM

VI. GEOMETRIAI TRANSZFORMCIK

MATEMATIKA 91

2. Egybevgsgi transzformcik a skon


1. K1 Vegynk fel egy ABC hromszget, tovbb egy t tengelyt, egy K pontot, egy v vektort s egy a irnytott szget! Szerkesszk meg az ABC hromszgnek a) a t tengelyre vett tkrkpt; b) a K pontra vett tkrkpt; c) a v vektorral eltolt kpt; d) az a irnytott szggel a K pont krl elforgatott kpt! a) b)

c)

d)

C B

C A

B A
9 . V F O LYA M

92 MATEMATIKA

VI. GEOMETRIAI TRANSZFORMCIK

2. K1 Adott egy rombusz. Tkrzzk a) az egyik oldalegyenesre; b) egy olyan egyenesre, amely a rombuszt kettvgja, de egyetlen cscsa sem illeszkedik r! a) bra a szveg alapjn:

b) bra a szveg alapjn:

3. K2 Adott kt egyenl sugar kr. Szerkessznk olyan a) egyenest; b) pontot, amelyre az egyik krt tkrzve megkapjuk a msikat! a) A kt kr kzppontjt sszekt szakasz felezmerleges egyenese lesz a tengely. b) A kt kr kzppontjt sszekt szakasz felezpontja lesz a kzppont. 4. K2 Szerkessznk hromszget, ha adott kt oldala s az ezekkel szemkzti szgek klnbsge! Adatok: a, b, a - b . Vzlatrajzot ksztnk. Tkrzzk a hromszget az AB oldal felezmerlegesre, C pont kpe legyen C . A CAC hromszg megszerkeszthet, mert adott kt oldala s a kzbezrt szgk. A CC szakasz felezmerleges egyenesre tkrzzk az A cscsot, ekkor kapjuk a hinyz B cscsot.

9.

V F OLYAM

VI. GEOMETRIAI TRANSZFORMCIK

MATEMATIKA 93

5. K2 Szerkessznk trapzt, ha adott a ngy oldala! Adatok: a, b, c, d. Az bra jellseit hasznljuk. Hzzunk prhuzamost D ponton t BC-vel! Ennek metszspontja AB-vel legyen B. Az ABD hromszg megszerkeszthet, mert ismert mindhrom oldalnak hossza. Az AB egyenes B-n tli meghosszabbtst elmetsszk a B kzppont c sugar krrel, gy kapjuk a B pontot. A D kzppont c sugar, s a B kzppont b sugar kr egyik metszspontja lesz a C (a msik metszspont a B). A megoldhatsg felttele: Ha teljeslnek a hromszg-egyenltlensgek az a c, b, d szakaszokra, akkor van megolds, s az egyrtelm. 6. E1 Adott hrom prhuzamos egyenes s az egyiken egy P pont. Szerkesszk meg a PQR szablyos hromszget olyan mdon, hogy a Q s az R pontok illeszkedjenek a msik kt egyenesre! Az bra jellseit hasznljuk. A P krli 60-os forgats a Q pontot az R-be transzformln, de nem ismerjk a Q pontot. Tudjuk, hogy Q illeszkedik q-ra s R illeszkedik r-re, ezrt q egyenes P pont krli 60-os q elforgatottja s r egyenes metszspontja adja az R pontot. R visszaforgatsval kapjuk a Q pontot.

9 .

V F O LYA M

94 MATEMATIKA

VI. GEOMETRIAI TRANSZFORMCIK

3. Alakzatok egybevgsga
1. K2 Igazoljuk, hogy ha kt szablyos hromszgnek egyenl a magassga, akkor a kt hromszg egybevg! Az bra jellseit hasznlva:

Tudjuk, hogy AT = Al T l . Mivel BTAB = Bl T l Al B, BATB = Bl Al T l B, ezrt BTAT , Bl T l Al T (egy-egy oldaluk hossza s a rajtuk fekv kt szgk pronknt egyenl). Ezrt AB = Al Bl . Ebbl mr kvetkezik, hogy ABCT , Al Bl C l T , hiszen egy-egy oldaluk hossza s a rajtuk fekv kt szgk pronknt egyenl. (60-os, hiszen szablyos hromszgekrl volt sz.) 2. K2 Igazoljuk, hogy ha kt rombusznak pronknt egyenlk az tli, akkor a kt rombusz egybevg! A kt-kt tl ngy-ngy egybevg derkszg hromszgre vgja a rombuszt. A rombusz tli felezve metszik egymst, ezrt egy ilyen hromszg befogi a fl tlkkal azonos hosszsgak. Vagyis egybevg derkszg hromszgekrl van sz. A kt rombusz valban egybevg. 3. K2 Bizonytsuk be, hogy kt egyenl szr hromszg egybevg, ha megegyeznek alapjukban s az alappal szemkzti szgkben! Az bra jellseit hasznljuk:

Mivel egyenl szrak a hromszgek, ezrt ki tudjuk fejezni az alapon fekv szgeket. o o ABCB = ACBB = 180 - a , Al Bl C l B = Al C l Bl B = 180 - a . Tudjuk, hogy a = al, ezrt 2 2 180o - a 180o - al . = 2 2 Vagyis ABCT , Al Bl C l T , hiszen egy-egy oldaluk hossza s a rajtuk fekv kt szgk pronknt egyenl.
9. V F OLYAM

VI. GEOMETRIAI TRANSZFORMCIK

MATEMATIKA 95

4. K2 Egy szablyos hromszg s egy ngyzet oldalait az brn lthat mdon, ugyanolyan arnyban sztosztottuk. Igazoljuk, hogy az osztpontok az els brn szablyos hromszget, a msodik brn ngyzetet hatroznak meg!

Az brn feltntetett adatok felhasznlsval belthat, hogy mindkt brn a sznes hromszgek egybevgk. Az egybevgsgbl kvetkezik, hogy PQ = QR = RP , vagyis PQR hromszg valban szablyos hromszg. Az egybevgsgbl a msodik brn is kvetkezik, hogy KL = LM = MN = NK , ezrt KLMN biztosan rombusz. A rombusz brmelyik cscsnl megmutathat, hogy derkszg van. Egy egyves s egy ktves szg sszege 90, gy pldul: LKNB = 90. Ezek alapjn KLMN ngyzet.

5. K2 Kt hatszg mind a 12 oldala egyenl hossz. Egybevg-e a kt hatszg, ha a) mindkettnek van 120-os szge; b) egy-egy tljukrl tudjuk, hogy egyenl hossz? a) Nem biztos. b) Nem biztos. Az brk mindkt rszre ellenpldt mutatnak.

9 .

V F O LYA M

96 MATEMATIKA

VI. GEOMETRIAI TRANSZFORMCIK

4. Szimmetria
1. K1 Milyen szimmetrit, szimmetrikat gyelhetnk meg az brkon? a) b) c) d) e)

a) b) c) d) e)

kzppontos szimmetria; tengelyes szimmetria; tengelyes szimmetria; 90-os forgsszimmetria; kzppontos szimmetria; kzppontos szimmetria; forgsi szimmetria.

2. K1 Csoportostsuk a a) hromszgeket; a szimmetriatengelyeik szma szerint!

b) ngyszgeket

a) Hrom szimmetriatengely: szablyos hromszgek; egy szimmetriatengely: egyenl szr hromszgek; nincs szimmetriatengely: a tovbbi hromszgek. b) Ngy szimmetriatengely: ngyzet; kt szimmetriatengely: tglalap, rombusz; egy szimmetriatengely: deltoid, hrtrapz; nincs szimmetriatengely: a tovbbi ngyszgek. 3. K1 Milyen szimmetrikkal rendelkezik a ngyzet? Tengelyes szimmetria, 90-os forgsszimmetria, kzppontos szimmetria. 4. K2 Milyen szimmetrikkal rendelkezik egy szablyos a) hatszg; b) htszg? a) Tengelyes szimmetria, 60-os forgsszimmetria, kzppontos szimmetria. o b) Tengelyes szimmetria, 360 -os forgsszimmetria. 7 5. K1 A felsorolt skidomokrl dntsk el, hogy tengelyesen vagy kzppontosan szimmetrikusake, forgsszimmetrikusak-e! Szablyos hromszg, egyenl szr hromszg, derkszg hromszg, paralelogramma, rombusz, szablyos tzszg, kr. Szablyos hromszg: tengelyesen szimmetrikus, forgsszimmetrikus; egyenl szr hromszg: tengelyesen szimmetrikus; derkszg hromszg: egyik sem (ha nem egyenl szr); paralelogramma: kzppontosan szimmetrikus; rombusz: tengelyesen s kzppontosan szimmetrikus; szablyos tzszg: tengelyesen s kzppontosan szimmetrikus, forgsszimmetrikus; kr: tengelyesen s kzppontosan szimmetrikus, forgsszimmetrikus.

9.

V F OLYAM

VI. GEOMETRIAI TRANSZFORMCIK

MATEMATIKA 97

6. E1 Bizonytsuk be, hogy az a ngyszg paralelogramma, amelynek kt szemkzti oldala prhuzamos s egyenl hossz! Az brn AB = CD, AB < CD . Legyen K a ngyszg tlinak metszspontja. ABKT , CDKT , mert AB = CD , a B-nl s a D-nl lv szgek egyenlk (vltszgek), az A-nl s a C-nl lv szgek is egyenlk (vltszgek). Az egybevgsg miatt: AK = KC, BK = KD . Ez azt jelenti, hogy az ABCD ngyszg tli felezve metszik egymst, vagyis paralelogramma.

7. E1 Bizonytsuk be, hogy a paralelogramma kt szemkzti oldalnak felezpontjn thalad egyenes a paralelogrammt kt paralelogrammra vgja! Az brn lthat ABEF ngyszgrl beltjuk, hogy paralelogramma. Mivel AD < BC , ezrt AF < BE . Mivel AD = BC , ezrt a feleekkora szakaszok is egyenlk: AF = BE . Vagyis ABEF ngyszg paralelogramma, mert kt szemkzti oldala prhuzamos s egyenl. Ugyangy bizonytannk, hogy DFEC ngyszg is paralelogramma.

5. Tovbbi nevezetes pontok s vonalak a hromszgben


1. K1 Egy hromszg oldalai a) 15, 17, 28; b) 4a, 8b + 2, 6c 1. Mekkork a hromszg kzpvonalai? a) 7,5, 8,5, 14; b) 2a, 4b +1, 3c - 0,5 .

2. K1 Egy hromszget a hrom kzpvonala ngy hromszgre vg. Ezek kzl az egyiknek 23 cm a kerlete. Mekkora az eredeti hromszg kerlete? 46 cm a nagy hromszg kerlete, mert minden oldala ktszer akkora, mint a kis hromszg megfelel oldala. 3. K2 Az ABC hromszg A-bl indul slyvonala F pontban metszi a BC oldalt. A hromszg slypontja S. Az ABF hromszg terlete 41 cm2. Mekkora a) az ABC; b) az SFC; c) az ABS hromszg terlete? a) Az ABF hromszg terletnek ktszerese, vagyis 82 cm2. b) Az ABF hromszg terletnek harmada, vagyis 13,6 cm2. (ACF s ABF hromszgek terlete egyenl, tovbb SF = 1 $ AF , s az ezekhez tartoz magassg pedig egyenl.) 3 c) Az ABF hromszg terletnek ktharmada, vagyis 27,3 cm2. (Az elzekben lertak miatt.)
: :

9 .

V F O LYA M

98 MATEMATIKA

VI. GEOMETRIAI TRANSZFORMCIK

4. K2 Egy hromszgbe berajzoltunk egy kzpvonalat s egy olyan slyvonalat, amely metszi ezt a kzpvonalat. A kt szakasz ngy vgpontja milyen ngyszget hatroz meg?

Az ABC hromszgben AF slyvonal, EG pedig egy kzpvonal. Mivel EF = AG s EF < AG (hiszen EF is kzpvonal a hromszgben), ezrt EFGA paralelogramma.

5. E1 Megszerkesztett egy hromszg kr rt krnek a kzppontja. Mutassuk meg, hogy ez a kzppont az oldalfelez pontok ltal meghatrozott hromszgben magassgpont! Az oldalfelez pontokat sszekt szakaszok kzpvonalak. A kzpvonal prhuzamos a szemkzti oldallal, ezrt a felezponton tmen merleges egyenes a kzpvonalra is merleges. Az eredeti hromszg oldalfelez merlegesei teht az oldalfelez pontok ltal meghatrozott hromszgben magassgvonalak. Ez pontosan a bizonytand lltst jelenti. 6. K2 Az ABC hromszgben D a magassgpont. Mutassuk meg, hogy az ABD hromszgben C a magassgpont! Nzzk az brt! Az ABD hromszgben AT2, BT1, DT3 magassg, ezek metszspontja ppen C. Vagyis valban C a magassgpont az ABD hromszgben.

6. Vektorok
1. K1 Az brn jellt vektorok kzl vlasszuk ki a) az egyenlket; b) az ellentetteket; c) azokat, amelyek nem egyenlk s nem ellentettek, de egyenl az abszolt rtkk! a) egyenlk: d s b; b) ellentettek: a s c; c) nem egyenlk s nem ellentettek, de egyenl az abszolt rtkk: e s f.
c

d e a

2. K1 Kt szablyos hromszg egymshoz illesztsvel rombuszt rajzolunk. Rajzoljuk be a rombusz mindkt tljt. Az oldalakat s az tlkat irnytsuk gy, hogy hat klnbz vektort kapjunk. Vlasszuk ki ezek kzl azokat, amelyek sszege 0! Pldul: DA + DC + BD = 0, DA + BC = 0, DC + BA = 0 .

9.

V F OLYAM

VI. GEOMETRIAI TRANSZFORMCIK

MATEMATIKA 99

3. K2 Egy kocka egyik cscsbl felvesszk a hrom klnbz lvektort. Adjuk meg ezekkel a vektorokkal a kocka ezen cscsbl indul laptl s testtl vektorokat! Az egy cscsbl indul lvektorok legyenek: a, b, c. Ekkor a laptl vektorok: a + b, b + c, a + c , a testtl vektor: a + b + c . 4. K2 Adott a, b s c vektor (semelyik kett nem egyenl egymssal). Szerkesszk meg az a) a + b; b) c a; c) a + b + c; d) 2b c; 1 3 e) 3c + a; f) 2b a 2 2 vektorokat!

a)

b)

ca
c a

a+

b
c) d)

b
+c

a c b
2b c

a+

a+

b
e) f)

2b

3c + 1

3 a 2

b a c b

9 .

V F O LYA M

100 MATEMATIKA

VI. GEOMETRIAI TRANSZFORMCIK

5. K2 Legyen ABCD egy tetszleges tglalap. Mutassuk meg, hogy a) AC = 2AD + DB ; b) AC - DB + CB = AD !

a)

b) D

7. Ponthalmazok
1. K1 Adott a skon egy K s egy C pont, tvolsguk 1 cm. Sznezzk be a sk azon pontjait, amelyek a K-tl 2 cm-nl nem kzelebb s a C-tl 3 cm-nl nem tvolabb tallhatk!

Az bra sznezett rsze mutatja a megoldst. 2. K1 Adott a skon egy A s egy B pont, tvolsguk 2 cm. Sznezzk be a sk azon pontjait, amelyeknek a) vagy az A vagy a B (vagy mindkett) ponttl mrt tvolsga nem nagyobb, mint 2 cm; b) az A s a B ponttl mrt tvolsga is 2 cm; c) az A s a B ponttl mrt tvolsga is nagyobb, mint 2 cm!

a)

b)

c)

9.

V F OLYAM

VI. GEOMETRIAI TRANSZFORMCIK

MATEMATIKA 101

3. K1 Adott a skon egy 2 cm sugar krvonal. Hatrozzuk meg azon pontok halmazt a skon, melyeknek tvolsga a krvonaltl kisebb, mint a) 1 cm; b) 2 cm; c) 2,5 cm!

a)

b)

2c m

c)

4. K2 Adott a skon egy 2 cm oldal szablyos hromszg. Hatrozzuk meg azon pontok halmazt a skon, amelyeknek tvolsga a hromszg hatrvonaltl kisebb, mint a) 0,5 cm; b) 1 cm!

a)

2c m

b)

2c m
A

9 .

V F O LYA M

102 MATEMATIKA

VI. GEOMETRIAI TRANSZFORMCIK

5. E1 Az brn lthat vzlat fellrl nzve egy ngyzet alak kutyalat mutat. A ngyzet oldala 80 cm. A bejrat egyik szlhez egy 160 cm-es przzal kiktttek egy kutyt. A bejrat 40 cm szles, s szimmetrikusan helyezkedik el az l szleihez kpest. Szemlltessk rajzzal a kutya ltal bejrhat terletet!

6. K2 A koordintaskon hatrozzuk meg azoknak a P ^ x; y h pontoknak a halmazt, amelyekre a) ] x +1g ^ y - 3h 2 0 ; b) x2 = y2 ; c) x + y = 4 ; d) x + y # 4 ! a) b) rhat gy is: ^ x - y h ^ x + y h = 0 . Vagyis: x = y vagy x = - y .

c) Ngy esetet kell megvizsglnunk. I. Ha x $ 0 s y $ 0 , akkor x + y = 4 , azaz II. Ha x 1 0 s y $ 0 , akkor - x + y = 4 , azaz III. Ha x 1 0 s y 1 0 , akkor - x - y = 4 , azaz IV. Ha x $ 0 s y 1 0 , akkor x - y = 4 , azaz

y = -x + 4. y = x + 4. y = -x - 4. y = x - 4.

9.

V F OLYAM

VI. GEOMETRIAI TRANSZFORMCIK

MATEMATIKA 103

Az egyes esetekhez kapcsold ponthalmazt szaggatott vonallal rajzoltuk, a felttelnek megfelel rszt szneztk.

d) Az elzeket felhasznlva kapjuk:

7. E1 A koordintaskon hatrozzuk meg azoknak a P ^ x; y h pontoknak a halmazt, amelyek koordintira fennllnak a kvetkez egyenltlensg-rendszerek! x$y a) xy 2 04 ; b) 4. ] y #2 x -1g ^ y -1h $ 0 a) A kt felttelnek eleget tev ponthalmazokat elszr kln-kln brzoljuk, majd a harmadik brn lthat a megolds.

b) A kt felttelnek eleget tev ponthalmazokat elszr kln-kln brzoljuk, majd a harmadik brn lthat a megolds.
y

1 1 x

9 .

V F O LYA M

104 MATEMATIKA

VI. GEOMETRIAI TRANSZFORMCIK

8. Szg, krv, krcikk


1. K1 Szmtsuk ki az r = 15 cm sugar krben az albb megadott kzpponti szgekhez tartoz krv hosszt s krcikk terlett! a) 60; b) 150; c) 210; d) 270. A 15 cm sugar kr kerlete 2 $ 15 $ r . Az ismeretlen krv hosszt jelljk i-vel. a) b) c) d) i 60o , amibl i 5r . 15,7 (cm). = = 30r 360o i 150o , amibl i 12,5r . 39,3 (cm). = = 30r 360o i 210o , amibl i 17,5r . 55,0 (cm). = = 30r 360o i 270o , amibl i 22,5r . 70,7 (cm). = = 30r 360o

2. K2 Szmtsuk ki az r = 8 cm sugar krben az albb megadott kzpponti szgekhez tartoz krv hosszt s krcikk terlett! a) 47; b) 162; c) 62 30; d) 27 48. A 8 cm sugar kr kerlete 16r, terlete 64r. Az ismeretlen krv hosszt jelljk i-vel, a krcikk terlett t-vel. o t 47o , amibl t . 26,2 (cm2). a) i = 47 o , amibl i . 6,6 cm, = 16r 360 64r 360o b) c) d) i 162o , amibl i . 22,6 cm, t 162o , amibl t . 90,5 (cm2). = = 16r 360o 64r 360o 62,5o 62,5o i t , amibl i . 8,7 cm, ,amibl t . 34,9 (cm2). = = o 16r 64r 360 360o 27,8o 27,8o i t ,amibl i . 3,9 cm, ,amibl t . 15,5 (cm2). = = o 16r 64r 360 360o

3. K2 rjuk fel vmrtkben: 22, 46, 100, 110, 200, 43 12, 53 32, 100 42! 22 0,384; 46 0,803; 100 1,745;
:

110 1,920;

200 3,491;

43 12 = 43,2 0,754;

53 32 = 53,5 3 0,934;

100 42 = 100,7 1,758.

4. K1 Hny fokosak azok a szgek, melyek vmrtke: r , 2r , 3r , r , 3r , 5r ? 2 3 2 4 4 2 r 90o ; = 2 2r 120o ; = 3 3r 270o ; = 2 r 45o ; = 4 3r 135o ; = 4 5r 450o . = 2

5. K1 Hny fokosak azok a szgek, melyek vmrtke: 2, 3, 4, 2,5, 3,1, 5,3, 11, 314? 2 114,59; 3 171,89; 4 229,18; 2,5 143,24; 53 303,67; 11 630,25; 314 17 990,87. 3,1 177,62;

9.

V F OLYAM

MATEMATIKA 105

VII. Kombinatorika
1. Sorrendek
1. K1 Hnyfle sorrendben tehetnek ki egy rubemutatn t klnbz termket? 5 $ 4 $ 3 $ 2 $ 1 = 120 -flekppen. 2. K1 Egy piacon az rus a zldsgesldkat 6-fle sorrendben helyezheti ki. Hny ldt tett ki? Hrmat, mert 6 = 3 $ 2 $ 1. 3. K1 t gyerek barlangszni indul. Az egyik keskeny jraton csak egyesvel frnek keresztl. Azt szeretnk, ha a legtapasztalatlanabb gyerek kerlne kzpre. Hnyfle sorrendben mehetnek t a keskeny jraton? A kzps gyerek helye nem vltoztathat, a ngy msik gyerek 4! = 24 -fle sorrendben haladhat. 4. K1 Az osztlyban a gyerekek felsoroljk, hogy nyolc tantrgy kzl sorrendben melyik ngy a kedvenck. Hnyfle klnbz sorrendet llthatnak fel a gyerekek? Az els helyre 8 tantrgyat sorolhatnak. A msodikra (tetszlegesen kivlasztott els tantrgy utn) 7-et. Ez 8 $ 7 -fle lehetsg. A harmadikra (a tetszlegesen kivlasztott els kt tantrgy utn) 6-ot, ez 8 $ 7 $ 6 -fle lehetsg. Vgl a negyedikre (a tetszlegesen kivlasztott els hrom tantrgy utn) 5-t sorolhatnak. gy az sszes lehetsges sorrendek szma 8 $ 7 $ 6 $ 5 = 1680 . 5. K2 a) Hnyfle klnbz sorrendben rakhat le egyms mell egy-egy darab 0 , 2 , 4 , 6 , 8 szmkrtya? b) Az a) feladatban megadott szmok kzl hny szmsor kezddik 0-val? c) Hny klnbz tjegy szm kpezhet az adott szmkrtykbl? d) Hny klnbz, 4-essel kezdd tjegy szm kpezhet az adott szmkrtykbl? a) b) c) d) 5 $ 4 $ 3 $ 2 $ 1 = 120 ; 1 $ 4 $ 3 $ 2 $ 1 = 24 ; 4 $ 4 $ 3 $ 2 $ 1 = 96 , vagy 120 - 24 = 96 szm kpezhet; 1 $ 4 $ 3 $ 2 $ 1 = 24 .

9 .

V F O LYA M

106 MATEMATIKA

VII. KOMBINATORIKA

2. Leszmolsok
1. K1 Hnyflekppen olvashat ki a KATEDRA sz az albbi brkon, ha csak jobbra s lefel haladhatunk? a) b)

K A T E

A T E D

T E D R

E D R A

K A T E D

A T E D R

T E D R A

6 a) e o = 20 ; 3

6 b) e o = 15 . 2

L A L A

A L A L A K A K

2. K1 Hnyflekppen olvashat ki a LALAK az albbi brn, ha csak jobbra s lefel haladhatunk? Leszmlljuk. A kt K bethz 4-4-flekppen juthatunk el, sszesen 8-fle kiolvass lehetsges. 3. K1 A kvetkez brn az A pontbl indulunk, s minden lpsben lefel megynk egyet jobbra vagy balra. rjuk be minden cscsponthoz, hogy oda hnyflekppen lehet eljutni!
A

4. K2 Induljunk el a hromszgrcson az A pontbl a B pontba!


B

A hrom megengedett haladsi irny:

Hnyfle tvonal van?

A rcsot ,,felegyenesthetjk, mert a metszspontok akkor is ugyangy helyezkednek el. Egy-egy cscspontba egy, kett vagy hrom msik cscspontbl rkezhetnk. Az egyes cscspontokba berva, hogy oda hnyflekppen juthatunk el az A ponttl, sszesen 1683-flekppen juthatunk el B-be.

9.

V F OLYAM

VII. KOMBINATORIKA

MATEMATIKA 107

5. E1 a) Egy 2 2 2 egysgoldal kockarcson egysgnyi lpsekkel lpegetve el akarunk jutni valamelyik testtl egyik vgpontjbl a msik vgpontba. Hnyflekppen tehetjk meg ezt, ha soha nem lpnk visszafel? b) Egy 2 2 2 egysgoldal kocka felsznn az oldalakkal prhuzamos, egysgnyi lpsekkel lpegetve el akarunk jutni valamelyik testtl egyik vgpontjbl a msik vgpontba. Hnyflekppen tehetjk meg ezt, ha soha nem lpnk visszafel? 6 a) 6-ot lpnk, 2-2 lpst 3 irnyba. Az egyik irny lpst e o -flekppen vlaszthatjuk ki, egy 2 4 msik irnyt e o -flekppen. A maradk lpst a harmadik irnyba tesszk meg. Az sszes 2 6 4 lehetsg szma: e o $ e o $ 1 = 90 . 2 2 b) Tbbflekppen is gondolkodhatunk. 1. A kockarcson trtn lpegetssel kapott esetekbl kihagyjuk azokat az eseteket, amelyeknl (ppen a harmadik lpsre) belpnk a kocka belsejbe. Vagyis azokat az eseteket, amelyekben thaladunk a kocka kzppontjn. A kocka kzppontjba 6-flekppen tudunk eljutni, onnan a vgpontba (mind a 6 esethez) 6-flekppen, vagyis sszesen 36 esetben haladunk t a kzpponton. 90 - 36 = 54 esetben maradunk a kocka felsznn. 2. Rajzoljuk meg a kezdponthoz csatlakoz hrom kockalapot ,,alulnzetben, rjuk fel minden csompontra, hogy oda hnyflekppen lehet eljutni. Majd rajzoljuk fel a vgponthoz csatlakoz 3 lapot ,,fellnzetben, induljunk ki a mr felcmkzett csompontokbl, s szmoljuk ssze a vgpontba vezet utakat!

6. E2 Hnyflekppen juthatunk el egy 10 fokbl ll lpcs aljrl a tetejre, ha a lpcsket tletszeren egyesvel vagy kettesvel vesszk? Szmoljuk ssze, hogy hnyflekppen lphetnk az egyes lpcsfokokra! Az elsre 1, a msodikra (lentrl vagy az els fokrl) 2, a harmadikra (az els vagy a msodik fokrl) 1 + 2 = 3, a negyedikre (a msodik vagy a harmadik fokrl) 2 + 3 = 5 , az tdikre (a harmadik vagy a negyedik fokrl) 3 + 5 = 8 , a hatodikra 5 + 8 = 13 , a hetedikre 8 + 13 = 21, a nyolcadikra 13 + 21 = 34 , a kilencedikre 21 + 34 = 55 , a tizedikre 34 + 55 = 89 lehetsg van.

9 .

V F O LYA M

V I I I . S TAT I S Z T I K A

MATEMATIKA 109

VIII. Statisztika
1. Adatok gyjtse, rendszerezse, jellemzse
1. K1 Kdoljuk a kvetkez adatokat, majd rjuk fel nvekv sorrendben! a) egy, egy, egy, ht, ngy, hrom, kett, hat, hat, ngy, kett, t, hrom, hat; b) o, m, k, n, m, a, p, a, h, m, u, m, k, n, n, e, n, a, e, m; c) ngyszg, hatszg, tszg, hatszg, kr, ngyszg, nyolcszg, tszg, hromszg, hatszg, kr. a) Pldul: 1, 1, 1, 7, 4, 3, 2, 6, 6, 4, 2, 5, 3, 6; rendezve: 1, 1, 1, 2, 2, 3, 3, 4, 4, 5, 6, 6, 6, 7. b) Pldul: 1, 2, 3, 4, 2, 5, 6, 5, 7, 2, 8, 2, 3, 4, 4, 9, 4, 5, 9, 2; rendezve: 1, 2, 2, 2, 2, 2, 3, 3, 4, 4, 4, 4, 5, 5, 5, 6, 7, 8, 9, 9. c) Pldul: 4, 6, 5, 6, 0, 4, 8, 5, 3, 6, 0; rendezve: 0, 0, 3, 4, 4, 5, 5, 6, 6, 6, 8. 2. K1 Hatrozzuk meg a kvetkez adatsorok tlagt, mduszt, medinjt! a) 1, 1, 3, 5, 3, 8, 0, 2, 5, 4, 5, 6, 6, 1; b) 1, 1, 1, 1, 1, 1, 1, 2, 2, 2, 2, 2, 2, 2; c) 1, 2, 3, 4, 5, 6, 7, 8, 9, 10, 10, 10, 10. a) tlag: 50 . 3,57 ; mdusz: 1 s 5; medin: a 3 s a 4 szmtani kzepe: 3,5. 14 b) tlag: 1,5; mdusz: 1 s 2; medin: az 1 s a 2 szmtani kzepe: 1,5. c) tlag: 85 . 6,54 ; mdusz: 10; medin: 7. 13 3. E1 Egy adatsor tlaga 4. Van kztk kt egyenl szm. Ha az egyiket kihagyjuk, akkor a maradk szmok tlaga 4,2, ha a msikat is kihagyjuk, akkor 4,5 lesz a maradk szmok tlaga. a) Hny szm volt eredetileg? b) Mi volt a kt egyenl szm? Ha n + 2 szm volt s x jelli az elhagyott szmokat, akkor a kapott tlagokbl a szmok sszege: 4]n + 2g = 4,2 ]n + 1g + x = 4,5 n + 2x . Ebbl 4n + 8 = 4,2 n + 4,2 + x = 4,5 n + 2x. (4n + x) -et kivonva mindegyik kifejezsbl: 8 - x = 0,2 n + 4,2 = 0,5 n + x . A jobb s a bal oldal sszege a kzps kifejezs ktszerese: 0,5 n + 8 = 0,4 n + 8,4 . Ebbl n = 4 . Visszahelyettestve az eredeti egyenletekbe x = 3 addik. Eredetileg n + 2 = 6 szm volt, kztk kt 3-as, ezeket hagytuk el. 4. K2 Egy tanul 10 osztlyzatnak tlaga 3,5. a) Legfeljebb hny elgtelenje lehet? b) Legalbb hny jelese van? c) Legfeljebb hny jelese van? d) Biztos-e, hogy van kzepese? A jegyek sszege 35. a) Akkor lesz a legtbb egyese, ha minden ms osztlyzata a lehet legnagyobb. Ha n darab 1-ese van, akkor a maradk 10 - n jegy sszege ha mind 5-s legfeljebb 5 $ ]10 - ng lehet. Msrszt az sszegk ppen 35 - n . Eszerint a legnagyobb olyan n szmot keressk, amelyre mg 5 $ ]10 - ng $ 35 - n . Ebbl 15 $ 4n , vagyis n # 3,75 . Mivel n csak egsz szm lehet, a legnagyobb ilyen n a 3. 3 $ 1 + 6 $ 5 + 1 $ 2 = 35 . b) Nem biztos, hogy van jelese, mert lehet, hogy 5-5 ngyese, illetve hrmasa van.

9 .

V F O LYA M

110 MATEMATIKA

V I I I . S TAT I S Z T I K A

c) Ha n darab jelese van, akkor a maradk 10 - n jegy sszege 35 - 5n . Akkor lesz a legtbb jelese, ha a tbbi osztlyzat minl kisebb. A maradk osztlyzatok sszege legalbb 10 - n . A legnagyobb olyan n szmot keressk, amelyre 10 - n # 35 - 5n . Vagyis amelyre 4n # 25 , n # 6,25 . n = 6 a legnagyobb ilyen egsz szm. 6 $ 5 + 3 $ 1 + 1 $ 2 = 35 . d) Az elzekben lttuk, hogy egyltaln nem kell, hogy kzepese legyen. 5. K2 Ksztsnk olyan t adatbl ll adatsort, amelynek medinja s mdusza egyarnt 3, az tlaga a) 2; b) 3; c) 4! a) Pldul: 0, 1, 3, 3, 3; b) pldul: 3, 3, 3, 3, 3; c) pldul: 3, 3, 3, 4, 7.

6. K2 Ksztsnk olyan t adatbl ll adatsort, amelynek medinja s tlaga egyarnt 3, a mdusza a) 2; b) 3; c) 4! Ha adott a mdusz s a medin, akkor az a) esetben 2, 2, 3 szerepel a szmok kztt, a c) esetben 3, 4, 4. A b) esetben ennl kevesebbet tudunk. a) Pldul: 2, 2, 3, 3,5, 4,5; b) pldul: 3, 3, 3, 3, 3; c) pldul: 1,5, 2,5, 3, 4, 4. 7. K2 Ksztsnk olyan t adatbl ll adatsort, amelynek mdusza s tlaga egyarnt 3, a medinja a) 2; b) 3; c) 4! a) Ilyen adatsor nincs, mert ha a kzps elem 2 s a mdusz 3, akkor a # b # 2 # 3 = 3 lehet az t szm. Ezek tlaga azonban kisebb, mint 3. b) Pldul: 3, 3, 3, 3, 3. c) Ilyen adatsor sincs, mert ha kzps elem 4 s a mdusz 3, akkor 3 = 3 # 4 # a # b lehet az t szm. Ezek tlaga azonban nagyobb, mint 3.

2. Adatok szemlltetse
1. K1 a) Olvassuk le a grakon adatait, s rjuk tblzatba ket! 2006-ban az 1000 fre jut szemlygpkocsik szma az egyes orszgokban:
600 500 400 300 200 100 10 rorszg Ausztrlia Kna Nagy-Britannia Nmetorszg Zimbabwe Banglades Indonzia Brazlia Omn India USA

b) Mit gondolsz, Magyarorszgon hny szemlygpkocsi jut 1000 fre? Nzz utna az interneten!
9. V F OLYAM

V I I I . S TAT I S Z T I K A

MATEMATIKA 111

a) Bangladesben Knban Indiban Indonziban Zimbabwban Brazliban Omnban rorszgban Nagy-Britanniban Ausztrliban Nmetorszgban az Egyeslt llamokban

1 2 4 5 10 90 110 300 460 480 500 600

b) Magyarorszgon ez az rtk krlbell 200 lehet. 2. K1 a) Olvassuk le a grakon adatait, rendezzk tblzatba ket!

Karib-tenger Bering-tenger Dl-knaitenger Fldkzitenger Jeges-tenger Indiai-cen Atlanti-cen Csendescen milli km2 10 50 100 150

b) Szemlltessk krdiagramon az arnyokat! c) Keressk meg, hogy a Balaton felszne hny ngyzetkilomter! Hny fokos kzpponti szggel rajzolhatnnk a krdiagramra? a) A grafikonrl leolvashat rtkek: 1. Csendes-cen 165 000 000 km2 2. Atlanti-cen 82 000 000 km2 3. Indiai-cen 73 000 000 km2 4. Jeges-tenger 14 000 000 km2 5. Fldkzi-tenger 2 500 000 km2 6. Dl-knai-tenger 2 500 000 km2 7. Bering-tenger 2 500 000 km2 8. Karib-tenger 2 000 000 km2 b) Milli ngyzetkilomterben szmolva az egyttes terlet 343,5. o Egymilli ngyzetkilomterhez tartoz szg: 360 . 1,05o . 343,5 c) Krlbell 595 km2. Ez a tbbi vzfellet arnyban krlbell 17 . A 360-os krben nagyjbl 0,0006o = 2m . 10 000 000

9 .

V F O LYA M

112 MATEMATIKA

V I I I . S TAT I S Z T I K A

3. K1 A tblzatban a gerincesekre vonatkoz adatok szerepelnek! Emlsk Madarak Hllk Ktltek Halak 4000 faj 9000 faj 6000 faj 4000 faj 21 000 faj

a) Szemlltessk az adatokat vonaldiagramon! b) Szemlltessk a gerinces fajok megoszlsi arnyt szalagdiagramon! a)

Halak Ktltek Hllk Madarak Emlsk 1


b)

10

15

20

ezer faj

4. K1 A tblzatban a gerinctelenekre vonatkoz adatok szerepelnek! Tsksbrek zeltlbak Puhatestek Gyrsfrgek Hengeresfrgek Laposfrgek Csalnozk Szivacsok 5500 faj tbb mint 1 000 000 faj 45 000 faj 7000 faj 12 000 faj 10 000 faj 7000 faj 10 000 faj

a) Szemlltessk az adatokat vonaldiagramon! b) Szemlltessk a gerinctelen fajok megoszlsi arnyt szalagdiagramon!

9.

V F OLYAM

V I I I . S TAT I S Z T I K A

MATEMATIKA 113

a) Ha arnyosan szeretnnk szemlltetni az adatokat, akkor az zeltlbak nagy szma miatt a tbbi adatot nem lehet leolvasni:

fajok szama 1 000 000 900 000 800 000 700 000 600 000 500 000 400 000 300 000 200 000 100 000 0 1. 2. 3. 4. 5. 6. 7. 8. llatcsoport

Ha viszont a tbbi adatot szeretnnk preczebben leolvasni, akkor az zeltlbak pontos brzolsrl kell lemondanunk:

fajok szama 50 000 45 000 40 000 35 000 30 000 25 000 20 000 15 000 10 000 5000 0 1. 2. 3. 4. 5. 6. 7. 8. llatcsoport

b) Az zeltlbak miatt ez a diagram nem lesz ltvnyos s jl hasznlhat. Az zeltlbak nlkl a szalagdiagram gy nz ki:

hengeresfergek

esb or uek tusk

puhatestuek

usf rgek gyur

laposfergek

csalanoz ok

szivacsok

9 .

V F O LYA M

114 MATEMATIKA

V I I I . S TAT I S Z T I K A

5. K2 Ksztsnk tbbfle grakont a kvetkez adatok alapjn! A Fld tavai Kaszpi-tenger Fels-t Viktria-t Huron-t Michigan-t Aral-t Tanganyika-t Bajkl-t Nagy-Medve-t Malawi-t Felszne (km2) 371 000 82 400 69 500 59 600 57 454 37 000 32 900 31 500 31 328 28 930 Mlysge (m) 995 406 85 228 281 68 1480 1620 413 614

Pldul a tavak felszne szalagdiagramon:

Kaszpi-tenger

A mlysgk oszlopdiagramon:

mlysg (m) 1500

1000

500 100 Viktria-t Huron-t Tanganyika-t Michigan-t Nagy-Medve-t Aral-t Kaszpi-tenger Bajkl-t Malawi-t Fels-t

9.

V F OLYAM

Aral-t Tanganyika-t Bajkl-t Nagy-Medve-t Malawi-t

Michigan-t

Viktria-t

Huron-t

Fels-t

V I I I . S TAT I S Z T I K A

MATEMATIKA 115

3. A ktarc statisztika
1. K1 Keressnk jsgban, interneten statisztikkat! Beszljk meg az osztlyban, hogy melyik mit jelent, mirl szl! Dntsk el, hogy van-e benne megtveszt graka vagy adat! 2. K1 Ksztsnk olyan slyozst, amelyekre az 1, 2, 3, 4 sorrendben vett szmok s a 2, 3, 6, 10 sorrendben vett szmok slyozott szmtani kzepe a) az els esetben nagyobb; b) a msodik esetben nagyobb! a) Ilyen nem lehet, mert a slyok pozitvak, s a szmok rendre nagyobbak a msodik esetben. b) Sok megolds van. Pldul: 10, 5, 2, 1. 3. K1 Ksztsnk olyan slyozst, amelyekre az 1, 2, 3, 4 sorrendben vett szmok s a 10, 6, 3, 1 sorrendben vett szmok slyozott szmtani kzepe a) az els esetben nagyobb; b) a msodik esetben nagyobb; c) egyenl! a) Sok megolds van. Pldul: 1, 1, 1, 10. b) Sok megolds van. Pldul: 1, 1, 1, 1. c) 1, 3, 1, 7.

9 .

V F OLYA M

You might also like